You are on page 1of 134

Banking & Economy PDF

Report Errors in the PDF - ebooks@affairscloud.com Copyright 2014-2020 @ AffairsCloud.Com 1


Banking & Economy PDF

This is Paid PDF provided by AffairsCloud.com, Our team is working hard


in back end to provide quality PDF. If you not buy this paid PDF
subscription plan, we kindly request you to buy pdf to avail this service.
Help Us To Grow & Provide Quality Service

CURRENT AFFAIRS PDF PLANS


Current Affairs PDF 2020: Weekly, Study, Pocket and Q&A PDF’s

Current Affairs PDF 2019: Study, Pocket and Q&A PDF’s

Current Affairs PDF 2018: Study, Pocket and Q&A PDF’s

CRACK HIGH LEVEL PDF PLANS


Crack High Level: Puzzle & Seating Arrangement Questions PDF 2019

Crack High Level: English Language Questions PDF 2019

Crack High Level: Data Interpretation Questions PDF 2019

Suggestions & Feedback are welcomed

Support@affairscloud.com

Report Errors in the PDF - ebooks@affairscloud.com Copyright 2014-2020 @ AffairsCloud.Com 2


Banking & Economy PDF

Banking, Finance & Economy PDF 2020 - July

Table of Contents
Banking, Finance & Economy News: July 2020 ............................................................................................................. 4
Banking, Finance & Economy Q&A: July 2020 ............................................................................................................ 86

Report Errors in the PDF - ebooks@affairscloud.com Copyright 2014-2020 @ AffairsCloud.Com 3


Banking & Economy PDF

Banking, Finance & Economy News: July 2020


Harsimrat Kaur Badal virtually launched Rs 10,000 cr PM-FME Scheme; Operation Greens scheme
extended to 18 more fruits, vegetables
As a part of Rs 20 Lakh crore “Atmanirbhar Bharat Abhiyan” to tackle the COVID-19 impact, Minister for Food
Processing Industries (FPI) Harsimrat Kaur Badal launched Centrally Sponsored Prime Minister Formalisation of
Micro food processing Enterprises (PM FME) scheme named “Sapno ki Udaan” to be implemented over a period of
five years from 2020-21 to 2024-25. It was launched virtually from the village Badal, Bathinda in Punjab in the
presence of Minister of State (MoS), FPI Rameswar Teli.
• With an outlay of Rs 10,000 crore, this scheme for the unorganized food processing sector would generate
total investment of Rs 35,000 crore and 9 lakh skilled and semi-skilled employment and benefit 8 lakh units
through access to information, training, better exposure and formalization.
• Notably, all the processes of the Scheme would take place on the management information system (MIS).
About PM FME scheme:
Aim– To take “local brand to global”
Objective– To provide financial, technical and business support for up gradation of existing micro food processing
enterprises.
Sharing of expenditure– The expenditure under the scheme would be shared in 60:40 ratio between Central and
State Governments, in 90:10 ratio with North Eastern and Himalayan States, 60:40 ratio with Union Territories (UTs)
with legislature and 100% by Centre for other UTs.
Approach to be used- One District One Product (ODOP)
ODOP will be adopted for this scheme under which the states will identify food product (specialized or perishable or
cereal) for a district on the basis of availability of raw material.
• Preference would be given to those enterprises which produce ODOP products and will also get support for
common infrastructure, branding & marketing.
• The Scheme also places focus on waste to wealth products, minor forest products and Aspirational Districts.
35% Credit-linked capital subsidy for upgradation of micro food processing units
Support to Individuals– Existing individual micro food processing units desirous of upgradation of their units could
avail credit-linked capital subsidy of 35% of the eligible project cost with a maximum ceiling of Rs.10 lakh per unit.
Support to FPOs/SHGs/Cooperatives– Seed capital of Rs. 40,000/– per Self Help Group (SHG) member would be
provided for working capital and purchase of small tools.
• Farmer producer organisations (FPOs) / SHGs/ producer cooperatives would be provided a credit linked grant
of 35% for capital investment along the value chain.
Infrastructure and branding support– Support would be provided through credit linked grant @ 35% for
development of common infrastructure. Support for marketing & branding would be provided to develop brands for
micro units and groups with 50% grant at State or regional level which could benefit large number of micro units in
clusters.
NIFTEM & IIFPT will be supported by MoFPI for training of units
National Institute of Food Technology Entrepreneurship and Management (NIFTEM) in Sonipat, Haryana and Indian
Institute of Food Processing Technology (IIFPT) in Tamil Nadu, two academic and research institutions under
Ministry of Food Processing Industries (MoFPI) along with State Level Technical Institutions would be supported by
ministry for training of units, product development, appropriate packaging and machinery for micro units.
• Apart from above, a free skill training programme will be launched in the next two months for scheduled
castes (SC) and scheduled tribes (ST) food processors. A three-month certificate course will provide training in
41 identified food processing topics.The training will be conducted in collaboration with NIFTEM and Food
Industry Capacity & Skill Initiative (FICSI).
• The participant handbooks and the facilitator’s guide created by the ministry through NIFTEM will be
converted into e-learning format with suitable digital content.

Report Errors in the PDF - ebooks@affairscloud.com Copyright 2014-2020 @ AffairsCloud.Com 4


Banking & Economy PDF
Operation Greens scheme extended to 18 more fruits, vegetables
The ministry also extended the operation greens from TOP (Tomato-Onion-Potato) crops to all perishable fruits and
vegetables with an objective to protect the growers of fruits and vegetables from making distress sale due to
lockdown and reduce post-harvest losses.
• The duration of the scheme will be for the period of six months from the date of notification on June 11, 2020.
• The benefit has been extended to additional 10 fruits (Mango, Banana, Guava, Kiwi, Lichi, Papaya, Citrus,
Pineapple, Pomegranate, Jackfruit) and 8 vegetables (French beans, Bitter Gourd, Brinjal, Capsicum, Carrot,
Cauliflower, Chillies (Green) and Okra).
• With this farmers and processors can now get the benefit of 50% subsidy on storage and transportation of 18
more fruits and vegetables from surplus production areas to major consumption centres.
About Ministry of Food Processing Industries (MoFPI):
Harsimrat Kaur Badal constituency– Bathinda, Punjab
MoFPI Scheme– Pradhan Mantri Kisan SAMPADA Yojana (Scheme for Agro-Marine Processing and Development of
Agro-Processing Clusters).

GoI and World Bank Sign Agreement to Increase Access to Affordable Housing in Tamil Nadu
On 29 June, The World Bank and the government of India signed an agreement along with the Government of Tamil
Nadu to help the low-income groups in the state of Tamil Nadu to get access to affordable housing.
Agreements signed:-
Legal agreements of two projects were signed to strengthen the state’s housing sector policies, institutions, and
regulations.
• First Tamil Nadu Housing Sector Strengthening Programme worth $200 million(Rs 1,510 crore),
• Tamil Nadu Housing and Habitat Development Project worth $50 million (Rs 377 crore).
The loans from the IBRD (International Bank for Reconstruction and Development) have a maturity period of 20
years (including grace period: 3.5 years)
Agreements Signed by:
World Bank: Mr Junaid Kamal Ahmad, Country Director (India)
Government of India: Sameer Kumar Khare, Additional Secretary, Department of Economic Affairs, Ministry of
Finance
Government of Tamil Nadu: Hitesh Kumar S. Makwana, Principal Resident Commissioner.
First Tamil Nadu Housing Sector Strengthening Programme:-
A loan for $200 million has been granted from IBRD.
It is a series of two single-tranche operations.
i) It supports the Government’s efforts to increase the availability of affordable housing by shifting the role of the
state to an enabler.
ii) The second operation looks to deepen these measures to make affordable housing more efficient and inclusive.
Note:- The Tamil Nadu’s Urban population is expected to increase to 63 percent by 2030.
Tamil Nadu Housing and Habitat Development Project:-
A loan for $50 million has been approved from IBRD.
• The project helps support innovations in housing finance and strengthen housing sector institution
• It Finances to Tamil Nadu Shelter Fund(TNSF)
Tamil Nadu Shelter Fund (TNSF)
• It is an innovation in housing finance in India and provides equity contribution of $35 million.
• The project strengthens the capacity of housing institutions including Tamil Nadu Slum clearance board,
Chennai Metropolitan Development Authority and Tamil Nadu Infrastructure Fund Management Corporation
Limited, asset Management company of TNSF.
About World Bank:
Headquarters: Washington D.C., United States
President: David R. Malpass

Report Errors in the PDF - ebooks@affairscloud.com Copyright 2014-2020 @ AffairsCloud.Com 5


Banking & Economy PDF
Swiggy in Partnership with ICICI Bank Launches an Industry- First Instant Digital Wallet, ‘Swiggy
Money’
Swiggy has launched its own digital wallet, ‘Swiggy Money’ in partnership with ICICI Bank to enable a ‘single-click
checkout experience’ on its platform. It is an industry- first instant digital wallet. The Swiggy Money is powered by
ICICI Bank’s ‘insta wallet service’ which is built on a cloud platform with Application Programming Interface
(API)integration.
Gist about the digital wallet
Usage
i.Swiggy customer can immediately use the wallet if he/she is an existing ICICI Bank customer.
ii.Non-ICICI Bank customers can also use this instantly after providing details of government ID to ICICI Bank
Benefits for users
i.Discounts &Instant withdrawal-The users can avail discounts & withdraw money instantly and use the money for
easy checkouts and hassle-free payment processing on future food orders
ii.Top up-They can top-up their wallet using various banking instruments and can purchase through single
click without multiple authentications.
iii.Split-Pay option– If the order value exceeds the wallet balance, the users will be given a ‘split-pay’ option. With
this option the users can make payment through a combination of money from their wallet and another payment
source/instruments to complete the transaction.
Key info
This service is enabled for restaurant orders & will be soon available for paying orders made through Grocery, Genie,
and POP offerings
Note– For ICICI bank it is the 3rd offering, alongwith with Swiggy. Ola and Amazon India have also adopted a ‘digital
wallet’ strategy.
About ICICI bank:
Headquarters– Mumbai, Maharashtra
Managing Director & Chief Executive Officer (CEO)- Sandeep Bakhshi
About Swiggy:
Headquarters– Bengaluru, Karnataka
Co-Founder– Sriharsha Majety

Former RBI Governor Chakravarthi Rangarajan Receives the First Prof. P. C. Mahalanobis Award on
National Statistics Day
On 29th June 2020, On account of National Statistics Day The Ministry of Statistics and Programme Implementation
(MoSPI) presented the first Prof. P. C. Mahalanobis National Award in Official Statistics’2020 to Dr. Chakravarthi
Rangarajan, Former Governor, Reserve Bank of India for his contribution to the National Statistical System (NSS) in
India.
Note:
The virtual event of the National Statistics Day was streamed live over various social media platforms.
Key Points:
i.Rangarajan received the Lifetime Achievement award for his contribution in providing solutions in the issues
related to the estimation of national income.
ii.In his acceptance speech he mentioned about the importance of credibility of official statistics.
iii.He headed the Prime Minister’s Advisory Council and he was the Chairman of the National Statistical Commission
to review the statistical system.
iv.Chief statistician Pravin Srivastava mentioned that the MoSPI is developing a generalised survey solution “eSigma”
to capture data by mobile devices which will be supplemented by the telephonic and web-based interfaces.
Prof. P.V. Sukhatme National Award:
For their contribution in the Statistics sector the Prof. P.V. Sukhatme National Award in Statistics 2020 is jointly
presented to,

Report Errors in the PDF - ebooks@affairscloud.com Copyright 2014-2020 @ AffairsCloud.Com 6


Banking & Economy PDF
• Dr Arvind Pandey, former Director National Institute of Medical Statistics (NIMS), Indian Council of Medical
Research (ICMR) and
• Dr. Akhilesh Chandra Kulshreshtha, Ex-Addl. Director General, MoSPI, Government of India.
Key People:
• Rao Inderjit Singh – Union Minister of State (Independent Charge) of Ministry of Statistics and Programme
Implementation (MoSPI) and Ministry of Planning
• Dr. Bibek Debroy – Chairman of the Prime Minister’s Economic Advisory Council and President of Indian Statistical
Institute
• Prof. Bimal Kumar Roy – Chairman of the National Statistical Commission (NSC)
• Dr Sangita Reddy – President of the Federation of Indian Chambers of Commerce & Industry and Joint General
Managing Director of Apollo Hospitals Enterprises Ltd.
• Prof. Sanghamitra Bandyopadhyay – Director of Indian Statistical Institute
About Ministry of Statistics and Programme Implementation (MoSPI) &Planning:
Minister of State(Independent Charge) – Rao Inderjit Singh

WTO Dispute Settlement Body Sets Up Panel Against India’s ICT Duties Based on EU’s Complaint
The World Trade Organization (WTO) Dispute Settlement Body (DSB) has established a panel after the
European Union (EU) has dragged India into the Dispute Settlement System of WTO for the second time against
import duties on ICT (information and communication technology) products, including mobile phones, cameras,
headphones and earphones, imposed by the country.
The DSB meeting with three members,which examined the requests, took place on June 29, 2020.
• The request for the establishment of a panel follows government-to-government formal consultations, which
took place on 21 May 2019 with India.
About the dispute:
India had previously stated in the WTO under a legally binding commitment (as part of the IT Agreement signed in
December, 1996) that it would not charge the abovesaid products.
Now India is levying a fee upto 20 % on them & it is clearly a violation of WTO rules by India. These charges will affect
the EU’s exports of 400 million euros annually.
Background:
First requests for panels on India’s ICT tariffs were from Japan and Chinese Taipei, but New Delhi, however refused to
accept any adverse ruling of the panel in February 2020 after its hearing process is over if the functioning of the
Appellate Body of the WTO is not restored by then.
Proceedings after the panel setup:
Following the panel establishment, India can’t block the request for a second time as per WTO rules.
About WTO Dispute Settlement Mechanism:
-The dispute settlement system of the WTO seeks to settle any trade dispute initially through consultation between
the member countries concerned. If this remedy is not successful, then the matter goes to a Dispute Panel.
– The decision of the dispute panel is final, but an appeal against its decision can be made to the Appellate Body-AB.
– The decisions of the Dispute Panel are reviewed by the Appellate Authority and after the report is submitted by the
Appellate Authority, it is final and binding on the 164 Member States.
About World Trade Organisation (WTO):
Headquarters– Geneva, Switzerland
Director-General- Roberto Azevêdo

World Bank Approves USD 750 Million MSME Emergency Response Program for India
On June 30, 2020 The World Bank approved a USD 750 million (about Rs.5600 Crore) Micro, Small, and Medium
Enterprises(MSMEs) Emergency Response Program to support severely impacted MSMEs in India due to COVID-19
crisis. The International Bank for Reconstruction and Development(IBRD) will provide the loan.

Report Errors in the PDF - ebooks@affairscloud.com Copyright 2014-2020 @ AffairsCloud.Com 7


Banking & Economy PDF
Major Highlights
i.The World Bank’s MSME Emergency Response program will address the immediate liquidity and credit needs of
around 1.5 million applicable MSMEs.
ii.This program will help them to handle current shock and protect millions of jobs.
Maturity Period of the loan– The loan has a maturity of 19 years including a 5-year grace period.
Significance of MSME sector– It is the backbone of Indian economy & contributes about 30 % to India’s GDP and
40% to exports. The sector employs about 150-180 million people.
World bank’s assistance to India
i.The World bank has lent USD 5.13 billion to India in the financial year 2020. It has distributed about four billion
dollars in FY 2020.
ii.Till now the World Bank has committed USD 2.75 billion to support India’s emergency COVID-19 response,
including the MSME project.
• First USD 1 billion emergency support was announced in April, 2020 to support India’s health sector.
• Another USD 1 billion project was approved in May, 2020 to increase cash transfers and food benefits to the
poor and vulnerable
Key Info
i.The World Bank group will distribute up to USD 160 billion to help more than 100 countries as a financial aid over
15 months, which will enable them to protect the poor and vulnerable, support businesses, and improve economic
recovery.
ii.This includes USD 50 billion of International Development Association(IDA) resources through grants and highly
concessional loans.
About World Bank:
Headquarters- Washington, D.C., United States
President- David R. Malpass

FDI Up by $19 bn in FY20; Portfolio Investment Down by $13.7 bn: IIP Data Released by RBI
In accordance with the India’s International Investment Position (IIP) data as at end-March 2020, released by Reserve
Bank of India (RBI), Foreign Direct Investment (FDI) in the country has raised by $19 billion while portfolio
investment declined by $13.7 billion during 2019-20.
Annual Variations (April-March, 2019-20)
International financial assets of Indian residents increased by US$ 73.9 billion due to the rise in reserve assets and
overseas direct investment by US$ 64.9 billion and US$ 13.0 billion, respectively, though other investments declined
marginally during the year.
International financial liabilities increased by US$ 16.3 billion as direct investment and other investments rose by
US$ 19.0 billion and US$ 11.0 billion, respectively.
Net claims of non-residents in India declined by US$ 57.6 billion in a year. On the other hand, these are lowered by
US$ 45.8 billion to US$ 379.3 billion in March 2020.
Ratio of International Financial Assets and Liabilities to GDP:
-The ratio of overseas financial assets to India’s Gross Domestic Product (GDP) in 2019-20 (at current market prices)
moved up to 26.5% in March 2020 (23.4% a year ago)
-The ratio of total claims of non-residents to GDP increased to 40.5 per cent in March 2020 (39.3 per cent a year ago).
-The ratio of net IIP to GDP improved to (-) 14.0% as at end-March 2020 [(-) 15.9%].
–The ratio of India’s international financial assets to international financial liabilities improved significantly to 65.4%
at end-March 2020 (62.1% a quarter ago; 59.5% a year ago).
Click Here for Official Link
About Reserve Bank of India (RBI):
Headquarters– Mumbai, Maharashtra
Formation– 1 April 1935
Governor– Shaktikanta Das

Report Errors in the PDF - ebooks@affairscloud.com Copyright 2014-2020 @ AffairsCloud.Com 8


Banking & Economy PDF
Deputy Governors– 4 (Bibhu Prasad Kanungo, Mahesh Kumar Jain, Michael Debabrata Patra, one is yet to be
appointed).

HDFC Bank Launches ‘e-Kisaan Dhan’ App for Farmers


On June 30, 2020 HDFC Bank has launched ‘e-Kisaan Dhan’ App for Farmers in India.
This app provides information and knowledge to farmers which will help them to meet the needs of the rural
ecosystem. With this app, farmers can access a bouquet of services in banking and agriculture, through their mobile
phones.
Gist about e-Kisaan Dhan app
i.Information– It will provide users information in regard to new government schemes and ways to access them.
ii.Services
• Value Added Services– The e-Kisaan Dhan app provides value added services like mandi prices, latest
farming news, weather forecast, information on seed varieties, SMS advisory, e-pashuhaat and Kisan TV among
others.
• Multiple Banking Services– Users can avail banking services, such as buying loans, opening bank accounts,
availing insurance facilities, calculate KCC loan eligibility online and acquiring Government social security
schemes.
• Traditional banking services– The app will help the farmers with the traditional banking services like
applying for loans, Fixed Deposits, Recurring Deposits, and Savings Accounts.
iii.Availability– The app can be downloaded from Google Play. It is now available in English only & will be soon
available in other Indian languages.
Key info- This app is part of the bank’s ‘Har Gaon Hamara’ initiative to reach rural and under-served area customers.
Note– HDFC Bank has distributed over 5 lakh agri loans and established 12 Krishi Dhan Vikas Kendras
About HDFC Bank:
Headquarters– Mumbai, Maharashtra
Chairman– Deepak S. Parekh
Managing Director– Aditya Puri

Fitch Slashes India’s Growth Forecast For FY22 to 8%; Projects Contraction at 5% For FY21
On June 30, 2020, Fitch Ratings Inc., an American credit rating agency in its June 2020 update of Global Economic
Outlook (GEO), has lowered India’s growth forecast for FY 2021-22 to 8% from 9.5 % projected in may 2020 due to
the toughest lockdown imposed by the central government to prevent the outbreak of Coronavirus (COVID-19).
However, Fitch has not made any change in its previous estimate of a – 5% in the economy during the current
financial year (FY 2020-21).
• Global Projections: Fitch has also predicted growth in the global economy at 4.9 % in 2021& in 2022 it is
estimated to be 3.4 %.
Past & future estimated growth rate:
The projected growth rate of Indian economy in 2019-20 is 4.2 % & will grow at 5.5 % in 2022-23.
The agency has forecast a 13.8 % decline in the economy in the July-September quarter 2020 as compared to 0.3 % in
April-June quarter 2020.
Other agencies estimates:
Earlier, S&P (Standard & Poor) had forecast – 5% in the Indian economy in the current financial year (FY21) & an
increase of 8.5 % in FY 22.
At the same time, rating agency Moody’s has forecast – 4% in the economy in the 2020 due to the COVID-19 epidemic
& around 8.7% in the 2021.
About Fitch Ratings:
Headquarters– New York, US
President– Ian Linnell

Report Errors in the PDF - ebooks@affairscloud.com Copyright 2014-2020 @ AffairsCloud.Com 9


Banking & Economy PDF
Rajkiran Rai G, Union Bank of India MD & CEO Term Extended for 2 years
Government extends the term of Rajkiran Rai G as the Managing Director(MD) and Chief Executive Officer(CEO) of
Union Bank of India(UBI) by 2 years till May 31, 2022.
About Rajkiran Rai G
i.The 3 year term of Rajkiran Rai G as MD & CEO of UBI was to end on June 30, 2020. He took charge of UBI on July 1,
2017.
ii.Prior to the appointment in UBI, he was an executive director at Oriental Bank of Commerce.
iii.He has over 3 decades of experience in the banking field.
About UBI:
Headquarters- Mumbai, Maharashtra
Chairman & Part-Time Non-Official Director– Kewal Handa

Indian marine fish landings rose 2.1% to 3.56mt in 2019: CMFRI; Tamil Nadu tops
In accordance with the report “Annual Marine Fish Landings in India for 2019” by Central Marine Fisheries Research
Institute (CMFRI), India’s marine fish production has increased marginally by 2.1% to 3.56 million tonnes (mt) in
2019 as compared to 3.49 mt in 2018.
• As per the report, Tamil Nadu grabbed the top in annual fish production with 7.75 lakh tonnes followed by
Gujarat (7.49 lakh tonnes), which had been holding the first position earlier, and Kerala (5.44 lakh tonnes)
which retained the third position.
• The estimated value of fish landings in the country last year was Rs 60,881 crore with 15.6% increase over
2018.
• It should be noted that the term “Landing” in the report means getting fish out of the water and brought to the
land.
• Notably, in the global front, India is ranked 3rd after China and Indonesia in marine fish production.
Key Points from report:
-The red toothed triggerfish, a commercially unimportant fish, became the most landed resource (2.74 lakh
tonnes). This fish has little demand in domestic markets and is mostly caught for the purpose of feed mills.
-The second highest landings was ribbon fish (2.19 lakh tonnes), followed by penaeid prawns (1.95 lakh tonnes)
and non-penaeid prawns (1.80 lakh tonnes).
-The Indian mackerel landings, which were in the first spot in 2018, declined by 43% in 2019.
–Scenario of other States in fish landing: States such as West Bengal (55 per cent), Andhra Pradesh (34 per cent),
Odisha (14.5 per cent), Karnataka (11 per cent) and Tamil Nadu (10.4 per cent) recorded an increase in landings,
while the marine fish catch decreased in Maharashtra (32 per cent), Goa (44 per cent) and Kerala (15.4 per cent),
compared to the previous year.
–Cyclone effects: As per report, 8 cyclonic storms in and around India, of which six turned out to be severe cyclones
(Fani in April, Vayu in June, Hika in September, Kyarr in October, Maha in October-November and Bulbul in October-
November) affected the fishing calendar days more adversely on the west coast of the country.
About Central Marine Fisheries Research Institute (CMFRI):
The Fishery Resources Assessment Division of the CMFRI estimated the annual marine fish landings through its
online data collection system.
Director– Dr. A. Gopalakrishnan
Headquarter– Kochi, Kerala

DFPD estimated Rs 1,48,938 cr as cost for distribution of foodgrains and pulses under PMGKAY
during April – Nov 2020
On June 30, 2020, Prime Minister Shri Narendra Modi, during televised address to the nation, announced the
extension of Pradhan Mantri Garib Kalyan Ann Yojana (PMGKAY) from July, 2020 till the end of November 2020. Click
Here to Read.
• Now, on July 1, 2020, Department of Food and Public Distribution (DFPD) under Ministry of Consumer
Affairs, Food and Public Distribution has worked out estimated cost for distribution of food grains (Rice and

Report Errors in the PDF - ebooks@affairscloud.com Copyright 2014-2020 @ AffairsCloud.Com 10


Banking & Economy PDF
Wheat) and pulses under PMGKAY, which will be Rs 1,48,938 crore approx for a period between 8
months from April to November 2020.
• Notably, Govt. of India is bearing the entire expenditure towards this Scheme.
Major Breakdown of Cost:
The estimations were made under the Targeted Public Distribution System (TPDS).
-An estimated cost of 32 million tonne for distribution of foodgrains (Rice and Wheat) from April to November 2020
will be approx. Rs 1,22,829 crores.
-An estimated expenditure for distribution of pulses during the period April-November 2020 will be Rs 11,800 crore
approx.
-An expenditure of Rs 1,930 crore would be required to be met towards transportation & handling and Fair Price
Shops (FPSs) dealers’ margins etc.
-In addition to above, the estimated cost for distribution of foodgrains to migrant labours for the period of two
months is Rs 3,109.52 crore.
-The foregone cost of Central Issue Price for foodgrains under the National Food Security Act (NFSA) is approximately
Rs.11,200 crore.
About Pradhan Mantri Garib Kalyan Ann Yojana:
Initially launched from April, 2020 for 3 months, the scheme aims to provide 5 kg free wheat or rice per month along
with 1 kg free whole chana (pulses) to each family per month covering 80 crore poor people which are severely
impacted by the lockdown to contain COVID-19 spread. Now, it has extended for 5 months till November 2020 i.e. the
overall period of the scheme becomes 8 months.
About Ministry of Consumer Affairs, Food and Public Distribution:
Union Minister– Ram Vilas Paswan (Constituency-Bihar)
Departments– Department of Food & Public Distribution (DFPD) and Department of Consumer Affairs (DCA)
Secretary (Food & Public Distribution)– Sudhanshu Pandey

GoI approved special liquidity scheme for NBFCs and HFCs through SBICAP as SPV
On July 1, 2020, the Government of India (GoI) has approved a scheme for improving the short-term liquidity position
of non-banking finance companies (NBFCs)/housing finance companies (HFCs) through a Special Purpose Vehicle
(SPV) in the form of Special Liquidity Scheme (SLS) Trust set up by SBI Capital Markets Limited (SBICAP), a
subsidiary of the State Bank of India (SBI). Means, SBICAP will manage this operation.
• This SLS of Rs. 30,000 crore was announced by Union Minister for Finance & Corporate Affairs Smt Nirmala
Sitharaman on 13th March 2020.
• It should be noted that the finances provided to the NFBCs/HFCs under this scheme should be used to repay
existing liabilities and not to expand assets.
Key Points:
Period of scheme: The Scheme will remain open for 3 months for making subscriptions by the Trust i.e.
till September 30, 2020.
Period of lending: The period of lending by the Trust shall be for a period of upto 90 days.
Instruments used in this scheme: Commercial papers (CPs) and Non-Convertible Debentures (NCDs).
Eligibility:
Any NBFC including Microfinance Institutions registered with RBI under the Reserve Bank of India Act, 1934
(excluding those registered as Core Investment Companies) and any HFC registered with the National Housing Bank
(NHB) under the National Housing Bank Act, 1987which is complying with the following broad conditions will be
eligible to raise funding:
• Capital to Risk (Weighted) Assets Ratio (CRAR)/ Capital adequacy Ratio (CAR) of NBFCs/HFCs should not be
below the regulatory minimum, i.e., 15% and 12% respectively as on March 31, 2019.
• Net Non-performing asset (NPA) is less than 6% as on 31.03.2019.
• Net profit in at least one of the two preceding financial years (i.e. 2017-18 and 2018-19)
• Rated as investment grade by a rating agency

Report Errors in the PDF - ebooks@affairscloud.com Copyright 2014-2020 @ AffairsCloud.Com 11


Banking & Economy PDF
• Not reported under SMA (Special Mention Accounts)-1 or SMA-2 category by any bank for their borrowing
during the period one year prior to 01.08.2018.
How will this scheme work?
RBI will provide funds for the Scheme by subscribing to government guaranteed special securities issued by the
Trust. GoI will provide an unconditional and irrevocable guarantee to the special securities issued by the Trust.
Click Here for Official Link
About Reserve Bank of India (RBI):
Headquarters– Mumbai, Maharashtra
Formation– 1 April 1935
Governor– Shaktikanta Das
Deputy Governors– 4 (Bibhu Prasad Kanungo, Mahesh Kumar Jain, Michael Debabrata Patra, one is yet to be
appointed).

ICICI bank Launched a Facility, ‘Insta Loans against Mutual Funds’, in Partnership with Computer
Age Management Services
On July 1, 2020 ICICI Bank in partnership with Computer Age Management Services(CAMS) launched ‘Insta Loans
against Mutual Funds’, a facility for retail customers to avail loans upto Rs 1 crore instantly by pledging their
holdings in both debt and equity Mutual Funds(MFs).
Gist about Insta Loans against Mutual Funds
Key info
i.The facility is an extension of the Insta Loan Against Shares(LAS).
ii.It is a full digital and paperless facility. It empowers customers to get this loan as an overdraft (OD) within a few
minutes without having to visit a branch and submit physical documents.
Features
Access to instant liquidity– It paves way for the customers to get access to instant liquidity by leveraging their
existing mutual fund portfolio, without selling them.
Pros of MFs- Customers can avail the loan against a vast range of debt and equity schemes of mutual funds serviced
by CAMS and approved by the Bank
Loan amount- Minimum loan amount for debt and equity schemes of MFs is Rs. 50,000. Maximum loan amount for
debt schemes of MFs is Rs 1 crore and for equity schemes of MFs is Rs. 20 lakh
Competitive margin– Minimum 50% for equity and 20% for debt MFs
No fixed EMI– The customers are not required to pay fixed EMI as the loan is available as an OD.
Interest Payment– Customers have to pay interest on the limit used and the number of days it is used.
No foreclosure charges– The customers can repay the principal at their own convenience, without any foreclosure
charges
About ICICI bank:
Headquarters– Mumbai, Maharashtra
Managing Director(MD) & Chief Executive Officer(CEO)– Sandeep Bakhshi

V Suryanarayanan Becomes the New MD of Chola MS General Insurance


On 30th June 2020, The Cholamandalam MS General Insurance Company promoted V Suryanarayanan, President and
COO of the company as the New Managing Director (MD) with effect from 1st July 2020 succeeding SS
Gopalarathnam.
i.SS Gopalarathnam, who joined the Murugappa Group in 1979 served the company for around 41 years in finance
and marketing function retired on July 1st and Suryanarayanan will take over his position.
ii.SS Gopalarathnam, started Cholamandalam MS General Insurance in 2002 as a founding member.
iii.Suryanarayanan was a Chartered Accountant and Company Secretary served in financial services sector of the
Murugappa Group for the past 24 years.
iv.Suryanarayanan served in Cholamandalam Investments and Finance for 10 years and moved to Chola MS as the
CFO in 2006 and became the President and COO in April 2019.

Report Errors in the PDF - ebooks@affairscloud.com Copyright 2014-2020 @ AffairsCloud.Com 12


Banking & Economy PDF
About Cholamandalam MS General Insurance:
Cholamandalam MS General Insurance is a joint venture of Murugappa Group and Mitsui Sumitomo Insurance
Company Ltd (Japan).
Chairman– MM Murugappan
Managing Director– V Suryanarayanan
HeadOffice – Chennai, Tamil Nadu

Karnam Sekar, Managing Director & CEO of Indian Overseas Bank Retires
Karnam Sekar, The Managing Director (MD) and Chief Executive Officer(CEO) of Indian Overseas Bank(IOB), retired
on 30th June 2020.
Karnam Sekar’s Career:
i.Sekar joined the State Bank of India(SBI) as the Probationary officer in December 1983.
ii.He served as the deputy managing director and chief credit officer of State Bank of India and played a major role in
formulating loan policy of SBI.
iii.He served as the SBI’s Manager of Treasury in Johannesburg, South Africa.
iv.He has been the MD and CEO of Dena Bank since 21st September 2018.
v.He joined IOB as an officer on special duty and a Director on 1st April 2019 after the merger of Dena Bank and
Vijaya Bank into Bank of Baroda.
vi.He was promoted as the MD and CEO succeeding Subramaniam Kumar on 1st July 2019.
About Indian Overseas Bank(IOB):
Tagline – Good People to Grow With.
Headquarter – Chennai, Tamil Nadu.

DAC approves Rs 38,900 cr capital acquisition of equipment for armed forces; 33 fighter jets to be
procured
Defence Acquisition Council (DAC) has given its consent for the capital acquisition of various platforms and
equipment required by the Indian Armed Forces, Army, Navy and the Air Force, worth Rs 38,900 crore. This decision
was taken during the DAC meet held on July 2, 2020 under the chairmanship of Raksha Mantri (Defence Minister)
Rajnath Singh.
• Notably, these approvals include acquisitions of Rs 31,130 crore from Indian industry signifying Prime
Minister (PM) Narendra Modi’s clarion call for ‘Atmanirbhar Bharat’ focused on indigenous design and
development.
Key Points:
-The equipments will be manufactured in India with the participation of Indian defence industry and several Micro,
Small and Medium Enterprises (MSMEs) as prime tier vendors.
-The indigenous content in some of these projects is up to 80 per cent of the project cost. A large number of these
projects have been made possible due to the Transfer of Technology (ToT) by Defence Research and Development
Organisation (DRDO) to the indigenous industry.
Projects to be developed for capital acquisition; Cost ranges Rs 20,400 crore
The cost of the design and development of the projects to be manufactured for the capital acquisition is in the range
of Rs 20,400 crore. The projects include the following:
• BMP armament upgrades and software defined radios for the Indian Army.
• Pinaka ammunition: Acquisition of Pinaka missile systems will enable raising additional regiments over and
above the ones already inducted. It is all-weather, indirect fire, free flight artillery rocket system.
• Long Range Land Attack Cruise Missile (LRLACM): Having a firing range of 1,000 kilometres to the existing
arsenal, LRLACM addition will bolster the attack capabilities of the Navy and the Air Force.
• Astra Missiles: Induction of Air-to-Air Missile (AAM) Astra, having Beyond Visual Range (BVR) capability, will
serve as a force multiplier to the strike capability of the Navy and Air Force.

Report Errors in the PDF - ebooks@affairscloud.com Copyright 2014-2020 @ AffairsCloud.Com 13


Banking & Economy PDF
DAC approved procurement of 33 fighter aircrafts & upgradation of existing 59 Mig-29 aircraft
DAC also approved the proposal for procurement of 33 fighter jets including 21 MIG-29 ((Mikoyan-Gurevich) and 12
Su-30 MKI (Modernizirovannyi, Kommercheskiy, Indiski where ‘Su’ is Sukhoi) aircraft along with upgradation of
existing 59 MIG-29 aircraft.
• The MIG 29 procurement and upgradation from Russia is estimated to cost Rs 7,418 crore.
• The Su-30 MKI will be procured from Hindustan Aeronautics Limited (HAL), India at an estimated cost of Rs
10,730 crore.
About Defence Acquisition Council (DAC):
DAC is chaired by Defence Minister (Currently-Rajnath Singh) which provide approvals to Capital acquisitions in the
Long Term Perspective Plan (LTPP)
About Hindustan Aeronautics Limited (HAL)
Chairman and Managing Director (CMD)– R. Madhavan
Headquarter– Bengaluru, Karnataka

ADB Joins Network For Greening the Financial System as Observer


On July 03, 2020, Asian Development Bank (ADB), a regional development bank, has joined the Paris (France)-
based Network for Greening the Financial System (NGFS) as an observer.
• The NGFS is a global forum that brings together central banks and supervisors committed to understanding
and managing the financial risks and opportunities associated with climate change.
How the ADB will support NGFS?
ADB’s approaches & experience in addressing climate risk management in the financial sector will help in
implementing climate finance targets and its expertise in providing assistance to support the transition of emerging
Asian countries into sustainable economies.
ADB as an observer:
• ADB now joins the ranks of the World Bank (WB), the International Finance Corporation (IFC), the
International Monetary Fund (IMF), and the Organisation for Economic Co-operation and Development
(OECD) as NGFS observers.
• Following the joining of ADB, NGFS now includes 66-member institutions and 13 observers that participate in
3 various work related to reducing climate risks, including supervision, macro-financial issues, and
mainstreaming green finance.
• ADB’s inclusion to the NGFS is aligned with the goals in its corporate strategy, the Strategy 2030, particularly
in tackling climate change, building climate and disaster resilience, and enhancing environmental
sustainability; fostering regional cooperation and integration; and strengthening governance and institutional
capacity.
About the NGFS:
At the Paris “One Planet Summit” on 12 December 2017, 8 central banks and supervisors established a NGFS to share
best practices and contribute to the development of environment and climate risk management in the financial sector
and to mobilize mainstream finance to support the transition toward a sustainable economy.
The NGFS is chaired by Frank Elderson, board member of De Nederlandsche Bank. The Secretariat is provided by
Banque de France.
About Asian Development Bank (ADB):
Headquarters– Mandaluyong, Philippines
President– Masatsugu Asakawa
Membership– 68 countries

RBI conducted OMO of purchase and sale of G-secs for Rs 10,000 cr each; Borrowing facility to banks
extended till Sept 30, 2020
On July 2, 2020, Reserve Bank of India (RBI) conducted the simultaneous purchase of Government Securities (G-Secs),
maturing between 2027 and 2033, and sale of 182 day and 364 day treasury bills (DTBs) under Open Market

Report Errors in the PDF - ebooks@affairscloud.com Copyright 2014-2020 @ AffairsCloud.Com 14


Banking & Economy PDF
Operations (OMO) for Rs 10,000 crore each. This OMO is a part of “Operation Twist” to ease pressure evolving
liquidity and market conditions. The main purpose of the OMO is to bring down the yields at the longer end.
• Purchases: Central Govt purchased G-Secs or GS – 6.79% GS 2027; 7.26% GS 2029; 6.68% GS 2031; and
6.57% GS 2033 – using the multiple price auction method aggregating Rs10,000 crore.
• Sellings: RBI sold four DTBs – two 182 DTBs maturing on October 15 and October 22, 2020; and two 364
DTBs maturing on April 22 and April 29, 2021.
Point to be noted:
-RBI reserves the right to decide on the quantum of purchase/sale of individual securities, accept bids/offers for less
than the aggregate amount, purchase/sell marginally higher/lower than the aggregate amount due to rounding-off or
accept or reject any or all the bid/offers either wholly or partially without assigning any reasons.
Click Here for Official Link
RBI extended borrowing facility provided to the banks till Sept 30, 2020
RBI has decided to extend the enhanced borrowing facility provided to the banks to meet their liquidity shortages till
September 30 from till June 30, 2020.
• Notably, as a temporary measure, RBI had increased the borrowing limit of scheduled banks under the
marginal standing facility (MSF) scheme from 2% to 3% of their Net Demand and Time Liabilities (NDTL) with
effect from March 27, 2020.
Static points about Reserve Bank of India (RBI):
-It was established on the recommendation of the Hilton Young Commission.
-RBI is responsible only for printing the currency notes. Minting of coins is done by the Government of India.
-Dr.Manmohan Singh is the only Prime Minister to have also served as the Governor of RBI.

HDFC Bank to Offer ‘ZipDrive’, an Online Instant Auto Loans to its Customers
On July 2, 2020 HDFC Bank announced that it will offer ‘ZipDrive’, an online instant auto loan to its customers in
1,000 cities. The auto loan disbursal product is only for customers with pre-approved offers. It is a technology-
enabled loan product where credit assessment is done by banking software.
Gist about ZipDrive
Availability– The product will be available to tier 2 and 3 cities across India, such as Bhimavaram in Andhra Pradesh,
Hardoi in Uttar Pradesh, Thalassery in Kerala, and Balasore in Odisha among other places
Loan can be availed within 10 sec– The customers can avail the loan from anywhere, anytime, through Netbanking
or by scanning QR code or through phone banking or through assistance from the bank’s phone banking team within
10 seconds. It is claimed to be the country’s fastest online auto loan disbursement offering.
Loan sanction & No pre-approval document– The loan is sanctioned using the bank’s proprietary algorithm and
analytics. No pre-approval documents are required.
Customer’s preference– The customers can choose the preferred car model, dealer, loan amount within the eligible
limits and tenure online. They can also opt on-road funding up to 100% of the value.
Convenience– It provides a convenient and contactless form for obtaining auto loans from the bank at attractive
prices and terms. The customers need not have to visit any branch
About HDFC Bank:
Headquarters– Mumbai, Maharashtra
Managing Director(MD)– Aditya Puri

Kotak Mahindra Bank & Innoviti Collaborates to Offer EMIs on Kotak Debit Cards Swiped on Innoviti
POS Terminals
On July 2, 2020 Kotak Mahindra Bank (Kotak) and Innoviti Payment Solutions Pvt. Ltd. announced that they have
collaborated to provide Equated Monthly Installments(EMI) billing options on Kotak Debit Cards which are swiped
on Innoviti Point of Sale(POS) terminals.

Report Errors in the PDF - ebooks@affairscloud.com Copyright 2014-2020 @ AffairsCloud.Com 15


Banking & Economy PDF
Key Info
i.Enables customers to pay for their purchases- The facility will enable about 10 million(1 crore)Kotak customers
to pay for their purchases in instalments by using the EMIs on debit cards facility at over 70,000 Innoviti POS
terminals across more than 1,000 cities.
ii.Credit access
• More than half of Innoviti’s POS terminals have been installed in 350 smaller cities and towns in India with a
population of less than 2 million.
• This will help consumers especially in tier 2 & 3 cities to access credit conveniently through their Kotak debit
cards.
iii.Process– The eligible Kotak customer to complete the purchase and to enable instant loan disbursement has to
ask the cashier for the Kotak Debit Card EMI facility, select the preferred EMI tenure and swipe the Kotak debit card
on Innoviti’s POS terminal
Note– The EMIs on Debit Cards facility is simple, easy and convenient to use.
About Kotak Mahindra Bank Limited:
Registered Office– Mumbai, Maharashtra
Managing Director & CEO– Uday Kotak
About Innoviti Payment Solutions Pvt. Ltd:
Registered office- Bangalore, Karnataka
CEO– Rajeev Agrawal

India’s External debt increased by 2.8% to $558.5 bn in FY20: RBI data


In accordance with the Reserve Bank of India (RBI) data, India’s external debt stood at $558.5 billion in March 2020,
an increase of $15.4 billion or 2.8% as compared to the March 2019.
• The largest component of the external debt is Commercial borrowings with a share of 39.4%, followed by
non-resident deposits at 23.4% and short-term trade credit at 18.2%.
• In terms of currency, United States (US) dollar-denominated debt continued to be the largest component of
India’s external debt, with a share of 53.7% at end-March 2020, followed by the Indian rupee (31.9%), yen
(5.6%), SDR (4.5%) and the euro (3.5%).
Key Points regarding external debt:
-There were valuation gains due to the appreciation of the U.S. dollar against the Indian rupee and other major
currencies which stood at $16.6 billion. Excluding the valuation effect, the increase in external debt would have been
$32 billion instead of $15.4 billion at end-March 2020 over end-March 2019.
–Long-term debt: At the end of March, 2020 long-term debt, with original maturity of above one year, was placed at
$451.7 billion, a rise of $17 billion over the level recorded in March 2019.
–Short-term debt on residual maturity basis: It constituted 42.4% of the total external debt and 49.5% of foreign
exchange reserves at the end of March, 2020. The ratio of short-term debt (original maturity) to foreign exchange
reserves declined to 22.4 per cent at end-March 2020 (26.3 per cent at end-March 2019).
–Short-term debt (with original maturity of up to one year): It declined to 19.1% at end-March 2020 from 20% at
end-March 2019
–Outstanding debt: Nonfinancial corporations faced the highest outstanding debt at 42.0 per cent, followed by
deposit-taking corporations (except the central bank) (28.3%), general government (18.1%) and other financial
corporations (7.5%).
–Instruments: Loans were the largest component of external debt, with a share of 34.8%, followed by currency and
deposits (24%), trade credit and advances (18.7%) and debt securities (17.4 %).
–Borrower: Outstanding debt of general government decreased, while that of non-government sector increased at
end-March 2020.
-Debt service (principal repayments plus interest payments) increased marginally to 6.5% of current receipts at end-
March 2020 as compared with 6.4% at end-March 2019.
-For FY20, the current account deficit narrowed to 0.9% of the Gross Domestic Product (GDP) compared to 2.1% in
FY19.

Report Errors in the PDF - ebooks@affairscloud.com Copyright 2014-2020 @ AffairsCloud.Com 16


Banking & Economy PDF
-India’s forex reserves increased by $ 59.5 billion during 2019-20.
Click Here for Official Link
About Reserve Bank of India (RBI):
Headquarters– Mumbai, Maharashtra
Formation– 1 April 1935
Governor– Shaktikanta Das
Deputy Governors– 4 (Bibhu Prasad Kanungo, Mahesh Kumar Jain, Michael Debabrata Patra, one is yet to be
appointed).

India’s GDP Likely to Contract by 6.4% in 2020-21 due to Lockdown: Care Ratings
On July 02, 2020, CARE Ratings Limited ( formerly Credit Analysis and Research Limited) in its report, ‘Revised GDP
growth projections for FY21’ has projected a 6.4 % contraction (or -6.4%) in India’s GDP (Gross Domestic Product)
growth for the current financial year FY 21 (2020-21) due to the continuous lockdown imposed for prevention of
coronavirus (COVID-19) epidemic.
Earlier in May 2020, the agency had predicted a 1.5 to 1.6 % decline in GDP for 2020-21.
2/3 of economic sector will operate at 50-70 % capacity:
As per the report , two-thirds of economic sector will operate at 50–70 % capacity by the end of the third quarter
(Q3) of FY21. The other sectors cannot even reach this level.
Services such as hospitality, tourism, entertainment, travel are estimated to takelong time to start functioning and get
closer to normal levels.
Estimates of good growth in agriculture sector:
Although it has been predicted that there will be positive 2.5% growth in the agricultural sector in FY 21, farmers’
income will not increase due to higher agricultural production. This is because excess supply can lead to a reduction
in prices.
Fears of decline in many sectors:
The report also stated that the secondary sector, including manufacturing, mining and electricity, could face a sharp
contraction of 9.5 % in FY 2021. The service sector, including construction, may shrink by 6.5 % in the same period.
About CARE Rating:
Headquarters– Mumbai, Maharashtra
Managing Director (MD) & Chief Executive Officer (CEO)– Ajay Mahajan

Intel Capital to buy 0.39% stake in Jio Platforms for Rs 1,894.50 crore
US based Electronic chip maker Intel’s investment arm, Intel Capital to buy 0.39% stake in Reliance Industries
Limited(RIL)’s Jio Platforms for Rs 1,894.50 crore. This is the 12th deal of Jio Platforms. With this investment, Jio
Platforms has raised Rs 1,17,588.45 crore from global investors since April 22.
Key Info
• With this investment, RIL holds 74.91% stake in Jio and 25.09% by the investors.
• The investment of Intel Capital will translate into a 0.39% equity stake in Jio Platforms on a fully diluted basis,
with an equity value of Rs 4.91 lakh crore and an enterprise value of Rs 5.16 lakh crore.
Previous Investment in Jio Platforms
Investor In Jio Platforms Date of Investment (2020) Investment Amount (Rs in Crores) % Stake
S.No
1 Facebook 22nd April 43,573.62 9.99%

2 Silver Lake Partners 4th May 5,655.75 1.15%

3 Vista Equity Partners 8th May 11,367.00 2.32%

4 General Atlantic 17th May 6,598.38 1.34%

Report Errors in the PDF - ebooks@affairscloud.com Copyright 2014-2020 @ AffairsCloud.Com 17


Banking & Economy PDF
5 KKR 22nd May 11,367.00 2.32%

6 Mubadala 5th June 9,093.60 1.85%

Silver Lake Partners – Additional


7 Investment 5th June 4,546.80 0.93%

8 Abu Dhabi Investment Authority 7th June 5,683.50 1.16%

9 TPG 13th June 4,546.80 0.93%

10 L Catterton 13th June 1,894.50 0.39%

11 Public Investment Fund(PIF) 18th June 11,367.00 2.32%

About Intel Capital:


Intel Capital has strategic global investments in 5G, cloud computing and artificial intelligence technology areas.
About Intel:
Headquarters- California, U.S.
Chief Executive Officer– Robert (Bob) H. Swan
About RIL:
Headquarters– Mumbai, Maharashtra
Chairman & Managing Director– Mukesh D. Ambani

Odisha Launches 1st of its kind scheme, ‘Balaram’ to provide Rs 1,040 crore credit to 7 lakh Landless
Farmers
On July 3, the Government of Odisha launched 1st of its kind scheme, ‘Balaram’ to provide agricultural credit of Rs
1,040 crore to landless farmers facing hardships during COVID 19.The Scheme is designed in collaboration
with National Bank for Agriculture and Rural Development (NABARD).
Key Points:
• Two state-run organisations – the Institute on Management of Agricultural Extension and the Agricultural
Technology Management Agency (ATMA) – will be the nodal agencies at the state and district levels
respectively for implementation of the scheme.
• In the next two years around seven lakh cultivators will be benefitted by ‘Balaram’.
• The beneficiaries decision was taken during a high-level meeting presided over by chief secretary A K
Tripathy.
• The farmers, who didn’t avail farm credit earlier, will get loans through Joint Liability Groups (JLG) which act
as ‘social collateral’.
• The banking sector will be mobilised through the State Level Bankers Committee (SLBC) for extending credit
support to farmers and sharecroppers who don’t get loans due to lack of resources.
• The ‘Krushak Sathis’ and village agricultural workers will be incentivised through ATMA for repayment of
FLGs by linking them to banks and facilitating loan repayments.
• Around 7000 branches of different banks and primary Agricultural Cooperative societies (PACS) in rural and
semi urban areas.
• Each lender will finance a minimum of 10 JLGs per year.
• A JLG consists of five members and a group will get 1.6 lakhs.
• 1.40 lakh JLGs will cover seven lakh landless cultivators in two years.
• The field level agricultural workers will get a credit of Rs 1,040 crore under ‘Balaram’.
About Odisha:
Capital: Bhubaneswar
Chief Minister: Naveen Patnaik

Report Errors in the PDF - ebooks@affairscloud.com Copyright 2014-2020 @ AffairsCloud.Com 18


Banking & Economy PDF
About NABARD:
Headquarters: Mumbai, Maharashtra
Chairman: G R Chintala

Edelweiss AMC to Launch Second Tranche of ‘BHARAT Bond ETF’ ; Target to Raise Rs 14000 crore
On July 03, 2020, Edelweiss Asset Management Company (AMC), a subsidiary of Edelweiss Capital Limited (ECL)
has decided to bring the second installment of the Bharat Bond ETF (Exchange Traded Fund) with two new series
to raise Rs 14,000 crore in July 2020.
The second batch is due to be launched on 14 July 2020 & the consignment will close on 17 July 2020. It will invest
only in AAA-rated bonds of Public Sector companies.
Consignment breakup:
Through the launch of these two new ETF series, which have maturities of 5 years and 11 years, Edelweiss aims to
raise an initial amount of Rs. 2,000 crores from institutional players with a greenshoe option of Rs. 6,000 crores 5-
year maturity period (April 2025) and initial amount of Rs. 1,000 crores with a green shoe option of Rs. 5,000 crores
in 11 years (April 2031).
The new five-year and 11-year ETFs track have yields of 5.65% and 6.76%, respectively, as on 1 July.
Who invest how much?
– Small retail investors will have to invest at least Rs 1,001 in this fund and in multiples of Re 1 thereafter. Here they
can invest up to 2 lakhs. While non-institutional investors, retirement funds and QIBs, it is to an extent of Rs 2 lakh
and in multiples of Re 1 thereafter.
– At the same time, the minimum amount for anchor investors has been fixed at Rs 25 crore and in multiples of Re 1
thereafter.
Initial Series:
Earlier in December 2019, the successful opening series of ETF with fixed maturity period of 3-years (2023) & 10-
years (2030) was introduced through which, Rs 12,400 crore was raised.
Why was Bond issued?
Generally, governments issue bonds to meet the fiscal deficit & it has put this option in front of investors to raise
money for Public Sector Undertaking (PSUs). Earlier governments used to launch bonds only for big investors but
now small investors can also join.
Bharat Bond ETF:
It is a kind of mutual fund, which will invest only in bonds issued by government companies.The bond will be listed
on the stock exchange, and tradable on exchanges. The funds that will come from investing in this bond will be
invested in companies included in this bond index. In such a situation, it is expected to get better returns. It will track
the Nifty Bharat bond Target Maturity Indices. BHARAT Bond Funds of Funds (FOF) is also being launched for
investors who do not have Demat accounts.
This ETF is an initiative of Department of Investment and Public Asset Management DIPAM),Government of India
(GoI), and then it mandates Edelweiss AMC to design and manage the product.
About Edelweiss Capital Limited:
Headquarters– Mumbai, Maharashtra
Chairman & CEO, Edelweiss Group– Rashesh Shah
Founded– 1995
Edelweiss AMC acts as the Investment Manager to Edelweiss Mutual Fund.
Edelweiss Mutual Fund CEO– Radhika Gupta

IndiaFirst Life Insurance Associates with BOB Financial to Offer COVID-19 cum Hospital Cash Cover
to BOB Credit Cardholders
On July 3, 2020 IndiaFirst Life Insurance Company Limited(IndiaFirst Life) associated with BOB Financial
Services Limited (BOB Financial), a wholly owned subsidiary of Bank of Baroda(BOB) to offer a group insurance
cover. The cover will provide financial support to the BOB credit card customers upon their hospitalisation or on
COVID-19 diagnosis.

Report Errors in the PDF - ebooks@affairscloud.com Copyright 2014-2020 @ AffairsCloud.Com 19


Banking & Economy PDF
Major Highlights
i.Through this association, the customers who choose IndiaFirst Life Insurance Hospi Care plan can avail the benefits
if diagnosed with COVID-19 positive and also for hospitalisation due to any other reason for more than 24 hours
ii.BOB credit card customers can voluntarily join online and select from frequency, cash benefit or hospitalisation
benefit option
Proposed benefits of the cover
The customers can avail any one option of the proposed benefits categories and the premium will be charged
accordingly. The proposed benefit cover are:
i.The COVID-19 Diagnosis Cash Benefit/Hospitalisation Cash Benefit amount is Rs. 5,000 & Rs. 10,000.
ii.Cash benefit if hospitalized for 2 or 4 times a year.
About India First Life Insurance Company Limited:
IndiaFirst Life is promoted by Bank of Baroda and Union Bank of India.
Headquarters– Mumbai, Maharashtra
MD & CEO – R. M. Vishakha
About BOB:
Headquarters– Alkapuri, Baroda(Vadodara)
Managing Director & CEO– Sanjiv Chadha

NLC, Coal India to form 50:50 JV to develop solar and thermal power assets
On July 3, 2020, NLC India Limited signed a pact with Coal India Ltd (CIL) to form a joint venture (JV) to develop
5,000 megawatts (MW) of solar and thermal power assets across the country. There will be equal equity participation
of both companies in the proposed JV i.e. in the ratio of 50:50. Notably, both undertakings are under the
administrative control of the Ministry of Coal.
About NLC:
NLC earlier known as Neyveli Lignite Corporation — has a power generation capacity of 5,192 megawatts besides
lignite mining capacity of 30 million tonnes per annum. Its projected capital expenditure up to the year 2025 is Rs
1.28 lakh crore with a debt-equity ratio of 70:30.
• It is a Navratna Central Public Sector Enterprises (CPSEs).
• Chairman cum Managing Director (CMD)– Rakesh Kumar
• Headquarter– Cuddalore, Tamil Nadu
About Coal India:
Its projected capital expenditure up to the year 2025 is Rs1.28 lakh crore with a debt-equity ratio of 70:30.
• It is a Maharatna CPSE.
• Chairman cum Managing Director (CMD)– Pramod Agrawal
• Headquarter– Kolkata, West Bengal

KKR to Buy 54% Stake in JB Chemicals for about Rs 3,100 crore


US private equity giant KKR has agreed to acquire a 54% stake(41.7 million or 4.17 crore fully paid up equity
shares) in Mumbai-based drug manufacturer JB Chemicals & Pharmaceuticals for approximately Rs 3100 crore.
KKR will make the investment through its subsidiaries, Tau Investments Holdings Pte. Ltd., Tau Holdco Pte. Ltd., and
KKR Asia III Fund Investments Pte.
Key info
• For this purpose KKR had signed an agreement. As part of the deal, KKR will acquire the 4.17 million fully paid
up equity shares for a price of Rs 745 per share from the JB founding JB Mody & family.
Major Highlights of the deal
i.After this transaction, KKR’s subsidiaries will together buy up to 20,093,346(20.93 million or 2.09 crore) shares of
JB Chemicals in the open offer, which is 26% of the voting share capital at a price of Rs 745 per share.
If the open offer is successful it will add another Rs 1,496 crore to the deal, which takes the total value of the deal to
Rs 4,605 crore (approximately).

Report Errors in the PDF - ebooks@affairscloud.com Copyright 2014-2020 @ AffairsCloud.Com 20


Banking & Economy PDF
Note: After the acquisition, the management under JB Mody, JB Chemicals’ chairman and managing
director, Jyotindra B. Mody will continue to run the company.
KKR’s deal in India(2020)
This is the 3rd deal by KKR in India in 2020 after it acquired 5 solar energy assets from Shapoorji Pallonji
Infrastructure Capital (“SP Infra”) SP for Rs 15.54 billion (approximately USD 204 million) on April 27 and 2.3% stake
in Jio Platforms for Rs 11, 367 crore. And this is the 2nd deal in the past 2 months.
About JB Chemicals and Pharmaceuticals:
Headquarters– Mumbai, Maharashtra
About KKR:
Managing Director(MD), Private Equity, Mumbai– Rupen Jhaveri
Co-Chairman and Co-CEO– George Roberts

India Ranked 117th in SDG Index 2020, Sweden Tops: Sustainable Development Report 2020
In accordance with the “Sustainable Development Report 2020- The Sustainable Development Goals and Covid-
19”, comprising of SDG index 2020, there will be severe negative impacts on most of the United Nations (UN)-
mandated Sustainable Development Goals (SDGs) due to COVID-19 pandemic. India, which has ranked at 117th
position with a score of 61.92 is also facing major challenges in 10 of the 17 SDGs including zero hunger, good health,
gender inequality among others. The index has been topped by Sweden.
• The report contains data on changes over time in 17 SDG indicators, the future of the SDGs amidst Covid-19, as
well as calculations for trajectories until 2030.
• The 2020 report has reviewed the performance of 193 UN Member States out of which 166 were ranked under
SDG index 2020. Following table shows top 3 countries under it:
Rank Country Score (out of 100)
117 India 61.92
1 Sweden 84.72
2 Denmark 84.56
3 Finland 83.77
As per 2020 report COVID-19 had negatively affected several SDGs including:
• SDG 1 (no poverty)
• SDG 2 (zero hunger)
• SDG 3 (good health and wellbeing)
• SDG 8 (decent work and economic growth)
• SDG 10 (reduced inequalities)
Meanwhile, the pandemic had brought “immediate relief” in following SDGs:
• SDG 12 (responsible consumption and production)
• SDG 13 (climate action)
• SDG 14 (life below water)
• SDG 15 (life on land)
The report identifies five key measures that global cooperation should include:
1. Disseminate best practices rapidly.
2. Strengthen financing mechanisms for developing countries.
3. Address hunger hotspots.
4. Ensure social protection.
5. Promote new drugs and vaccines.
SDG Annual Report:
Since its launch in 2016, this annual report provides the most up-to-date data to track and review the performance of
all 193 UN member states on the 17 SDGs.

Report Errors in the PDF - ebooks@affairscloud.com Copyright 2014-2020 @ AffairsCloud.Com 21


Banking & Economy PDF
SDG Report 2020:
It was written by lead author Jeffrey Sachs and a team of independent experts working at the Sustainable
Development Solutions Network (SDSN) and Bertelsmann Stiftung (foundation in Germany), and published
by Cambridge University Press. The report outlines the likely short-term impacts of Covid-19 on the Sustainable
Development Goals (SDGs) and describes how the SDGs can frame the recovery. The report also tracks progress by
countries towards the SDGs.
Click Here for Official Link

India Ranked 31st in INFORM Risk Index 2020 for Humanitarian crises & Disasters; Somalia Tops
In accordance with the “INFORM Report 2020:Shared evidence for managing crises and disasters” released by
research centre INFORM, operating under European Commission (EU), India has ranked at 31st place with an inform
risk of 5.4, means risk of humanitarian crises and disasters . The report ranked the countries based on “INFORM Risk
Index” which has been topped by Somalia with an inform risk of 8.9. The report has analyzed 191 countries.
• Notably, this report is produced by the United Nations Office for the Coordination of Humanitarian Affairs
(UN-OCHA) on behalf of all INFORM Partners.
Ranks of Top 3 Countries:
Rank Country Inform Risk
31 India 5.4
1 Somalia 8.9
2 Central African Republic 8.6
3 South Sudan & Yemen 8.1
KeyPoints:
i.North Korea ranks among the most dangerous places to live as its people are exposed to high risks of natural
disaster, poverty and social injustice.
ii.North Korea was rated as the 39th most dangerous place.
About INFORM Risk Index:
It is the first global, objective and transparent tool for understanding the risk of regions based upon crises and
disasters. A higher figure means more exposure to risks. The report considers the following while framing the index
• Natural Factors: Tsunami, Earthquake, Drought, Flood, Epidemic and Tropical Cyclones
• Human Factors: Human Conflicts and conflict risks
• Socio-Economic factors: Inequality, vulnerable people, deprivation and aid dependency
• Institutional: DRR, Governance
• Infrastructure: Communication, Physical infrastructure, Access to health system
About Inform:
INFORM is a multi-stakeholder forum for developing shared, quantitative analysis relevant to humanitarian crises
and disasters. INFORM is a collaboration of the Inter-Agency Standing Committee and the European Commission.
• The Joint Research Center of European Commission is the scientific and technical lead for INFORM.
Click Here for Official Link
About United Nations Office for the Coordination of Humanitarian Affairs (UN-OCHA):
Under-Secretary-General for Humanitarian Affairs and Emergency Relief Coordinator– Mark Lowcock
Headquarter– New York, United States (US) & Geneva, Switzerland

SBM Bank India Partnered with Mastercard to facilitate Real-time domestic & cross-border
payments & remittances through ‘Mastercard Send’
SBM Bank India has partnered with Mastercard to enable its customers to make real-time domestic & cross-border
payments(Business to Consumer- B2C) and remittances quickly and efficiently through ‘Mastercard Send’. This
partnership allows the bank’s customer base to offer quick, convenient and secure payments experiences.
Significance of MasterCard Send– Mastercard Send is a secured and innovative solution. It has played a key role in

Report Errors in the PDF - ebooks@affairscloud.com Copyright 2014-2020 @ AffairsCloud.Com 22


Banking & Economy PDF
diversifying MasterCard payment flows and the payment experience for customers.
Key info about SBM Bank India
i.It is the first bank to receive a banking license from the Reserve Bank of India (RBI) and to establish a universal
banking business in India through the Wholly Owned Subsidiary (WOS) mode. It is promoted by Government of
Mauritius.
ii.It has been leading the collaboration narrative to access the best in class solutions with its switching partner, YAP
as banking evolves to become more personal and platform-agnostic.
Headquarters– Mumbai , Maharashtra
Managing Director(MD) and Chief Executive Officer(CEO)– Sidharth Rath
About Mastercard:
Headquarters– New York, United States
CEO – Ajay Banga ( Michael Miebach will take over as CEO on January 2021)

YES Bank Launches a Digital Solution, ‘Loan in Seconds’ for Instant Disbursement of Retail Loans
On July 6, 2020 YES Bank launched a digital solution, ‘Loan in Seconds’ for instant disbursement of retail loans. It is
available for the bank’s pre-approved liability account holders. The algorithm evaluates the loan application in real-
time and eliminates the need for documentation.
Key Info
i.The solution aims to provide customers, hassle-free loans to help them meet their immediate financial needs.
ii.All the eligible customers under the solution will receive communication from the bank with the link to apply for
the loan.
iii.Those customers have to verify and accept the final offer.
About YES Bank:
Headquarters– Mumbai, Maharashtra
MD and CEO- Prashant Kumar
Tagline– Experience our Expertise

Former RBI Governor Urjit Patel Authored a Book Titled ‘Overdraft: Saving the Indian Saver’
Former Reserve Bank of India (RBI) governor Urjit Patel has authored a book titled ‘Overdraft: Saving the Indian
Saver’, which focuses on the issue of Non-Performing Assets (NPAs) that has affected Indian banking in recent years
and also its causes. The book will be published by HarperCollins India and will be released by the end of July. He was
born in Nairobi, Kenya on 28 October 1963
Key info about the book
i.The book highlights Urjit Patel’s efforts as the RBI governor in dealing with it, where he has explained the problem
and how it exploded and also about how he would have resolved the issue if he had not been prevented.
ii.With his 30 years of experience in macroeconomics, he worked out the ‘9R’ strategy, which will save savings, rescue
banks and protect them from unscrupulous racketeers.
Note– This book will be available from July 31 and the pre-order of the book has been started on Amazon.in.
About Urjit Patel
i.Currently he is appointed as the Chairperson of the National Institute of Public Finance and Policy (NIPFP), a
research body under the control of the ministry of finance for a 4 year term on June 22, 2020,
ii.Prior to this he was the 24th Governor of the RBI. He took charge on September 5, 2016. He resigned in December
2018 before the end of his three-year term in September 2019, due to major differences with the government.
iii.He was appointed as Deputy Governor of RBI in 2013 for a period of three years. He began his career as an
economist with the International Monetary Fund in 1990.
Important post held by him– Urjit Patel served as an advisor (Energy and Infrastructure) with the Boston
Consulting Group. He was principal/deputy in the G-20 and BRICS Finance Ministers’ and Central Bank Governors’
groups from 2013 to 2018.

Report Errors in the PDF - ebooks@affairscloud.com Copyright 2014-2020 @ AffairsCloud.Com 23


Banking & Economy PDF
What is the 9 R strategy?
Refuse, Rethink, Reduce, Reuse, Repair, Refurbish, Remanufacture, Repurpose, Recycle, Recover
About Kenya:
Capital– Nairobi
Currency– Kenyan shilling
President– Uhuru Kenyatta

3rd Anniversary of GST day 2020: July 1


Goods and Service Tax(GST) day is observed annually on July 1. This year marks the 3rd anniversary of the launch
of the day and was marked by the Central Board of Indirect Taxes and Customs(CBIC) and all its field offices across
India. Most of the interactions with stakeholders to mark this day were made in virtual mode due to COVID-19.
Key info
i.The day was 1st celebrated on July 1, 2018 to commemorate the first year of the unprecedented reform of Indian
taxation(GST)
ii.GST plays a vital role to break the barriers to build an Atma Nirbhar Bharat and to take forward the motto of One
Nation One Tax One Market.
Gist about GST
i.GST came into effect on 1st July 2017, through GST Act passed in parliament on 29th March 2017.
ii.GST has replaced various indirect taxes in India like, excise duty, Value-Added Tax (VAT) and services tax among
others.
About CBIC:
It is a part of the Department of Revenue under the Ministry of Finance, Government of India.
Headquarters– New Delhi, India
Chairman– M Ajit Kumar

Central Govt approved funds for Implementation of Jal Jeevan Mission (JJM) in Himachal Pradesh &
Goa
On the lines of the announcement made by the Prime Minister Narendra Modi in 2019 regarding Jal Jeevan Mission
(JJM) which aims to provide “Functional Household Tap Connection” (FHTC) to 18 Crore rural households of the
country by the year 2024, the Centre has earlier approved funds for the implementation of Jal Jeevan Mission for
Odisha, Meghalaya, Assam & Bihar. Click Here to Read. After that Madhya Pradesh, Jharkhand & West Bengal received
fund allocations for the same. Click Here to Read and then to Maharashtra, Karnataka, and Tripura. Click Here to
Read and then Telangana. Click Here to Read. Now extending this scheme further for other states, Centre has now
approved funds to the following states for FY 2020-2021:
HP to provide tap connections to all rural households by 2022; Received Rs. 326.20 Cr funds from Central
Govt for FY21
On 4th July 2020, during a video conference between Union Minister of Jal Shakti, Gajendra Singh Shekhawat and
Chief Minister of Himachal Pradesh (HP) Jairam Thakur, the Union Government allocated Rs. 326.20 Crore for the
effective implementation of Jal Jeevan Mission (JJM) in Himachal Pradesh for FY 2020-21. Including the state’s share
the total funds available for providing Functional Household Tap Connection (FHTC) is Rs. 371 Crore.
• Himachal Pradesh is planning 100% coverage by August, 2022, ahead of national goal by 2024.
• Out of 17.04 lakh rural households in the State of Himachal Pradesh, 9.52 lakh (55.87%) are already provided
with FHTCs. Out of the remaining 7.52 lakh households, HP plans to provide tap connections in 2.44 lakh
households during 2020-21.
• HP has been allocated Rs. 429 Crore under 15th Finance Commission Grants to Panchayati Raj Institutions
(PRIs) and 50% of it is to be used for water supply and sanitation.
Goa plans to provide 100% FHTC in rural areas by 2021; Received Rs. 12.40 Cr funds from Central Govt for
FY21
The government of India has allocated Rs. 12.40 Crore for FY20-21 to Goa on the basis of Annual Action Plan (AAP)
presented by the state to the Ministry of Jal Shakti. The funds have been increased from Rs. 3.08 Crore.

Report Errors in the PDF - ebooks@affairscloud.com Copyright 2014-2020 @ AffairsCloud.Com 24


Banking & Economy PDF
• Goa is planning 100% FHTCs to all rural households by 2021. Out of 2.6 lakh households in the State, 2.29 lakh
households have already been provided with FHTC.
• Goa will be one of first few States to achieve the feat and become 100% ‘Har Ghar Jal’ State.
• In 2020-21, Goa will get Rs.75 Crore as 15th Finance Commission Grants to PRIs.
About JJM:
It aims to enable every rural household to have assured availability of potable water at a service level of 55 litres per
capita per day (lpcd) on a regular and long-term basis. As a mission target, all states are presenting their
implementation plans to the National level Committee headed by Secretary, Department of Drinking Water &
Sanitation, Govt. of India for approval.
• The estimated outlay of the mission is Rs 3.60 Lakh Crore with Central and State share of Rs. 2.08 Lakh Crore
and Rs.1.52 Lakh Crore respectively.

India rises one place to 34th in JLL’s 11th edition of the Global Real Estate Transparency Index 2020;
UK tops
On July 07, 2020, According to the 11th edition of the ‘Global Real Estate Transparency Index 2020’, which covers
99 countries and territories and 163 cities, released by US based property consultant Jones Lang LaSalle’s
(JLL), India has improved by one place to 34th with a total score of 2.69 under ‘semi-transparent’ zone.Regulatory
reforms, better market data and sustainability initiatives led to a one-point improvement in the nation’s ranking.
Global status:
– While, the United Kingdom (UK) has topped the list with a score of 1.31followed by United States ( 1.35 points)
,Australia (1.39 points), France, Canada, New Zealand, the Netherlands, Ireland, Sweden and Germany.
– India’s neighboring country Sri Lanka ranks 65th position while Pakistan is at 73rd position.
Here’s the list of top 3 nations at Global Real Estate Transparency Index 2020:
Country Investment Market Listed Regulatory Transaction
Rank /Territory Score Performance Fundamentals Vehicles & Legal Processes Sustainability
34th India 2.69 3.55 2.35 1.92 2.63 1.92 3.18
United
1 Kingdom 1.31 1.00 1.8 1 1.17 1 2.36
United
2 States 1.35 1.15 1.41 1 1.24 1.28 2.45
3 Australia 1.39 1.18 1.64 1 1.51 1.13 2
India at Real Estate Transparency Index :
• Government’s efforts are paying off: A major reforms by the government and the effect of continuous
improvement in Indian real estate has excited global investors. Institutional investment created a new norm of
$ 5 billion annually over the last three years.
• Major structural reforms such as the Real Estate Regulation and Development Act 2016 (RERA), GST, Benami
Transaction Prohibition (Amendment) Act, 2016, Insolvency and Bankruptcy Code and digitisation of land
records in the realty sector have brought more transparency. This was a largely irregular area a few years ago.
• Central government’s objective of providing ‘Housing for all’ by 2022 is being achieved through regulatory and
fiscal incentives as well as providing tax benefits to sovereign funds for investment in affordable housing.
India’s Rank in Previous editions:
India was ranked 35th in the index during 2018 bi-annual survey, while the country was at 36th position in 2016 and
39th in 2014.
How is the index prepared?
– Transparency is assessed by evaluating the cost and 210 indicators which are divided into 14 topic areas & then
grouped into 6 various factors including performance measurement, market fundamentals, governance of listed
vehicles, regulatory & legal frameworks, transaction process and environmental sustainability.
– Countries ranked 1-10 are categorised ‘highly transparent’ and those between 11-33 are transparent. Nations
ranked between 34 to 59 are considered semi-transparent, between 60-80 are considered to have low transparency
and those below (81-99) are seen as opaque. So, India is in the semi-transparent zone.

Report Errors in the PDF - ebooks@affairscloud.com Copyright 2014-2020 @ AffairsCloud.Com 25


Banking & Economy PDF
About Jones Lang LaSalle’s (JLL):
Headquarters– Chicago, Illinois, United States
President & CEO– Christian Ulbrich
CEO and country head (India)– Ramesh Nair

New School Infrastructure worth Rs 1.94 cr Funded by India inaugurated in Nepal via Video
Conferencing
A new four-storey school building, developed with the Indian assistance of Rs 1.94 crore (Nepalese Rupees (NR)
31.13 million) under the Nepal-Bharat Maitri Development Cooperation programme has been inaugurated in
Shri Saptmai Gurukul Sanskrit Vidyalaya, located in Ilam district of Nepal to boost the learning environment for
students. The project was implemented by the District Coordination Committee (DCC), Ilam.
• The inauguration was made through video conferencing with the participation of Nepalese officials, school
management committee and members of the Indian Embassy.
• Shri Saptmai Gurukul Sanskrit Vidyalaya was set up in 2009 and has the unique qualification of imparting
vedic as well as modern education including Sanskrit.
About India’s financial assistance to Nepal:
In 2015, a massive earthquake has adversely impacted the educational infrastructures, which has also killed 9,000
people in various parts of Nepal. In this regard, India has provided financial assistance to Nepal for the construction
of several such schools with an aim to upgrade the education for the Nepalese students.
• It should be noted that in 2009, the first polytechnic of its kind in Nepal providing technical education in three
engineering streams and several vocational courses namely “Manmohan Memorial Polytechnic (MMP)” was
inaugurated with an estimated cost of Rs.32 crore.
About Nepal:
Capital- Kathmandu
Prime Minister– Khadga Prasad Sharma Oli
President– Bidhya Devi Bhandari

Fino Payments Bank launches ‘Bhavishya’ Savings Account for Minors Aged between 10-18 years
On July 7, 2020 Fino Payments Bank Limited launched ‘Bhavishya‘, a savings account scheme for minors aged
between 10-18 years. The subscription-based savings account aims to inculcate banking and saving habits in
childrens.
About Bhavishya Savings Account
Key Feature
i.The Bhavishya Savings Account can be only opened through the Electronic-Know Your Customer(e-KYC) mode.
ii.The account comes with a free Platinum RuPay Debit card and there is no requirement to maintain any minimum
monthly average balance. The annual Subscription charges for the account is Rs.349 (inclusive ofGoods and Service
Tax-GST)
iii.The minors should have a mobile number that is different from the parent’s number for security reasons.
iv.If the children attain 18 years, the account will be upgraded to a regular savings account, if Re-KYC is submitted
with updated information.
v.The transactions will be allowed only through Aadhar Authentication and Debit Card can be used for financial/non
financial transactions. Only ATM(automated teller machine) transactions are allowed on Debit Cards(as of now 5 free
ATM transactions per month).
vi.The maximum threshold limit of the account is Rs. 50,000.
Major Benefits
Usage of account for various govt schemes- The account can be used to get benefits of various government
schemes for children such as scholarship and DBT (direct benefit transfer) subsidy amount.
Accidental Insurance/Permanent disability cover– The Accidental Insurance/Permanent disability cover of Rs.2
Lakhs is facilitated with Rupay Platinum Debit Card and Rs.1 Lakh cover on Rupay Classic Debit card.

Report Errors in the PDF - ebooks@affairscloud.com Copyright 2014-2020 @ AffairsCloud.Com 26


Banking & Economy PDF
Aadhaar seeding facility– There is an Aadhaar seeding facility to receive scholarship subsidy and other benefits
directly in the account.
Mobile Banking access– There is availability of Mobile Banking access (with view rights) so as to manage the
account, anywhere and anytime.
Free cash deposit, cash withdrawal & SMS alert– There will be free cash deposit limit upto Rs.25,000 per month,
free cash withdrawal at Fino Bank branches and free SMS alerts.
Note:
• The bank will initially introduce the account in Uttar Pradesh(UP), Bihar, and Madhya Pradesh(MP) and then
gradually in other states.
• The bank is likely to open around 1 lakh accounts by the end of FY21
About Fino Payments Bank Limited:
Headquarters– Maharashtra
MD and CEO– Rishi Gupta

Karur Vysya Bank Ties- up with Star Health Insurance For Bancassurance
On July 6, 2020, Karur Vysya Bank (KVB), a Scheduled Commercial Bank in India has joined hands with Star Health
and Allied Insurance Co Ltd for bancassurance (selling of life assurance and other insurance products and services
by bank).
Bancassurance Pact:
• As per the pact, KVB will market wide range health insurance products of the Star Health through its 780
branches.
• In addition, all bank customers will be able to access value-added services like STAR WELLNESS and get
telehealth consultation using TALK TO STAR.
What is bancassurance?
Bancassurance is a partnership between a bank and an insurance company, where the bank sells the tied insurance
company’s insurance products to its clients.
About Karur Vysya Bank:
Headquarters– Karur, Tamil Nadu.
President & Chief Operating Officer(COO)- Natarajan Jagannathan
Tagline– Smart Way to Bank
About Star Health and Allied Insurance:
Headquarters– Chennai, Tamil Nadu
Chairman & CEO – Mr. V. Jagannathan

Deutsche Bank and Google Cloud to Enter into a Multi-Year, Strategic Partnership
On July 7, 2020 Deutsche Bank and Google Cloud have proposed to enter into a strategic, multi-year partnership.
The partnership focuses on redefining how the bank develops and offers its financial services.
Significance of the partnership
The partnership enables co-innovation between the two companies to produce the next generation of technology-
based financial products for clients and will provide Deutsche Bank accessing right to cloud services.
Key info about the partnership
i.Google and Deutsche have signed a Letter of Intent and have also planned to sign a multi-year contract shortly.
ii.Deutsche Bank expects the partnership will generate more than 1 billion euros (USD 1.13 billion) in accumulated
earnings before income and tax (EBIT) over the next ten years.
iii.The deal is part of a 13 billion euro ( USD 14.70 billion)technology investment Deutsche bank has planned up to
2022 so as to restructure itself to recover from years of losses.
Background
In the beginning of 2020, Deutsche bank invited bids from Google, Microsoft , and Amazon to replace the bank’s
outdated and fragmented technology networks.

Report Errors in the PDF - ebooks@affairscloud.com Copyright 2014-2020 @ AffairsCloud.Com 27


Banking & Economy PDF
About Deutsche Bank:
Headquarters– Frankfurt, Germany
Chief Executive Officer(CEO)– Christian Sewing
President– Karl von Rohr
About Google:
Headquarters– California, United States
CEO– Sundar Pichai

Liberty General Insurance launches ‘Liberty Assure’, an industry 1st service


Liberty General Insurance launches 1st of its kind innovative service ‘Liberty Assure’. The Motor(car) insurance
customers who avail this service, are not required to pay the ‘Compulsory Deductible’ cost for each claim. In
addition to this no premium will be charged from the buyers to provide this unique cost-effective feature at the time
of raising a claim/loss.
• The service is a part of its existing Private Car Package Policy under the Regulatory Sandbox approval received
from the Insurance Regulatory and Development Authority of India(Irdai).
Liberty Assure’s additional benefit, ‘Service Warranty’
i.The Liberty Assure will have an additional benefit which includes ‘service warranty’ for customers, covering all
defects which arise from the repair of an accidental vehicle.
ii.The Company’s Preferred Provider Network (PPN) workshop will make such a loss good by providing the required
repair or replacement of the defect at no additional cost.
iii.The service warranty will be available up to 6 months or 10000 kms and will be effective from the date of delivery
of vehicle from Liberty’s PPN Workshop.
iv.If the customer avails the repair option at Liberty’s PPN workshops, they will also get some free value-added
services such as pick-up & drop, exterior car wash among others.
Availability & Eligibility of the Liberty Assure
Eligibility– The customers who intimate their vehicle’s own damage partial loss claim(s) under Private car package
policy during July 03, 2020 to December 31, 2020 will be eligible to avail the benefits, specified under this unique
feature at only the company’s PPN workshops.
Availability- It will be available to the customers across eight locations- Delhi-NCR, Bangalore, Ahmedabad, Jaipur,
Kolkata, Hyderabad, Mumbai, Chandigarh.
What is ‘Compulsory Deductible’?
It is a mandatory component of every auto insurance that is purchased for the private car. It is a fixed amount set by
the car insurance company, which is deducted at the time of claims. It is determined based on the engine capacity of
an individual’s private car.
About Liberty General Insurance:
Headquarters– Mumbai, Maharashtra
CEO and Whole Time Director– Roopam Asthana

Ola Enters into a Strategic Partnership with PhonePe to Enhance its Digital Payments Experience for
its Customers
On July 7, 2020 India based ridesharing company, Ola announced that it has entered into a strategic partnership
with PhonePe to boost its digital payments experience for its customers.
Significance of the partnership
i.This partnership will help Ola users to link their existing PhonePe account with their Ola account to make payments
for future rides.
ii.This partnership will also enable PhonePe to provide their services to millions of customers on the Ola platform.
Key Info
i.The customers of Ola can use all the payment options of PhonePe, including the PhonePe wallet, to make payments
in addition to Unified Payment Interface (UPI).
Availability of the option– Currently, the option to link PhonePe account with the Ola app is available to Android

Report Errors in the PDF - ebooks@affairscloud.com Copyright 2014-2020 @ AffairsCloud.Com 28


Banking & Economy PDF
users in India and will be soon available to iOS users.
Note– Ola has announced that customers can avail a cashback upto Rs 200 for the first two rides when paid using
PhonePe. Already, Google Pay and Paytm were integrated with Ola for digital payment.
Recent launch of Ola
Recently, Ola has launched ‘Ride Safe India’ to encourage customers to choose digital channels amid Covid-19
About Ola:
Headquarters– Bengaluru, Karnataka
Co-Founder &Chief Executive Officer(CEO)– Bhavish Aggarwal
About PhonePe:
Headquarters– Bengaluru, Karnataka
CEO– Sameer Nigam

Germany to Become First Major Economy to Phase out of Coal, Nuclear Power
German became the first industrialized country to phase out of both coal and nuclear energy, On 3rd July 2020, both
the houses of the German parliament approved the bill for shutting down the coal fired power plant by 2038 and to
provide around 45 billion USD (40 billion Euro) for the affected regions.
Key Points:
i.This phase out is a part of Germany’s “energy transition” to reduce the use of fossil fuels and to support the use of
renewable resources.
ii.German closed its last black coal mine in 2018 and committed to close its nuclear power plants by 2022.
iii.GreenPeace and other environmental groups protest against this plan advocating that this won’t speed up the
process to achieve the targets of the Paris climate accord by reducing the greenhouse gas emissions by 2050.
iv.Germany utilises a great amount of coal which could affect the upcoming generation with 18years worth of carbon
dioxide.
About Germany:
President– Frank-Walter Steinmeier
Minister of Environment– Svenja Schulze
Capital– Berlin
Currency– Euro

AIIB Releases USD 50 mn as 1st Tranche to L&T Infrastructure Finance for Renewable Energy
On 7, 2020 Asian Infrastructure Investment Bank(AIIB) has released USD 50 million in the first tranche of assistance
to L&T Infrastructure Finance Ltd(LTIF), to fund renewable Energy Projects in the country. The released amount is
part of a sanctioned USD 100 million External Commercial Borrowing(ECB) loan.
• This is the 1st loan of AIIB to a non-banking financial company (NBFC) in India.
Gist about the project
i.The objective of the project is to enhance the supply of renewable energy in India by mobilizing private capital.
ii.On completion, the project will increase private capital investment in renewable energy projects in India, which will
increase the provision of affordable, clean energy and reduce the level of greenhouse gas emissions.
Major Highlights
i.The loan proceeds will be used to finance large and mid-scale wind and solar power infrastructure projects in India.
ii.LTIF will establish an Environment & Social Monitoring System that will screen, categorise, appraise, contract and
monitor sub-projects supported by AIIB in accordance with the AIIB Environmental and Social Standards(ESS).
iii.With closure of this financing deal with AIIB, LTIF has further diversified its long term funding sources.
Benefits of the partnership
• This collaboration will enable the company to enhance its environmental and social capability, which help it to
tap the international market for green finance, in the future.
• The partnership will also aid LTIF to tap into the international market of green finance.

Report Errors in the PDF - ebooks@affairscloud.com Copyright 2014-2020 @ AffairsCloud.Com 29


Banking & Economy PDF
Note
i.L&T Infra Debt Fund (LTIDF), a unit of LTFH for refinancing operational infrastructure projects, intends to raise Rs
1,000 crore by way of Principal Protected Market-Linked Debentures. The Credit Analysis & Research
Limited(CARE), a rating agency has assigned “AAA” to this instrument.
ii.L&T Finance had previously mentioned that it had plans to raise up to Rs 2,000 crore by issuing preference shares
through public offer or private placement.
About L&T Finance Holdings Ltd:
LTIF is a wholly owned subsidiary of L&T Finance Holdings Ltd, a part of Larsen & Toubro Limited
Headquarters– Mumbai , Maharashtra
MD & CEO and Director on the Board of LTIF– Dinanath Dubhashi
About AIIB:
Headquarters– Beijing, China
President and Chairman of Board of Directors– Jin Liqun

Karur Vysya Bank Enters into Bancassurance Corporate Agency Partnership with Bajaj Allianz Life
Insurance
On July 8, 2020, Karur Vysya Bank (KVB), a Scheduled Commercial Bank in India has joined hands with Bajaj
Allianz Life Insurance for bancassurance (selling of life assurance and other insurance products and services by
bank).
Bancassurance Pact:
As per the pact, KVB will market all retail and group products of Bajaj Allianz including savings, retirement,
investment, protection and critical illness life insurance products through its 780 branches.
Key Points:
i.Through this partnership, Bajaj Allianz will strengthen its penetration in southern India, leveraging tech-enabled
servicing solutions & comprehensive life insurance products to KVB customers.
ii.KVB will also benefit from the advantage to offer their customers a value-packed life insurance solutions for their
financial requirements under a single roof.
iii.Earlier on July 6, 2020, KVB has joined hands with Star Health and Allied Insurance Co Ltd for bancassurance.
What is bancassurance?
Bancassurance is a partnership between a bank and an insurance company, where the bank sells the tied insurance
company’s insurance products to its clients.
About Karur Vysya Bank:
Headquarters– Karur, Tamil Nadu.
President & Chief Operating Officer(COO)- Natarajan Jagannathan
Tagline– Smart Way to Bank
About Bajaj Allianz Life Insurance:
Headquarters– Pune, Maharashtra
MD & CEO- Tarun Chugh

NSDC Collaborates with Microsoft India to Provide Digital Skills to Indian Youth
On July 08, 2020, The National Skill Development Corporation (NSDC), a not-for-profit public limited company has
joined hands with tech giant company Microsoft India Private Limited, a subsidiary of American software company
Microsoft Corporation, to enhance the digital skills of the country’s youth. Under this, more than 1 lakh youth will be
digitally enabled in the country in the next one year.
About the partnership:
• As part of the tie-up, Microsoft’s learning resource center, Microsoft Learn, will be integrated with NSDC’s
eSkill India portal to provide access to personalised learning paths and critical technology from entry level to
advanced level like AI (Artificial Intelligence), machine learning (ML) and cloud computing so that the youth
can be exposed to the nuances of today’s digital economy.

Report Errors in the PDF - ebooks@affairscloud.com Copyright 2014-2020 @ AffairsCloud.Com 30


Banking & Economy PDF
• Through the portal, e-skilling events, webinars and virtual sessions will be conducted across the country to
support students, training partners and higher-education trainees enrolled under NSDC’s skill training
programs.
Key Points:
i.The partnership with NSDC is an extension of Microsoft’s Global Skilling Initiative. Under this, the company is
teaching new digital skills to 25 million people all over the world, which is needed in the coronavirus (Covid-19)
economy.
ii.The integration between the two firms will give the youth a chance to redefine their skills for the digital economy.
About Microsoft India Private Limited:
Headquarters Hyderabad, Telangana
President– Anant Maheshwari
About National Skill Development Corporation (NSDC):
Headquarters– New Delhi
CEO and MD- Dr.Manish Kumar

Rajnath Singh e-Inaugurated Rs 43 cr Six Strategic Bridges in J&K; Built by Border Road
Organisation
On July 9, 2020, Defence Minister Rajnath Singh inaugurated six strategic bridges in the border areas close to the
International Border (IB) and Line of Control (LoC) in Jammu and Kashmir (J&K) through video conferencing, from
New Delhi. These were built in a record time by the Project Sampark of Border Road Organisation (BRO). The total
outlay of the following bridges is Rs 43 crore:
• Tarnah-I Bridge (160m)
• Tarnah-II Bridge (300m)
• Palwan Bridge (91m)
• Ghodawala Bridge (151m)
• Pahadiwala Bridge (61m)
• Panyali Bridge (31m)
Of the above, the two bridges are located on the Tarnah Nallah in Kathua District while the other four are on
Akhnoor-Pallanwala road in Akhnoor/Jammu district .
Key Points:
-These bridges would facilitate the movement of the Armed Forces in strategically important sectors and also
contribute towards the overall economic growth of remote border areas.
-They were constructed through the segmental technique of construction and it would be an alternate route to NH 44
from Pathankot, Punjab to Jammu benefiting four lakh people in 217 villages.
-The annual budget of the BRO raised to Rs 8,050 crore in 2019-2020 and for 2020-2021 it is likely to be Rs 11,800
crore in order to improve infrastructure in border areas.
Participants during e-inauguration: The six bridges were inaugurated in the presence of Minister of State (MoS)
(Independent Charge) and MoS Prime Minister’s Office, Ministry of Personnel, Public Grievances & Pensions,
Department of Atomic Energy and Department of Space Dr Jitendra Singh, Member of Parliament, Jammu Shri Jugal
Kishore Sharma, Chief of Army Staff Gen Manoj Mukund Naravane, Defence Secretary Dr Ajay Kumar, and along with
senior Army and Civil Administration officials.
About Border Road Organisation (BRO):
Director General– Lt Gen Harpal Singh
Headquarters– New Delhi

CBDT signed an MoU with SEBI for the Data Exchange


The Central Board of Direct Taxes (CBDT) and Securities and Exchange Board of India (SEBI) signed a Memorandum
of Understanding (MOU) for the data exchange between the two organisations via video conference.
Highlights of MoU:
• It facilitates sharing of data and information automatically between the two organisations on a regular basis.

Report Errors in the PDF - ebooks@affairscloud.com Copyright 2014-2020 @ AffairsCloud.Com 31


Banking & Economy PDF
• On request and suo moto basis, both organisations exchange information available in the databases for doing
their functions under various laws.
• The MoU comes into force from the date of signing.
• CBDT and SEBI are already collaborating through various existing mechanisms.
• A Data Exchange Steering Group is constituted and will meet periodically to review the status of data exchange
and improve effectiveness of the data sharing mechanism.
Mou Signed By:
Smt. Anu J. Singh, Pr. Directorate General of Income Tax (DGIT) (Systems), CBDT and Smt. Madhabi Puri Buch, Whole
time member, SEBI in the presence of senior officers from both organizations via video conference.
Note:
The MoU marks a new era of cooperation and synergy between SEBI and CBDT.
About Securities and Exchange Board of India:
Headquarters: Mumbai
Chairman: Shri Ajay Tyagi
About Central Board of Direct Taxes:
Headquarters: New Delhi
Chairperson: Pramod Chandra Mody

Tamil Nadu Tops Market Borrowings Among States in India With a Collection of ₹30,500 crore for
2020-21 : RBI
On July 09, 2020, According to data from the Reserve Bank of India (RBI), Tamil Nadu (TN) has topped market
borrowings among States in India with a collection of Rs.30,500 crore in the fiscal 2020-21(FY 21).
• TN, which has accounted for 17% of the borrowings done through the issue of bonds (known as state
development loans), has been followed by Maharashtra ₹25,500 crore (14%), Andhra Pradesh ₹17,000 crore
(9%), Rajasthan ₹17,000 crore (9%).
TN’s borrowing:
Earlier on July 7, 2020, Tamil Nadu raised ₹1,250 crore at the low interest rate of 6.63% for the 35-year bonds and
₹1,250 crore at the rate of 4.54% for the three-year bonds in the auction conducted by RBI.
• In addition, it has also raised an additional amount of ₹500 crore in place of originally planned ₹2,000 crore
through the issue of 35 years and 3 years bond with ₹1,000 each .It also have a ‘green shoe’ option to raise a
further ₹250 crore in each of these securities.
What is the need?
The state Government has raised the amount through the issue of long tenure bonds rather than short-terms funding
tools like Ways and Means Advances (WMA) to meet the increased expenditure and falling revenue situation amid the
coronavirus (COVID-19) pandemic.
The Finance Secretary projected that the TN will witness a shortfall in revenue of about ₹12,000-₹13,000 crore a
month.
The GST (Goods and Services Tax) collection of TN has sharply reduced by 15% in June 2020, higher than the 3% fall
in national GST collections due to the continuous lockdown.
About Reserve Bank of India (RBI):
Headquarters– Mumbai, Maharashtra
Governor– Shaktikanta Das
Deputy Governors– 4 (Bibhu Prasad Kanungo, Mahesh Kumar Jain, Michael Debabrata Patra, one is yet to be
appointed).

Global economy to contract by 5.2 pc in 2020: D&B’s Country Risk and Global Outlook Report
In accordance with the Dun and Bradstreet’s (D&B) Country Risk and Global Outlook Report, which has covered
132 countries, the global economy is likely to contract by 5.2% in 2020. This is the biggest decline since the Second
World War and a far stronger contraction than the 1.7% recorded in 2009 during the global financial crisis. As per
report, the global economy will not reach pre-pandemic levels of activity again before 2022.

Report Errors in the PDF - ebooks@affairscloud.com Copyright 2014-2020 @ AffairsCloud.Com 32


Banking & Economy PDF
Indian Front:
-India’s economy is expected to contract this fiscal year after four decades of positive growth.
-In March, 2020 D&B downgraded India’s rating to DB5c from DB4d, stating the magnitude of downgrade and the risk
level highest since 1994.
-DB5 means high risk and denotes that considerable uncertainty is associated with expected returns. Businesses are
advised to limit their exposure and/or select high risk transactions only.
About Dun & Bradstreet (D&B):
Dun & Bradstreet’s Country Risk Indicator provides a comparative, cross-border assessment of the risk of doing
business in a country. The risk indicator is divided into seven bands, ranging from DB1 to DB7, with DB1 being lowest
risk.Each band is subdivided into quartiles (a-d), with ”a” representing slightly less risk than ”b” (and so on).
Headquarter– New Jersey, United States (US)
Location in India– Mumbai, Maharashtra
Chief Operating Officer (COO), International & Chairman of the Board and Interim Country Leader– Julian
Prower
Managing Director (India)– Manish Sinha
Chief Economist (India)- Arun Singh

ADB Renews MoU with IEA to Scale up Collaboration on Energy Sector Sustainability and Resilience
in Asia & Pacific Region
On July 10, 2020, Asian Development Bank (ADB), a regional development bank has renewed a memorandum of
understanding (MOU) for 3 years with the Paris (France) based International Energy Agency (IEA) during IEA’s
Clean Energy Transitions Summit 2020 with the aim to further increase the collaboration on sustainability with a
focus on energy sector resilience in Asia and the Pacific as this sector is a key promoter of growth and human
development, especially during recovery from the impacts of the coronavirus (COVID-19) pandemic.
Background:
ADB, which earlier inked a 3-year MOU with IEA in March 2017 to share knowledge and best practice in energy sector
data and analysis, on-the-ground engagement, capacity building, technology, and innovation, among other areas in
ADB’s developing member countries.
Key Points:
i.The MoU will pave the way to overcome unique knowledge and bridge the experience gap that prevents the
development of sustainable energy systems in ADB’s developing member countries and enhance IEA’s data collection
and capacity building efforts in Asia and the Pacific.
ii.From the year 2008 to 2019, ADB has offered over $23 billion in clean energy, including both sovereign and
nonsovereign initiatives. In addition to that in 2019, it provides climate financing amounting $6.56 billion that results
in meeting its target of doubling its annual climate investments from 2014 one year ahead of schedule.
iii.By the year 2030, ADB aims to provide 80 billion dollars from its own resources and for at least 75 % of its country
operations to feature climate adaptation and mitigation initiatives.
About Asian Development Bank (ADB):
Headquarters– Mandaluyong, Philippines
President– Masatsugu Asakawa
Membership– 68 countries
About International Energy Agency (IEA):
Headquarters– Paris, France
Executive Director– Fatih Birol

ADB & EAA Foundation Collaborated in $100 million Project to Provide Quality Primary Education
for out of school children
On July 9, 2020, The Asian Development Bank (ADB) , a regional development bank, will help India and 9 other
Asian nations to ensure quality education for the out-of-school and at-risk children in the primary level.
In its preliminary effort, ADB has entered into a 5-year agreement with Qatar-based Education Above All (EAA)

Report Errors in the PDF - ebooks@affairscloud.com Copyright 2014-2020 @ AffairsCloud.Com 33


Banking & Economy PDF
Foundation to work jointly for project worth of $100 million to put at least 3.2 lakh out-of-school children,
including refugees and internally displaced children into quality primary education.
Countries picked up for the project:
Initially,10 nations have been selected for projects namely- Afghanistan, Bangladesh, India, Indonesia, Myanmar,
Nepal, Pakistan, Papua New Guinea, Philippines, and Sri Lanka.
Pact signed by:
The pact has been signed at an event organised by EAA, ADB, the State of Qatar, Qatar Fund for Development and the
Islamic Solidarity Fund for Development, by Director General of ADB’s Strategy, Policy and Partnerships Department
Tomoyuki Kimura and Foundation Chief Executive Officer Fahad al Sulaiti at a special side event to the United Nations
High Level Political Forum 2020 that connects global experts to discuss the impact of coronavirus (COVID-19) on
education and its mitigation measures.
About Asian Development Bank (ADB):
Headquarters– Mandaluyong, Philippines
President– Masatsugu Asakawa
Membership– 68 countries
About Education Above All (EAA) Foundation:
Location– Doha, Qatar
Chairperson– Sheikha Moza bint Nasser

IRDAI Revised Arogya Sanjeevani Policy limit;Minimum Insurance limit Rs.50000 & Maximum
Insurance limit beyond 5 Lakh
The Insurance Regulatory and Development Authority of India (IRDAI) has allowed all general and health insurance
companies to increase the sum insured on the Standard Individual Health Insurance Product namely “Arogya
Sanjeevani” beyond Rs 5 lakh.
• This decision has been taken in view of the increasing cost of healthcare in the country amid the covid-19
pandemic.
• Arogya Sanjeevani, is a product for which all insurers are mandated to offer had a cap of Rs 5 lakh on the sum
insured and the minimum cover for it was of Rs 1 lakh. The regulator has now allows insurers to offer sum
insured as low as Rs 50,000 with no upper limit. The sum insured has to be in multiples of Rs 50,000.
About Arogya Sanjeevani:
It is one of the most affordable health insurance products available today, but the policy comes with a 5% co-pay and
2% sub-limit on room rent. A 5% co-payment clause means that the person who insured will pay 5% of the claim
amount and the insurer will pay the rest. The policy covers room, boarding and nursing expenses but only up to 2%
of the sum insured, subject to a maximum of Rs 5,000 a day.
About IRDAI:
Headquarters– Hyderabad, Telangana
Chairman– Dr. Subhash Chandra Khuntia

Mastercard to Invest Rs 250 cr to Support SMEs in India Amid COVID-19 Crisis


On July 9, 2020 Mastercard has announced that it will invest Rs 250 crore(USD 33 million) to support Indian Small
and Medium Enterprises(SMEs) to enable the SMEs to restart itself and help them in business recovery. SMEs have
been hit hardest during the lockdown caused by COVID-19 due to their dependence on the cash-economy and
disruption of the supply chain.
About the investment
As part of this investment, Mastercard will provide various initiatives for the following reasons:
• To help small businesses to pave way for growing digital payments awareness and provide low-cost
acceptance solutions through online and offline which are simple, safe and secure.
• To drive inclusive growth by enabling small merchants and kirana stores with access to credit, knowledge and
tools which will drive operational efficiencies.
• To empower women entrepreneurs by enhancing their business intelligence.

Report Errors in the PDF - ebooks@affairscloud.com Copyright 2014-2020 @ AffairsCloud.Com 34


Banking & Economy PDF
Key Info
i.These initiatives are an extension of the global commitment of Mastercard to inject finance, that pledges to bring a
total of one billion people and 50 million micro and small businesses into the digital economy by 2025.
ii.Mastercard has also pledged USD 250 million to support small businesses globally in April, 2020.
iii.These new initiatives are built on the partnership of Mastercard with the Confederation of All India Traders (CAIT)
to promote digital payment awareness and support merchant on-boarding for digital commerce.
Benefits of Initiatives
i.Initiatives will enable India’s small businesses and entrepreneur to gain increased access to credit.
ii.They will build operational efficiencies through improved customer and also will help in inventory management,
employee engagement and regulatory compliance.
Significance of the SMEs– The small businesses contribute nearly 35% of the GDP, which makes it the foundation
of India’s economy
Note– More than 90% of SMEs were closed for business during the extended nationwide lockdown
Mastercard’s partnership to help offline merchants and Kirana stores
Mastercard, Axis Bank Ltd & Worldline India partnered to launch a digital point-of-sale(POS) solution ‘Soft POS’, the
1st financial payments service in India. It will help offline merchants and Kirana stores accept payments using their
smartphones. It will transform everyday smartphones into merchant POS terminals.
About MasterCard:
Headquarters– New York, United States
Chief Executive Officer– Ajay Banga ( Michael Miebach will take over as CEO on January 2021)

SEBI Restructures Advisory Committee on Mutual Funds Under the Chairmanship of Usha Thorat
On July 10, 2020, Securities and Exchange Board of India (SEBI), the regulator of the securities and commodity
market in India, has reconstituted the committee advising it on matters related to regulation and development of
the mutual fund industry by including five new members.
• The new 20 member committee, will be chaired by Usha Thorat, former deputy governor of Reserve Bank of
India (RBI).
i.The previous 15 members panel that formed in 2013 was headed by Janki Ballabh, former Chairman of SBI.
Terms of Reference (ToR) of the committee are as follows,
• This committee will advise SEBI on issues related to regulation and development of the mutual fund industry
for ensuring investor Protection.
• It will also suggest SEBI on disclosure requirements &advise measures required to be taken for change in the
legal framework to introduce simplification and transparency in the mutual fund regulations.
Composition of the panel:
Mr. Ananth Narayan Associate Professor, Finance, SPJIMR; Mr. Ashish Chauhan MD & CEO, BSE ; Mr. Brij Gopal Daga
Independent Trustee Director, Motilal Oswal Mutual Fund; Shri. Deepak Ranjan Deputy Director, Ministry of Finance ;
Mr. Dhirendra Kumar CEO, Value Research India Pvt. Ltd ; Mr. Kailash Kulkarni CEO, L&T Investment Management
Ltd ; Mr. K. N. Vaidyanathan Executive Vice President, Mahindra & Mahindra Ltd ; Ms. Monika Halan Consulting
Editor, Mint Money ; Dr. M. S. Kamath General Secretary, Consumer Guidance Society of India ; Mr. Nilesh Shah
Managing Director, Kotak Mahindra Asset Management Company Ltd; Mr. Nilesh Vikamsey Partner, Khimji Kunverji
& Co; Mr. Nitin Vyakaranam CEO, ArthaYantra are some of the members of the panel which also include,
– Mr. N. S. Venkatesh Chief Executive, Association of Mutual Funds in India
– Mr. Rajnish Narula CEO, Canara Robeco Asset Management Company Ltd
– Mr. Sandeep Parekh Managing Partner, Finsec Law Advisors
– Mr. Sanjay Sapre President, Franklin Templeton Asset Management (India) Pvt. Ltd.
– Mr. Saurabh Mukherjea CEO, Marcellus Investment Managers Ltd
– Mr. Sunil Gulati Independent Trustee, SBI Mutual Fund
– Mr. S V Muralidhar Rao Executive Director, SEBI

Report Errors in the PDF - ebooks@affairscloud.com Copyright 2014-2020 @ AffairsCloud.Com 35


Banking & Economy PDF
About Securities and Exchange Board of India (SEBI):
Headquarters– Mumbai, Maharashtra
Chairman– Ajay Tyagi

Maharashtra Govt & SBI to Set up Stress Fund to Revive Slum Rehabilitation Authority projects
On July 9, 2020 Maharashtra Housing Minister Jitendra Awhad announced that the state govt and State Bank of
India(SBI) will set up a stress fund to revive Slum Rehabilitation Authority(SRA) projects(about 370 projects) in
the Mumbai Metropolitan Region (MMR) and other urban areas of the state. For this purpose, the state govt
will inject Rs 700 crore to Rs 1,000 crore in this fund as a financial support. SRA will act as the nodal agency under
the Housing Ministry to implement the slum redevelopment schemes.
• The state govt is in discussion with State Bank of India(SBI) and other banks to create the fund under
Shivshahi Punarvasan Prakalp (SPP).
• The funds will be raised by state-run Shivshahi Punarvasan Prakalp Limited (SPPL), through banks, but the
fund will be managed and distributed by the banks.
Other govt announcements
Slum Redevelopment Policy– The State govt has also announced a slum redevelopment policy to ensure quick
access to housing for slum dwellers in MMR.
Scrutinizing Levels– Before, the files were approved after scrutinizing at 6 levels, now it will be at 3 levels. This will
reduce the timeline by 50% at all stages.
Premiums- Various premiums in regard to such projects have been eased by reduction and deferred payments.
Maintenance deposit and rent payable
i.The maintenance deposit of Rs. 40,000 per tenant will now be payable after obtaining the occupation certificate for
the rehabilitation building.
ii.SRA has capped the rent structure during the rehabilitation phase to Rs 12,000 and Rs 10,000 per month in the city
and suburb zones respectively. In case of extended suburbs which come under the MMR region, has capped rental
structure to Rs 8000 per month.
Eligibility of slum dwellers
The eligibility of slum dwellers to avail the benefits of slum rehabilitation projects will now be fully carried out by the
SRA. Earlier, this was done by Municipal Corporation of Greater Mumbai (MCGM), SRA and collector office.
Intimation of Approval and Letter of Intent
i.According to the current policy, applications for intimation of approval (IOA) were accepted after issuance of letter
of intent, but now it will be approved simultaneously and completion certificate (CC) issued within seven days of the
application.
ii.The intimation of approval by SRA will now be granted with the issuance of a Letter of Intent(LOI) within a week of
receiving an application.
iii.Before, Annexure 3 was required before LOI, which is now needed before CC.
Responsibility to vacate slums
SRA will take the responsibility to vacate the slums where the builders have received 70% consent of the slum
dwellers
Other info: All the SRA will have with health centres, authorized committees to relocate religious structures within
the project among others.
About Maharashtra:
Capital– Mumbai
Governor– Bhagat Singh Koshyari
Chief minister– Uddhav Thackeray

Manipur CM N. Biren Singh Virtually Inaugurates 5 Water Supply projects, worth Rs 48.39 crore and
Rapid Antigen Detection Testing Centre
On July 8, 2020 Manipur Chief Minister(CM) N. Biren Singh inaugurates 5 Water Supply projects worth Rs 48.39
crore, which have a total capacity of more than 26.15 millions of liters per day(MLD) through video conferencing

Report Errors in the PDF - ebooks@affairscloud.com Copyright 2014-2020 @ AffairsCloud.Com 36


Banking & Economy PDF
from the Chief Minister’s Secretariat, Imphal. With addition to the projects one office building of Weight and
Measures Department and Rapid Antigen Detection Testing Centre at Jiribam were also inaugurated.
• The New Development Bank has sanctioned Rs 3,000 crore to provide drinking water to every household of
the state and work orders for the projects have been issued.
5 Water Supply Projects
i.The Integrated Water Supply Project for Imphal Planning Area Phase-1(15.89 MLD) at Koirengei, is
constructed with a project cost of Rs 8.58 crores. The project was started in March 2018 and completed in March
2020.
ii.The Integrated Water Supply Project for Imphal Planning Area Phase-1 i/c Pumping
Station at Sangakpham(0.80 ML) is constructed with a project cost of Rs 4.44 crores. The project was started in
March 2018 and completed in March 2019.
iii.The Integrated Water Supply Project for Imphal Planning Area Phase-1(9.08 MLD) at Canchipur was
constructed at the project cost of Rs 6.10 crores. The project was started in March 2018 and completed in March
2020.
iv.The Water Supply Scheme at Heingang Awang Leikai(0.384 MLD) was constructed at the project cost of Rs 5.88
crores. It was started in August 2018 and completed in March 2020.
v.The Water Supply distribution network for Churachandpur was constructed at the project cost of Rs 2339.36
Lakhs or Rs 23.3936 crores. It is to promote tourism under the North East Special Infrastructure Development
Scheme (NESIDS). It was started in February 2019 and completed in June 2020.
Rapid Antigen Detection Testing Centre
i.Rapid Antigen Detection Testing Centre will help in early detection of Coronavirus infection and quick diagnosis
ii.The testing facilities will also be made available to other districts of the state shortly.
iii.The state govt plans to open a COVID Care Centre in Jiribam district.
Office building of Weight and Measures Department
The office building of the Controller of Legal Metrology and Research and Development Centre, Lamphel was
constructed at a project cost of Rs 2 crores during 2018-2019.
Key People– Minister for CAF&PD Karam Shyam, Minister of Agriculture V. Hangkhanlian, Deputy Chairman of State
Planning Board S. Rajen Singh among others participated in the inauguration.
About Manipur:
Capital– Imphal
Governor– Najma Heptulla

India Moves up a Rank, Becomes the 2nd Largest Foreign Investor in UK for the year 2019-20; US
tops
According to the United Kingdom’s Department for International Trade (DIT) inward investment statistics for 2019-
2020, India has become the second largest Foreign Direct Investor in the United Kingdom (UK) after the United States
in 2019, by investing in 120 projects and creating 5,429 new jobs in the UK.In the previous year(2018), India was
ranked as the 3rd largest Foreign Direct Investor of UK.
Top Foreign Direct Investors in UK 2020:
• The United States of America remains as the top source of Foreign Direct Investment (FDI) of the UK,
delivering 462 projects and 20,131 jobs.
• The List is followed by India, Germany, France, China and Hong Kong.
Improvisations:
• India increased with a 4% FDI increase for the UK on 2018-19 with 1,852 new inward investment projects in
the 2019/2020 financial year.
• India’s projects in 2018-2019 were at 106, resulting in a job creation figure of 4,858.
• From the FDI report, the number of jobs safeguarded by FDI has increased by 29 per cent this year.
• The DIT has worked with investors to safeguard 26 percent more jobs compared to 2018-2019.

Report Errors in the PDF - ebooks@affairscloud.com Copyright 2014-2020 @ AffairsCloud.Com 37


Banking & Economy PDF
About United Kingdom:
Prime Minister- Boris Johnson
Capital- London

IHD India Receives ‘Development Marketplace Award 2020’ from World Bank and SVRI
Institute for Human Development (IHD), an Indian non-profit institution conferred with ‘Development
Marketplace Award 2020:Innovations to Address Gender-Based Violence’ from World Bank Group and Sexual
Violence Research Initiative (SVRI) for the project alcohol, intimate partner violence and household economy: an
assessment of liquor ban in Bihar. Apart from India, other 8 non profit organizations from various countries have
received this award.
Other Development Marketplace Award 2020:
• Jordan Forum for Business and Professional Women; and Hakoura for Educational Programs for Testing
reintegration model “trust” for women and girls’ survivors of gender-based violence with their families and
communities in Jordan (Jordan).
• Viktimolosko drustvo Srbije/Victimology Society of Serbia for Sexual violence at the universities in Serbia: Raising
awareness and developing innovative mechanisms of victim support (Serbia).
• Monash University for Impact on men’s attitudes and behaviours of a multi-component, gender-informed, father-
inclusive intervention for early child development in Vietnam (Vietnam).
• Universidad Catolica Boliviana for Estrategia comunitaria con un enfoque inclusivo para la prevención y atención
de la violencia contra las mujeres (Bolivia).
• Johns Hopkins University for Improving global violence prevention by optimizing the construction of outcome
measures for impact evaluation of interventions (Global).
• Nossas for Mapa do Acolhimento (Brazil).
• KHANA Center for Population Health Research for WhatsApp chatline-SMARTgirl Chatline offering 24-hour
gender-based violence response and survivor support for female entertainment workers in
Cambodia (Cambodia).
• Project Empower for Zethembe (South Africa).
Click here to read more about the winners.
Development Marketplace Awards:
• Development Marketplace is to address Gender-Based Violence (GBV). This award was first launched in 2016
to honour GBV victims and survivors
• World Bank Group and Sexual Violence Research Initiative (SVRI) joint hands to support and provide fund for
researchers and innovators in low- and middle-income countries to study the violence against women and to
reduce the violence against them.
• This was held in memory of Hannah Graham, daughter of long time World Bank Group employee John
Graham.
• The winning teams receive up to US$100,000 each. These two partnerships have spent $5 million (USD) in
award funding.
About Institute for Human Development (IHD):
Institute for Human Development (IHD), non-profit autonomous institution was established in the year 1998 and
managed under the Indian Society of Labour Economics (ISLE). It aims to contribute towards building a society that
fosters and values an inclusive social, economic and political system that is free from poverty and deprivation.
Chairman – Prof. S.R. Hashim

Jharkhand Govt signs MoU with HDFC to Offer Digital Payment Solutions to its Traffic Dept
On July 9, 2020 Jharkhand government signed Memorandum of Understanding(MoU) with HDFC Bank to provide
digital payment solutions to the state’s transport department.
Key Info
i.The Digital solution will enable the traffic police to equip themselves with Point of Sale (POS)machines to collect
traffic- related penalties.

Report Errors in the PDF - ebooks@affairscloud.com Copyright 2014-2020 @ AffairsCloud.Com 38


Banking & Economy PDF
ii.The traffic rules violators can also use the HDFC Bank’s online payment facility to pay a fine.
Signing of MoU
The MoU was signed in the presence of Transport Secretary K Ravi and the bank’s Circle Head, Abhishek Kumar
About Jharkhand:
Capital– Ranchi
Chief Minister– Hemant Soren.
Governor– Draupadi Murmu.

Visa Partners with Federal Bank to Deploy Visa Secure to the Bank’s Cardholders
Visa, enters into partnership with Federal Bank to deploy Visa Secure to the bank’s cardholders. Visa Secure will
provide additional authentication layers for issuers and merchants to combat online frauds. In addition to this, the
upgraded technology provides a quick and smooth checkout experience to consumers.
• This partnership will also enhance the focus of the bank on customer-centric digital solutions that make online
shopping experiences seamless.
About Visa Secure
i.Visa Secure is a global authentication program that uses the latest standards of the EMV (Europay, Mastercard, and
Visa) 3DS (3D Secure) protocol.
ii.It is built on the EMVCo 3DS 2.0 protocol which makes e-commerce more protected. It will minimise customer
friction at point of purchase.
Note- The merchants and issuers have started to rollout Visa Secure globally.
Advantage of Visa Secure
i.Visa Secure will facilitate more user experiences across multiple payment channels, which includes mobile, web, in-
app, and connected devices.
ii.It will enable merchants to integrate the authentication process into the shopping experience in a better way.
iii.It will also provide quick, simple, secure and convenient authentication to cardholders.
About Visa:
Headquarters– Foster City, California, United States
Chairman and CEO– Alfred F. Kelly, Jr.
About Federal Bank:
Headquarters– Aluva, Kerala
MD & CEO– Shyam Srinivasan
Tagline – Your Perfect Banking Partner

India’s annual median GDP growth in FY21 to contract 4.5%: FICCI survey
In accordance with the latest round of Economic Outlook Survey of FICCI (Federation of Indian Chambers of
Commerce and Industry), India’s annual median Gross Domestic Product (GDP) growth forecast for FY2020-21 has
been declined to (-) 4.5% with a minimum and maximum growth estimate of (-) 6.4% and 1.5% respectively, as
compared to January 2020 estimate of 5.5%. The current round of the survey was conducted in the month of June
2020.
• The key reason behind this sharp decline is the lockdown to contain the rapid spread of COVID-19 which has
severely impacted economies and healthcare globally. Now in Post lockdown, continuation of social distancing
and quarantine measures is also affecting the growth prospects.
• The official growth numbers for the first quarter (Q1FY21) are expected to be released by the end of August
2020.
• As per the survey, there is a possibility of V-shaped recovery with government expenditure as in the current
situation; it is the only helping hand for financial position of the economy.
Key Points from survey- Growth prediction for Agriculture sector; Industry, service sector suffer
-The quarterly median forecasts indicate GDP growth to contract by (-) 14.2% in the first quarter of 2020-21, with a
minimum estimate of (-) 25% and a maximum estimate of (-) 7.4%.

Report Errors in the PDF - ebooks@affairscloud.com Copyright 2014-2020 @ AffairsCloud.Com 39


Banking & Economy PDF
-Economic activity-wise annual forecast indicated a median growth of 2.7% for agriculture and allied activities for
2020-21.
-Industry and services sector, on the other hand, are expected to contract by 11.4% and 2.8%, respectively in 2020-
21.
-The median growth forecast for IIP (India Industrial Production) has been put at (-) 11.5% for the year 2020-21
with a minimum and maximum range of (-) 15.3% and 1% respectively.
–Wholesale Price Index (WPI) based inflation rate is projected at -0.3% in 2020-21, with a minimum and maximum
range of (-) 1.5% and 2.5% respectively.
-On the other hand, Consumer Price Index (CPI) based inflation has a median forecast of 4.4% for 2020-21, with a
minimum and maximum range of 3.3% and 6% respectively.
-The median current account balance forecast has been pegged at (-) 0.3% of GDP for 2020-21.
About FICCI (Federation of Indian Chambers of Commerce and Industry):
Secretary General– Dilip Chenoy
President– Sangita Reddy
Headquarter– New Delhi

US remain India’s top trading partner in 2019-20 with USD 88.75 billion bilateral trade
In accordance with the data of the Ministry of Commerce and Industry, the United States (US) remained India’s top
trading partner for the second consecutive fiscal in 2019-20 as bilateral trade between the US and India stood at USD
88.75 billion as against USD 87.96 billion in 2018-19.
• The trade gap between the countries has also increased to USD 17.42 billion in 2019-20 from USD 16.86 billion
in 2018-19.
• On the other hand, the bilateral trade between India and China has dropped to USD 81.87 billion in 2019-20
from USD 87.08 billion in 2018-19. Trade deficit between the two nations have declined to USD 48.66 billion in
2019-20 from USD 53.57 billion in the previous fiscal.
• It should be noted that as per the study, the trend of widening Indo-US trade ties will continue in the coming
years but with regards to China, India is considering to cut import dependence and boost domestic
manufacturing.
Reason behind increasing Indo-US bilateral ties
Presence of Indian diaspora in the US is one of the main reasons for increasing bilateral trade.
Both nations are negotiating a limited trade pact with a view to remove differences at trade front and boost
commercial ties.
• While India is seeking relaxation in US visa regime, exemption from high duties imposed by the US on certain
steel and aluminium products, and greater market access for its products. On the other hand, the US wants
greater market access for its farm and manufacturing products, dairy items, medical devices, and data
localisation, apart from cut on import duties on some information and communication technology products.
Static Points:
-In 2018-19, the US first surpassed China to become India’s top trading partner.
-China was India’s top trading partner since 2013-14 till 2017-18. Before China, United Arab Emirates (UAE) was the
country’s largest trading nation.
About Ministry of Commerce and Industry:
Union Minister– Piyush Goyal (Constituency- Maharashtra)
Minister of State (MoS)– Hardeep Singh Puri, Som Parkash

BSE signs MoU with IIT Alumni Council to Encourage Startups Listing and Develop a ‘High Investor
Depth’ Platform
On July 13, 2020 BSE(formerly Bombay Stock Exchange) informed that it has signed a Memorandum of
Understanding(MoU) with IIT Alumni Council to encourage the listing of startups and develop a ‘High Investor
Depth’ platform for those startups.

Report Errors in the PDF - ebooks@affairscloud.com Copyright 2014-2020 @ AffairsCloud.Com 40


Banking & Economy PDF
Benefits of the MoU
i.The alliance will ensure transparency of the entity, improvement in governance, market discovery of share price,
provide liquidity for private equity investors and enable retail participation.
ii.It will help to minimise the listing time, complexity and cost for small and medium enterprise (SMEs) and start-ups
by streamlining, automating and standardizing documentation processes. For this, the category-1 merchant banks
will be empanelled.
iii.It will also enable to restore the startup platform by attracting a wide range of venture capital and private equity
funds to act as underwriters and market makers (backed by the BSE members in the front-end)
Key Info
i.IIT Alumni Council proposes to use Artificial Intelligence(AI)and other automation tools to reduce time, cost and
efforts involved in the listing process. This will improve the efficiency of the listing process and increase access to a
pool of investors for startups.
ii.The associated and volunteer managed funds will be encouraged to underwrite, act as anchor investors and as
market makers for theInitial Public Offering (IPO)s.
About BSE
Headquarters– Mumbai(Bombay), India
Chairman (Public Interest Director)– Justice Vikramajit Sen
MD & CEO– Ashishkumar Chauhan
About IIT Alumni Council:
The IIT Alumni Council is the largest global body of alumni, students and faculty across all the 23 IITs with over 100
city chapters globally. It is a virtual, borderless, volunteer-led, flat organisation.
President and Chief Volunteer- Ravi Sharma

Google Launches $10 Billion Digitization Fund in India:Google for India 2020
On 13 July, Alphabet Inc. & Google Chief Executive Officer Sundar Pichai had announced an investment of $10 billion
(Rs 75,000 crore) in India through the “Google for India Digitisation Fund” at the 6th Annual, Google for India
Event 2020 and also a First virtual event of Google in India.This will be utilised in another five to seven years to
digitize the Indian Economy.The fund will build India-first products and services.
The fund will be used to accelerate Google’s effort through a mix of Investments, partnerships, operations,
infrastructure, and ecosystem investments.
The Minister of communications, Electronics and IT, Law and Justice, Ravi Shankar Prasad took part in the
event.India’s HRD Minister Ramesh Pokhriyal participated to address the education landscape in India.
Investments are focused on 4 key areas of digitisation:
• Enabling affordable access to the internet and information for Indians in their own language.
• Providing various services relevant to the unique needs of India.
• Empowering business to embark in digital transformation
• Leveraging technology and Artificial Intelligence (AI) for social good areas like Healthcare, education and
agriculture.
Google is Set to Partner with Central Board of Secondary Education (CBSE) to Promote Online Education in
India
• Google will partner with CBSE and provide new initiatives to digitize classrooms.
• Google grants $1 million to the Kaivalya Education Foundation through Global Distance Learning Fund which
is a part of google.org, aims to help teachers in delivering virtual education.
• Google to allow one million teachers in 22,000 schools in India by the end of 2020.
• G Suits will be provided by Google. G Suit includes google tools like google docs, sheets and forms.
• It partners with Prasar Bharti, a public broadcasting agency for edutainment series for small and medium-
sized business in the country.
Notable apps of Google:-
Google Pay ,one of the familiar global product and Google’s AI-powered reading tutor app Bolo (now called Read
Along) is currently running in 180 countries.

Report Errors in the PDF - ebooks@affairscloud.com Copyright 2014-2020 @ AffairsCloud.Com 41


Banking & Economy PDF
About Google:
Headquarters- California, United States
CEO- Sundar Pichai
About CBSE:
Headquarters- New Delhi
Chairperson- Manoj Ahuja, I.A.S

Ministry of Shipping approved Rs 107 Crore for Modern Firefighting Facilities in Haldia Dock,
Kolkata
On 14th July 2020, Mansukh Laxmanbhai Mandaviya, Union Minister of State (I/C) for Shipping approved Rs.107
Crore to improve the firefighting facilities on five jetties of Haldia Dock Complex of Kolkata Port. The modern
firefighting facilities will enable safe and secure handling of the movement of petrochemical goods in the Haldia Dock
Complex.
The Need for Modern Firefighting Facility:
i.The current facility does not ensure the safety in handling the Liquefied petroleum gas (LPG) and other petroleum
products according to the guidelines of the Oil Industry Safety Directorate (OISD) of the Ministry of Petroleum and
Natural Gas.
ii.The LPG and Liquefied natural gas (LNG) cargo at the Haldia dock is expected to increase in the future, prioritising
the safety and security of the cargo operation in the major ports the ministry of shipping approved the fund to update
the existing facility into a modern firefighting facility.
About Ministry of Shipping & Ministry of Petroleum and Natural Gas:
Union Minister of State(MoS) for Shipping – Mansukh Laxmanbhai Mandaviya
Union Minister of Petroleum and Natural Gas– Dharmendra Debendra Pradhan

8.9% of the world’s population, went hungry in 2019: UN’s State of Food Security and Nutrition in
the World 2020
In accordance with the United Nations'(UN) annual report “The state of food security and nutrition in the world-
Transforming food systems for affordable healthy diets”, nearly 690 million people, or 8.9% of the world’s
population, went hungry in 2019, an increase of 10 million since 2018 and of nearly 60 million since 2014. The report
warned that the COVID-19 pandemic may add an additional 83 to 132 million people to the ranks of the
undernourished in 2020.
• The number of hungry people worldwide has been slowly rising and if this trend continues, the number of
undernourished people will exceed 840 million by 2030.
• The report has been prepared by 5 UN agencies, Food and Agriculture Organization (FAO), International Fund
For Agricultural Development (IFAD), United Nations Children’s Fund (UNICEF), World Food Programme
(WFP) and World Health Organization (WHO).
Key Points:
-The report noted that after steadily declining for decades, chronic hunger slowly began to rise in 2014 and continues
to do so. As of now, the world is neither progressing towards Sustainable Development Goal (SDG) target 2.1, of
ensuring access to safe, nutritious and sufficient food for all people all year round, nor towards target 2.2, of
eradicating all forms of malnutrition.
-Asia is home to the greatest number of undernourished people, an estimated 381 million.
-Africa has the most as a percentage of the population with nearly 20% of the continent’s people undernourished.
–3 billion people or more can’t afford to acquire the food needed for a healthy diet.
-In 2019, 21.3 percent (144.0 million) of children under 5 years of age were estimated to be stunted, 6.9 percent
(47.0 million) wasted and 5.6 percent (38.3 million) overweight, while at least 340 million children suffered from
micronutrient deficiencies.
-If current food consumption patterns continue, diet-related health costs linked to mortality and diet-related non-
communicable diseases are projected to exceed USD 1.3 trillion per year by 2030.

Report Errors in the PDF - ebooks@affairscloud.com Copyright 2014-2020 @ AffairsCloud.Com 42


Banking & Economy PDF
About United Nations
Headquarter– New York, United States (US)
Secretary General– Antonio Manuel de Oliveira Guterres

NABARD organised a ‘digital choupal’ to mark its 39th Foundation Day; 2 schemes each of Rs 5,000
cr announced
On July 12, 2020, National Bank for Agriculture and Rural Development (NABARD) has celebrated its 39th
Foundation day.On July 13, 2020, NABARD has organized its first “digital choupal”, a video conference which
brought together farmers from across the country who have been associated with NABARD in 7 development projects
in rural India viz. Rural Livelihoods Matter Immensely, Empowering Tribal Communities, Soil Health is Real Health,
Direct Marketing to Urban Consumers, Climate Resilience, Climate Proofing of Koutuguda and Collectivisation of
Weavers.
• On this occasion, NABARD has announced Rs 5,000 crore of refinance scheme for banks and financial
institutions and also earmarked another Rs 5,000 crore for turning Primary Agricultural Credit Societies
(PACS), into multi service centres. These are explained as follows:
Rs 5,000 crore of refinance scheme for banks and financial institutions
NABARD has announced Rs 5,000 crore of refinance scheme for banks and financial institutions for providing finance
to the beneficiaries of its 2,150 watershed development projects. The concessional line of assistance will be available
for three years from 2020-21 to 2022-23.
• These projects cover 23.04 lakh hectares of rainfed area watershed and tribal development project areas.
• The scheme will help the migrants, who have returned to their villages from urban areas following the COVID-
19 crisis, take up new occupations.
NABARD allocated Rs 5,000 cr for turning PACS into multi service centres
NABARD also introduced a grant-based scheme for computerisation of Primary Agricultural Credit Societies (PACS),
which will enable these ground-level entities to turn into multi service centres for providing seamless credit services
to their farmer members. PACS are building blocks of the country’s cooperative banking structure and are one-stop
shops for meeting the varied needs of the farmers
• 5,000 PACS will be upgraded in 2020 followed by 15,000 PACS in FY 2022 and 15000 PACS in FY 2023.
• PACS can support farmers in post-harvest and marketing activities in addition to playing a key role in the
physical and financial supply chain of commodities by acting as spokes to the upcoming Gramin Agriculture
Markets (GrAMs).
About National Bank for Agriculture and Rural Development (NABARD)
Chairman– G R Chintala
Headquarter– Mumbai, Maharashtra

IRDAI mandated insurance companies to launch ‘Corona Kavach’ & ‘Corona Rakshak’ amid
increasing COVID-19 cases
On July 10, 2020, Insurance Regulatory and Development Authority of India (IRDAI) made it mandatory for all
general and health insurers to launch short-term standard health policies for coronavirus disease (Covid-19)
viz. Corona Kavach and Corona Rakshak, due to rapid increase in COVID-19 cases in India. Notably, the premium
under both the products shall be the pan-India basis and no geographic location or zone-based pricing will be
allowed.
• Minimum entry age for both the policies will be 18 years and the maximum 65.
• Dependent children shall be covered from the age of three months to 25 years.
About Corona Kavach
Despite being a coronavirus-specific policy, the standard indemnity-based Covid-19 policy, ‘Corona Kavach’, will
cover the cost of treatment of any co-morbid conditions, including pre-existing conditions, along with the treatment
for the coronavirus infection or disease.
• Tenure range– 3.5 months to 9.5 months.
• Minimum sum insured– Rs 50,000

Report Errors in the PDF - ebooks@affairscloud.com Copyright 2014-2020 @ AffairsCloud.Com 43


Banking & Economy PDF
• Maximum sum insured– Rs 5 lakh.
• 5% discount for healthcare workers.
Under an indemnity plan, a policyholder is reimbursed the hospitalisation expenses to the extent of the sum insured
which includes room and boarding charges along with PPE kits, gloves, masks and such other similar expenses and
even AYUSH treatment. It will also cover home care treatment expenses of up to 14 days.
About Corona Rakshak
Corona Rakshak is a single-premium plan which will pay out 100% of the sum insured as lump-sum if the
policyholder is hospitalised, at least for 72 hours, after testing positive for Covid-19. The policy will cease to exist
once the claim is paid out.
• Minimum sum insured– Rs 50,000
• Maximum sum insured– Rs 2.5 lakh
• Tenure range– 3.5 months to 9.5 months.
Points to be noted:
-There is no option to pay premiums in instalments for both these products. Both policies will come with a 15 days
waiting period, during which no claim will be accepted.
-Dependent children shall be covered from the age of three months to 25 years. Only the indemnity policy comes with
the family floater option.
About Insurance Regulatory and Development Authority of India (IRDAI):
Chairman– Dr. Subhash Chandra Khuntia
Headquarter– Hyderabad, Telangana

Bloomberg Billionaires Index-Mukesh Ambani becomes World’s 6th richest man, surpassing Elon
Musk, Sergey Brin and Larry Page
On July 14, 2020 the Bloomberg Billionaires Index has ranked Mukesh Ambani, chairman of Reliance Industries
Ltd(RIL) as the World’s 6th richest man with a worth USD 72.4 billion after surpassing Elon Musk, co-founder &
CEO of Tesla and Alphabet Inc co founders, Sergey Brin and Larry Page.
Reason Behind
Ambani’s net worth has been raised as he has sold his shares in Jio Platform.
Key Info
i.Larry Page is ranked in 7th position with a net worth of USD 71.6 billion, Sergey Brin in 9th position(USD 69.4B) and
Elon Musk in 10th position(USD 68.6 billion).
ii.Warren Buffett is ranked in 8th position with a networth of USD 69.7B, after he gave away USD 2.9 billion to charity
Global Rankings
Top 3 in Global wise
Rank Name Total Net Worth
1 Jeff Bezos(US) USD 184B
2 Bill Gates(US) USD 115B
3 Bernard Arnault(France) USD 94.5B
Other Indians in Top 100
77 Azim Premji USD 16.9B
89 Shiv Nadar USD15.8B
Note– The Bloomberg Billionaires Index is a daily ranking of the world’s richest people.

Central Govt Provides Rs 1500 crore to Haryana for its Crop Diversification and Ponds Rejuvenation
Central government provides financial aid of Rs 1500 crore to Haryana for water conservation, which includes Rs
500 crore to expand crop diversification under the ‘Mera Pani Meri Virasat’ scheme and Rs 1000
crore to rejuvenate the ponds and treat water.

Report Errors in the PDF - ebooks@affairscloud.com Copyright 2014-2020 @ AffairsCloud.Com 44


Banking & Economy PDF
Key Info
The Central govt will also be providing Rs 1000 crore under the Jal Jeevan Mission(a central government initiative
under the Ministry of Jal Shakti, aims to ensure access of piped water for every household in India)
Other announcements
i.A scheme for rejuvenation of 14,000 ponds is implemented in the state in a phased manner.
ii.About 1000 water wells to be established in the state to control the decreasing groundwater table under the Atal
Bhujal Yojana
About ‘Mera Pani Meri Virasat’ Scheme
i.‘Mera Pani Meri Virasat’ scheme was launched for the purpose of crop diversification from maize / cotton / millet /
pulses / horticulture crops in 1.00 lakh hectares of land.
ii.The objective of crop diversification through the scheme is to promote 2 latest technologies with sustainable
farming, increase production and enable crop options to increase the income of the farmer.
iii.Under this scheme, farmers are encouraged to grow alternative crops other than paddy.
About Haryana:
Capital– Chandigarh
Governor– Satyadeo Narain Arya
Chief Minister– Manohar Lal Khattar

Nitin Gadkari Inaugurates and Lays Foundation Stones of 11 Highway Projects Worth Around Rs
20,000 crore in Haryana Virtually
On July 14, 2020 Union Minister for Road Transport, Highways and Micro, Small and Medium
Enterprises(MSMEs) Nitin Gadkari inaugurated and laid foundation stones of 11 Highway projects worth around
Rs 20 thousand crore in Haryana through a web based program(video conferencing) presided over by Haryana Chief
Minister, Manohar Lal Khattar. The projects are part of new economic corridors in the state.
• The 3 projects worth Rs 2,240 crore were inaugurated and foundation stones were laid for 8 projects worth Rs
17,757 crore.
Advantage of these projects
i.It will provide a smooth connectivity within the State and also to other States like Punjab, Rajasthan, Delhi and Uttar
Pradesh.
ii.It will help to cut down travel time, reduce fuels and cost and also enhance development in backward areas of the
State.
Key Info
A compensation of Rs 10,000 crore has been deposited directly in the accounts of the recipients for acquisition of the
land in association with these projects
Key People- Deputy Chief Minister Dushyant Chautala, Ministers and Members of Parliament and MLAs from the
state were also present in the event among others.
Inaugurated Projects
The projects for inauguration include, the following:
i.The 35.45 km 4-lane Rohna/Hasangarh to Jhajjar section of National Highway(NH) 334B at a cost of Rs 1183 crore.
ii.The 70 km 4-laning of Punjab-Haryana Border to Jind section of NH 71 at a cost of Rs 857 crore
iii.The 85.36 km 2-lane with paved shoulders Jind-Karnal Highway on NH 709 at a cost of Rs 200 crore.
Projects for which foundation stone laid
The projects for which foundation stone laid include the following:
i.The 227 km 6-lane access controlled Greenfield expressway from Ismailpur to Narnaul on NH 152D in 8 packages at
a cost of Rs 8650 crore.
ii.The 46 km 4-lane Gurugram Pataudi-Rewari section of NH 352W at a cost of Rs 1524 crore.
iii.The 14.4 km 4-lane Rewari Bypass at a cost of Rs 928 crore.
iv.The 30.45 km 4-lane Rewari-Ateli Mandi section of NH 11 at a cost of Rs 1057 crore.
v.The 40.8 km 6-lane Narnaul Bypass on NH 148B, NH 11 and Narnaul to Ateli Mandi section of NH 11 at a cost of Rs
1380 crore.

Report Errors in the PDF - ebooks@affairscloud.com Copyright 2014-2020 @ AffairsCloud.Com 45


Banking & Economy PDF
vi.The 40.6 km 4-lane Jind-Gohana (Pkg 1, Greenfield alignment) of NH 352A at a cost of Rs 1207 crore.
vii.The 38.23 km 4-lane Gohana-Sonipat section of NH 352A at a cost of Rs 1502 crore.
viii.The 40.47 km 4-lane UP-Haryana Border to Roha on NH 334B at a cost ofRs 1509 crore.
About Haryana:
National Park(NP)– Kalesar NP, Sultanpur NP
Wildlife Sanctuary(WLS)– Kalesar WLS, Bir Shikargah WLS, Chhilchhila WLS, Nahar WLS, Abubshahar WLS,
Bhindawas WLS, Khaparwas WLS, Khol Hi-Raitan WLS.

NABARD disbursed Rs 2.51 cr grant assistance to SHGs & Rs 221.89 cr to support 298 watershed
project in Karnataka
National Bank for Agriculture and Rural Development (Nabard) has disbursed Rs 2.51 crore of grant assistance to Self
Help Groups (SHGs) for promotion, skill development and digitisation of records in Karnataka. This funding is a part
of EShakti or Digitisation of SHGs an initiative of Micro Credit and Innovations Department (mCID) of NABARD. The
project is being implemented in 250 districts across the country.
• In Karnataka, the project E-Shakti is under implementation in seven districts. So far, 1.17 lakh SHGs are
digitised. The cumulative grant assistance of Rs 6.47 crore was released for SHG promotion.
NABARD extends grant assistance of Rs 221.89 cr to support 298 watershed project in Karnataka
In Karnataka, NABARD has extended a grant assistance of Rs 221.89 crore to support 298 watershed projects in over
three lakh hectares benefitting 66,500 families. The State has about 75% of rainfed area. The projects have resulted in
improving the crop productivity, diversification, soil and water conservation and socio-economic condition of the
beneficiaries.
• Notably, NABARD celebrated its 39th Foundation Day on July 12, 2020 and has announced Rs 5,000 crore of
refinance scheme for banks and also earmarked another Rs 5,000 crore for turning Primary Agricultural Credit
Societies (PACS), into multi service centres.Click here to Read about NABARD’s “digital choupal”.
Point to be noted:
-To mitigate the COVID 19 impact on rural population, NABARD has released Rs 2200 crore in the state to the Co-
operative and Regional Rural Banks under a special line of credit over and above the normal limits.
About National Bank for Agriculture and Rural Development (NABARD):
Chairman– G R Chintala
Headquarter– Mumbai, Maharashtra

CBDT launched tool for banks, co-operative societies & post offices to ascertain TDS applicability
rates on cash withdrawals
A new functionality of Income Tax Department namely “Verification of applicability u/s 194N” which is available
on www.incometaxindiaefiling.gov.in since July 1, 2020 has now made accessible for banks, co-operative societies,
and post offices through web-services by Central Board of Direct Taxes (CBDT). This feature will enable banks and
post offices to ascertain the TDS (Tax deduction at source) applicability rates on cash withdrawals of above Rs 20
lakh in case of a non-filer of the income-tax return (ITR) and that of above Rs 1 crore in case of a filer of the ITR.
• Through this tool, the entire process of ascertaining the TDS is automated and will be linked to the bank’s
internal core banking solution.
Amendment in Income Tax Act, 1961 w.e.f. July 1, 2020
As per Central Board of Direct Taxes (CBDT), there are cases when huge amount of cash is being withdrawn by
persons who have never filed ITR. In order to ensure filing of return by these persons and to keep track on cash
withdrawals by the non-filers, and to curb black money, the Finance Act, 2020 w.e.f. 1 July, 2020 further amended
Income-tax Act, 1961 to deduct TDS at the rate of 2% if cash withdrawal exceeds Rs.1 crore[if the person
withdrawing cash is a filer of the income-tax return] and If the person withdrawing cash is a non-filer of the income-
tax return, TDS is deductible at the rate 2% if cash withdrawal exceeds Rs.20 lakh and at the rate of 5% if it exceeds
Rs.1 Crore.

Report Errors in the PDF - ebooks@affairscloud.com Copyright 2014-2020 @ AffairsCloud.Com 46


Banking & Economy PDF
How will it ease the work for banks and post offices?
With the availability of “Verification of applicability u/s 194N” tool, now bank/co-operative society/post office has to
enter only the Permanent Account Number (PAN) of the person who is withdrawing cash for ascertaining the
applicable rate of TDS. On entering PAN, a message will be instantly displayed on the departmental utility indicating
the rate.
About Section 194N
In order to discourage cash transactions and move towards a cashless economy, the Finance (No.2) Act, 2019 has
inserted section 194N in the Income Tax Act w.e.f. 1 September, 2019. This section was again amended by the Finance
Act, 2020 by clause 83A of the Finance Bill, 2020.
About Central Board of Direct Taxes (CBDT):
CBDT, a part of the Department of Revenue in the Ministry of Finance works as the policy making body for the Income
Tax department.
Chairperson– Pramod Chandra Mody
Headquarter– New Delhi

Hurun Research’s World Richest Man List- Mukesh Ambani ranks 5th;Jeff Bezos tops
According to the Hurun’s Research’s World Richest Man List, Reliance Industries Limited (RIL) chairman & CEO
Mukesh Ambani (63) with worth 78 billion USD ranked 5th richest man in the world.He sustains as the wealthiest
Indian and he is the only Asian Tycoon in the world’s top 10 richest people as on July 2020.
i.Ambani’s wealth was down by 1% compared to pre-COVID levels.
ii.Mukesh Ambani shared his position with Tesla’s Founder, Elon Musk and Steve Ballmer (Former Microsoft CEO).
iii.Ambani has surpassed Warren Buffett’s whose net worth is 76 billion.
iv.Vaccine King Cyrus Poonawalla is another Indian in the global rich list at 57th position.
Hurun’s Global Ranking:
Rank Name Wealth
1 Jeff Bezos 200 Billion USD
2 Bill Gates 110 Billion USD
3 Bernard Arnault 94 Billion USD

ADB Appoints Election Commissioner of India, Ashok Lavasa as its New Vice-President
The Asian Development Bank(ADB) has appointed Ashok Lavasa as its Vice-President for Private Sector Operations
and Public–Private Partnerships, who is currently serving as one of the Election Commissioners of India(since
January 2018). He will succeed Diwakar Gupta, whose term will end on 31 August, 2020.
About Ashok Lavasa
i.Ashok Lavasa, a retired 1980 batch IAS officer of Haryana cadre. He was born on 21st October 1957.
ii.Post held– Prior to the post of Election Commissioner he served as Union Finance Secretary of India. Earlier he
served as the Union Secretary for the Ministry of Environment, Forests, and Climate Change and also as Union
Secretary for the Ministry of Civil Aviation among other senior posts.
Note– He still has over 2 years’ term left as an Election Commission of India.
There is also a possibility that he would have appointed as the Chief Election Commissioner (CEC) in 2021, because
current CEC Sunil Arora will retire on April 2021.
Awards Received– The International Alumnus Impact Award in 2019 from Southern Cross University, Australia, the
Distinguished Alumnus Award in the year 2015, LifeTime Achievement Award in the year 2018, Alumnus of the
Decade Award in the year 2019(by the Hindu College Old Students Association)
About Diwakar Gupta:
i.Diwakar Gupta assumed office as the Vice president of ADB in August 2015.
ii.Prior to this post he served as a Senior Advisor (Banking Project), Aditya Birla Nuvo Ltd., Mumbai.
iii.He also served as the Managing Director and Chief Financial Officer of the State Bank of India(SBI). He joined the
SBI in 1974 and held several positions at its various units and subsidiaries.

Report Errors in the PDF - ebooks@affairscloud.com Copyright 2014-2020 @ AffairsCloud.Com 47


Banking & Economy PDF
iv.He also worked as Independent Director on the boards of various business councils, financial services companies
and other private organizations in India.
About ADB:
Headquarters– Metro Manila, Philippines
President– Masatsugu Asakawa

Defence Ministry Entrust Power to Armed Force to Make Emergency Purchase Worth Rs.300 crores
On 15th July 2020, The Defence Acquisition Council (DAC) headed by Rajnath Singh, Minister of Defence has granted
special financial power to the armed forces to make emergency purchases of armed weapons worth upto Rs.300
crores to meet the emerging operational requirement.
Note:
Earlier, DAC has granted the special financial power to the Armed force following the Pakistan’s Pulwama attack on
14th February 2019.
Reason for granting Special power:
i.The ongoing situation on the Northern Borders with China and the need to strengthen the armed forces the DAC
decided to transfer the financial power to the armed forces.
ii.The DAC delegated the power to the armed forces to reduce the procurement timeline and to ensure the order
placement within 6 months and begin the delivery within 1 year.
iii.This is followed by India’s plan to purchase unmanned aerial vehicles, light tanks and anti-tank guided missiles
from the International arms market.
India’s recent arms purchase:
In June 2020, DAC approved the purchase of 21 MiG 29’s from Russia and approved the 12 Sukhoi’s manufactured
under licence in India by Hindustan Aeronautics Limited(HAL)
About Ministry of Defence:
Union Minister– Rajnath Singh
Minister of State– Shripad Yesso Naik

Govt to Increase Public Health Expenditure to 2.5% of GDP by 2025


In a meeting with the 15th Finance Commission the Union Health Minister Harsh Vardhan highlighted that the
government aims to gradually increase the public health expenditure to 2.5% of Gross Domestic Product(GDP) by
2025.
Statements made by Harsh Vardhan
i.The National Health Policy 2017 anticipates to raise public health spending to 2.5% of GDP by 2025.
ii.The Minister asserted that the outlay for primary health expenditure will also be made nearly two third of the total
public health expenditure in the country.
iii.The policy recommends that the state govt must spend nearly 8% of their total budget on the health sector by
2020.
To establish 1.5 lakh health and wellness centres by 2022
i.The government under the Aayushman Bharat programme proposes to establish 1.5 lakh health and wellness
centres by 2022. Of this about 19,000 have been already established.
ii.These centres will enhance healthcare facilities in the rural and Urban areas.
4% Health and Education Cess
Government has announced 4% health and education cess on personal income tax and corporation tax. Earlier it was
3%.

NABARD Launched Development Projects worth Rs 44 lakhs in Andaman and Nicobar Islands
National Bank for Agriculture and Rural Development(NABARD) launched development projects of Rs 44
lakhs in Andaman and Nicobar Islands to strengthen the Atmanirbhar Bharat through rural entrepreneurship.
• The development projects include formation and development of Self-Help Groups(SHGs).

Report Errors in the PDF - ebooks@affairscloud.com Copyright 2014-2020 @ AffairsCloud.Com 48


Banking & Economy PDF
Highlights of the Development projects
i.The development project will help in skill development of SHGs members to promote rural entrepreneurship at the
micro level.
ii.NABARD will conduct 385 village-level programmes. It will also provide training to leaders of newly formed SHGs
with an amount of more than RS 10 lakh.
iii.Women who are attached to SHGs in Little Andaman will be trained for Mushroom Cultivation and Bamboo
Handicrafts making
iv.A four -month long training programme for tour and travel agents in Swaraj Dweep will also be conducted.
Micro-Entrepreneurship Development Programme
An annual Micro-Entrepreneurship Development Programme to be conducted for 150 members of various SHGs in
Mayabunder in Middle Andaman.
NABARD partnered with CARI
NABARD has also partnered with the Centre of Agriculture Research Institute(CARI) to conduct agricultural
programmes with an amount of Rs 20 lakh.
Atmanirbhar Bharat
i.Banks are providing a sales platform to sell local products by financing Rural Haats in Gram Panchayats as a part of
Atmanirbhar Bharat initiative.
ii.A total of Rs 7.6 lakhs have been released to establish one such Haat in Namunaghar Panchayat of South Andaman.
About NABARD:
Headquarters– Mumbai, Maharashtra
Chairman– G R Chintala
About Andaman and Nicobar Islands
Capital– Port Blair
Lieutenant Governor– Admiral Devendra Kumar Joshi

Cushman & Wakefield’s 2020 Global Manufacturing Risk Index :India Ranked 3rd in cost and
operating conditions ;China Tops
In accordance with the “Cushman & Wakefield’s 2020 Global Manufacturing Risk Index (MRI)” India ranks 3rd in
the Baseline Scenario,3rd in Cost Scenario & 30th in Risk Scenario.
The Index ranks the countries in the above 3 Scenarios based on most-suitable locations for global manufacturing
among 48 countries in terms of cost, competitiveness and operating conditions.
Top 3 Ranking
In Baseline scenario, India is ranked at 3rd position after China and United States (US).It gives equal importance to
cost, competitiveness and operating conditions.
In cost scenario, India is at 3rd position after China and Vietnam. The MRI cost scenario places greater emphasis on
cost reduction to give a higher score to countries where operating costs, including labour, are lower.
In the risk scenario, India ranks 30th which has been topped by Canada. Countries that present lower levels of
political and economic risks are ranked higher.
The index had accessed the countries in four key areas viz. Bouncebackability, Conditions, Costs, and Risks. Following
table shows the top 3 countries:
Rank Country
Baseline Scenario
1 China
2 United States (US)
3 India
Cost Scenario
1 China
2 Vietnam

Report Errors in the PDF - ebooks@affairscloud.com Copyright 2014-2020 @ AffairsCloud.Com 49


Banking & Economy PDF

3 India
Risk Scenario
30 India
1 Canada
2 United States (US)
3 Singapore
India: Upcoming manufacturing hub globally
As per property consultant Cushman & Wakefield India is an upcoming manufacturing hub globally from an operating
conditions and cost-competitiveness perspective. In the backdrop of the COVID-19 scenario, the Indian government
has reiterated its agenda to launch fresh measures that would give a boost to setting up of a more conducive
environment to make India a manufacturing hub.
About MRI:
MRI is an annual ranking of the most suitable locations for global manufacturing among 48 countries in Europe,
Americas and the Asia-Pacific which scores each country against 20 variables.
• The MRI 2020 also included analysis of the impact of COVID-19 on the global manufacturing sector and ranked
countries by their projected ability to restart their manufacturing sectors once confinement measures are
relaxed and business starts to return to normal.
About Cushman & Wakefield:
President– John Forrester
Headquarter– Chicago, Illinois, United States (US)

Global Skills Index 2020 Report: India Ranked 34th in Business Domain, 51st in Data Science Domain
& 40th in Technology Domain
The report of Global Skills Index 2020, an in depth view at the state of the skills across the world prepared by
“Coursera”, World’s largest online learning platform shows that India Surges ahead of the peers in the business and
technology skill sets because of the growing young population.
India’s rank in the Global Skills Index 2020:
• In the Data Science Domain India secured the global rank 51(lagging) and secured 12th rank in the Asia
Pacific region.
• In the Business Domain India secured the global rank 34 (emerging)and secured 8th rank in the Asia Pacific
region.
• In the Technology Domain India secured the global rank 40 (emerging) and secured 10th rank in the Asia
Pacific region.
Global Skills Index 2020:
i.The global skills index report benchmarks the skill proficiency for 60 countries, 10 industries and 11 fields of study
in the domain of Data Science, Business and Technology.
ii.The report is made through looking across the 65 million users on the platform and drawing on the rich
performance data of the users in the past 12 months.
India & Top 3 Ranking:
Rank Country
Data Science Domain
51 India
1 Russia
2 Switzerland
3 Belgium
Business Domain

Report Errors in the PDF - ebooks@affairscloud.com Copyright 2014-2020 @ AffairsCloud.Com 50


Banking & Economy PDF

34 India
1 Switzerland
2 Austria
3 Denmark
Technology Domain
40 India
1 Russia
2 Belarus
3 Switzerland
Key Points:
i.The report stated that India’s workforce is expected to increase 27% to 600 million by 2022 providing an
opportunity for India to become a global talent hub for developing technology.
ii.The surplus talent will be visible in the industries like financial services, technology, telecommunications and
manufacturing by 2030.
iii.India’s focus on providing affordable and accessible education at college level will produce A’ level talents.
iv.The Skill India Initiative of the Government of India supports skilled and employable youths.
About Coursera:
Founders– Daphne Koller and Andrew Ng
CEO– Jeff Maggioncalda
Headquarters– Mountain View, California, United States of America

NABARD Sanctioned Rs 795 crore to West Bengal for Reconstructing Infrastructure Damaged by
Cyclone Amphan
National Bank for Agriculture and Rural Development(NABARD) has sanctioned Rs 795 crore to West
Bengal to reconstruct infrastructure which was damaged by Cyclone Amphan(May) under Rural Infrastructure
Development Fund (RIDF).
Background
i.The state government appealed for Rs 1,028 crore from NABARD to repair the damage under Rural Infrastructure
Development Fund (RIDF).
ii.After the appeal Rs 795 crore was sanctioned.
Rs 145 crore provided under RIDF
NABARD has already provided Rs 145 crore under RIDF to the state as special liquidity support to enhance rural
infrastructure considering the COVID-19 pandemic.
Prime Minister’s announcement to provide financial aid to West Bengal in view of the cyclone
i.Prime Minister Narendra Modi announced a rehabilitation package of Rs 1,000 crore for West Bengal.
ii.An advance assistance of Rs 1,000 crore will be arranged by the government of India.
iii.In addition to these, Rs 2 lakh will be given from the Prime Minister’s Relief Fund to the family of the deceased and
Rs 50,000 each for the seriously injured
Highlights of Cyclone Amphan
i.Super cyclone Amphan was formed over south-east Bay of Bengal on May 16. It made landfall between Digha in
West Bengal and and Hatiya island in Bangladesh with winds of about 120mph (190km/h) on May 20.
ii.This is the second pre-monsoon cyclone to form in the Bay of Bengal in the last two years.
iii.It is equivalent to a Category-5 hurricane on the Saffir-Simpson Hurricane Wind Scale, as per the space agency
NASA.
iv.The name ‘Amphan’was given by Thailand.
Other Related News:
i.World Bank grants Rs. 1,950 cr loans to West Bengal to tackle COVID- 19.

Report Errors in the PDF - ebooks@affairscloud.com Copyright 2014-2020 @ AffairsCloud.Com 51


Banking & Economy PDF
ii.NABARD grants Rs 1,050 crore special liquidity facility (SLF) to West Bengal during FY 21 for the benefit of farmers
& poor people.
About NABARD:
Headquarters– Mumbai, Maharashtra
Chairman– G R Chintala
About West Bengal:
Capital– Kolkata
Chief Minister– Mamata Banerjee
Governor– Jagdeep Dhankhar.

ICRA projects India’s GDP growth to contract by 9.5% in FY21 from earlier 5%
ICRA Limited (formerly Investment Information and Credit Rating Agency) projects India’s Gross Domestic
Product(GDP) to contract by 9.5%(-9.5%) in FY21 from 5% earlier forecasted as ongoing lockdowns in some states
have affected the recovery seen in May and June 2020.
ICRA’s Other projections for India
i.India’s economy to contract by 25% in the 1st quarter of FY21, in the 2nd quarter to contract by 12.4%, in the 3rd
quarter to contract by 2.3% and in the 4th quarter to contract by 1.3%.
ii.The agricultural Gross Value Added (GVA) to increase by 3.5- 4% in FY21.
iii.The rural economy to partly counter the slowdown in urban economy and is optimistic in regard to the outlook for
agricultural growth and rural consumption.
iv.It anticipates that there will be more unevenness as different regions are moving in and out of lockdowns and
continuing labour supply mismatches which affect the supply chains and consumption patterns.
Note– Most of the analysts have projected India’s GDP to contract in the range of 5-6.5% in FY 21.
About ICRA:
Registered Office– New Delhi, India
President– Ramnath Krishnan
Other Related News:
i.India’s GDP to contract by 4% in fiscal 2021 & downgraded its sovereign rating to ‘Baa3’ from ‘Baa2: Moody’s.
ii.Indian economy likely to contract by 1.5% in FY21: Reserve Bank of India (RBI) sponsored Survey of Professional
Forecasters (SPF) survey.

RBI Approves reappointment of Shyam Srinivasan as the MD & CEO of Federal Bank
On 16th July 2020, The Reserve Bank of India(RBI) approved the reappointment of Shyam Srinivasan as the
managing Director and Chief Executive officer of Federal bank Limited with effect from September 23, 2020 till
September 22, 2021.
• Under Regulation 30 of Securities and Exchange Board of India (SEBI), RBI approved the reappointment of
Shyam after the completion of the current term which ends on 22nd September 2020, along with the
remuneration proposed by the Federal Bank.
Limit of tenure:
i.In June 2020, RBI proposed to limit the tenure of the CEO’s and Whole Time Directors(WTDs) from the promoter
and non-promoter groups to 10 years and 15 years respectively.
ii.The RBI also proposed to limit the upper age limit of the CEOs and WTDs of the Banks to 70 years.
About RBI:
Governor– Shaktikanta Das
Headquarters– Mumbai, Maharashtra
About Federal Bank:
Tagline– Your Perfect Banking Partner
Managing Director & CEO– Shyam Srinivasan
Headquarters– Aluva, Kerala

Report Errors in the PDF - ebooks@affairscloud.com Copyright 2014-2020 @ AffairsCloud.Com 52


Banking & Economy PDF
Related News:
i.Federal bank, MoneyGram tie-up for direct-to-bank deposits in India
ii.RBI extends regulatory benefits under SLF-MF scheme dated 27.4.2020 to all banks.

UN’s Global Multidimensional Poverty Index: India Record Largest Reduction in Number of People
Living in Poverty
On 16th July 2020, The United Nations’ “The Global Multidimensional Poverty Index(MPI) 2020: Charting pathways
out of multidimensional poverty: Achieving the SDGs” states that India records the largest reduction in the number of
People living in poverty as around 273 million people moved out of the multidimensional poverty over 10 years
(2005/2006 – 2015/2016).
The report released by the Human Development Report Office of the United Nations Development
Programme (UNDP) and the Oxford Poverty and Human Development Initiative (OPHI) focuses on how
multidimensional poverty has declined.
Multidimensional Poverty:
The multidimensional poverty encloses various deprivations experienced by the poor people like poor heath, lack of
education, inadequate standard of living, poor quality of work, threat of violence and hazardous living environments.
Features of the Report:
i.The data of the reports shows that out of 75 countries studied, 65 countries have reduced multidimensional poverty
levels between 2000 and 2019.
ii.Among the 65 countries, 50 countries have reduced the number of people living in poverty.
Armenia (2010–2015/2016), India (2005/2006–2015/2016), Nicaragua (2001–2011/2012) and North Macedonia
(2005/2006–2011) have reduced their global MPI value by half in 5.5 to 10.5 years, these countries accounts one fifth
of the world’s population.
iii.The information related to the 273 million people moving out of poverty in India was based on the population data
from the United Nations Department of Economic and Social Affairs (UNDESA).
iv.The report states that both India and Nicaragua have halved the MPI value among children in 10 and 10.5 years
respectively.
Impacts of COVID-19:
i.The report accounts the data before the COVID-19 pandemic.
ii.With no available data to measure the rise of global poverty after the pandemic, simulations based on the expected
impacts of Covid on nutrition and school attendance suggest that progress across 70 developing countries is expected
to experience a setback.
iii.Around 50% of the 1.3 billion (644 million) are under the age of 18, and 107 million are 60 or above 60 who are
vulnerable to COVID-19.
Impacts on Children:
i.The report states that around 1.3 billion people i.e 22% of the population across 107 developing countries lives in
multidimensional poverty which burdens the children.
ii.Around 10 countries accounted for more than 60% of the unvaccinated children and 40% of the children are
unvaccinated for DTP3(diphtheria, tetanus and pertussis) lives in Nigeria, India, Pakistan and Indonesia.
iii.Around 2.6 million children in India are undervaccinated.
About United Nations Development Programme (UNDP):
Administrator– Achim Steiner
Headquarters– New York, United States of America
Recent Related News:
i.India ranks 74th, Sweden tops: WEF’s global Energy Transition index 2020
ii.NCAER Business confidence index (N-BCI) declined to 77.3 in Q4 FY20

Report Errors in the PDF - ebooks@affairscloud.com Copyright 2014-2020 @ AffairsCloud.Com 53


Banking & Economy PDF
India Remains as Lower-Middle-Income Economy: World Bank Country Classifications by Income
Level 2020-2021
According to the New World Bank country classifications by income level 2020-2021, India remains in the
classification, Lower-Middle-Income Economy. In total 218 countries have been included in this classification.The
World Bank classifies countries into four income groups namely: low income , lower-middle income, upper-middle
income, and high-income economies.
• The classifications are updated each year on July 1 and are based on Gross National Income(GNI) per capita in
current USD(using the Atlas method exchange rates) of the previous year.
For FY 2021 the classification was made on the basis of the following GNI per capita
Low-income economies: GNI per capita, below USD 1,035
Lower Middle-income economies: GNI per capita between USD 1,036 and USD 4,045
Upper middle-income economies:GNI per capita between USD 4,046 and USD 12,535
High-income economies:GNI per capita of above USD 12,536
Note: The GNI per capita used for FY 2021 classification do not reflect the impact of COVID-19.
Neighbouring Countries:
Classification
Country New group Old group GNI per Capita(2019) as of July 1, 2020
Indonesia Upper-middle income Lower-middle income USD 4,050
Mauritius High income Upper-middle income USD 12,740
Nepal Lower-middle income Low income USD 1,090
Sri Lanka Lower-middle income Upper-middle income USD 4,020
Highlights
i.Economies that moved to a higher category- In addition to Mauritius and Nepal, Benin, Nauru, Romania and
Tanzania have moved to a high category compared to the previous year classification
ii.Economies that moved to a lower category- In addition to Sri Lanka Algeria, Sudan have moved to a lower category
compared to their previous year classification.
iii.In total 29 countries are classified under Low income which include Afghanistan, Ethiopia, Mozambique among
others. In total 50 countries are classified under Lower-middle income which include, Bangladesh, Kenya, Bolivia
among others.
iv.In total 56 countries are classified under Upper-middle income economies which include Albania, Brazil, China
among others.In total 83 countries are classified under High income economies which include, Australia, Austria,
Canada among others.
About World Bank
Headquarters– Washington, D.C., United States
President– David R. Malpass
Recent Related news
i.India Retains its Position as 3rd-largest Economy on PPP basis for 2017; China Tops: World Bank Data.
ii.India remains at 108 in FIFA rankings, Belgium tops.

Centre’s fiscal deficit to increase to 7.6% & India’s GDP to contract by 5.3% in FY21: India Ratings
and Research’s Report
As per the research report of India Ratings and Research(Ind-Ra), Centre’s fiscal deficit is estimated to increase
to 7.6% in FY21, which is twice the budget estimate as the country spends more to mitigate the impact of COVID-19
pandemic, facing a shortfall in revenues. The report projects India’s Gross Domestic Product(GDP) to contract
by 5.3% in FY21.
Key Info
• The aggregate of the central and state fiscal deficit in FY21 is 12.1%, with the states contributing 4.5%.
• States like Assam, Goa, Gujarat and Sikkim are likely to experience double digit contraction in GDP.

Report Errors in the PDF - ebooks@affairscloud.com Copyright 2014-2020 @ AffairsCloud.Com 54


Banking & Economy PDF
Note:
i.The decrease in the growth and revenue will impact the fiscal deficit, which is considered an important
macroeconomic health indicator.
ii.The government has already announced a stimulus package that damaged the fiscal by 1.1%.
Highlights
Financial Sector-There will be a significant impact on the asset quality of the financial sector due to growth
slowdown. The Banks and Non-banks will require more capital to continue lending.
Supply Side– The supply side has been severely disrupted by the pandemic, as production and sale were allowed
only in the areas classified as ‘essential’ during the lockdown.
Manufacturing Sector-The reverse migration from cities and industrial cities to their hometowns will delay the
recovery of the manufacturing sector and may also translate into higher wages.
Significant remittance-The states who receive significant remittances( India, the largest recipient of such transfers
from any country’s diaspora) will be impacted as return and/or repatriation of expatriates to India has its own
consequences for the Indian economy.
About Ind-Ra:
Ind-Ra is a 100% owned subsidiary of the Fitch Group.
Headquarters– Mumbai, Maharashtra
Managing Director & CEO– Rohit karan Sawhney
Recent Related news
i.India’s economy is predicted to -7.3% for FY 21 due to the 2nd wave of COVID-19: Organisation for Economic Co-
operation and Development (OECD)’s June 2020 Economic Outlook (EO).
ii.India’s GDP to Fall 3.1% in 2020; Geo-Political Tensions Will Increase Risk: Moody’s Global Macro Outlook report.

Indian direct selling industry records USD 2.477 bn sales in 2019, ranked 15th globally: WFDSA‘s
report
In accordance with the report of “The Global Direct Selling – 2019 Retail Sales” by the World Federation of Direct
Selling Associations (WFDSA), India was ranked 15th in the global direct selling industry with sales of USD 2.477
billion in 2019. This has shown a growth of 12.1% in comparison to the 19th rank in 2018. The ranking has been
topped by the United States (US) which contributed 20% of the global direct selling industry with sales of USD 35.21
billion, followed by China with 13% contribution at 2nd rank, and Korea and Germany with 10% contribution each at
3rd position.
• In terms of the number of direct sellers, India has been ranked at 6th position, providing employment to 57.50
lakh people.
• On the global front, the direct selling industry has reported an overall decline of 4.3 % in sales to USD 180.479
billion in 2019.
Following table shows the top three global positions:
Rank Country
15 India
1 United States (US)
2 China
3 Korea & Germany
Key Points:
-As per the latest joint report by Indian Direct Selling Association (IDSA) and data insight firm Kantar, India has
recorded the highest year on year growth rate of 12.1% .
-India has also recorded the highest Compound annual growth rate (CAGR) of 16.3% over the period of last three
years, amongst the top 20 direct selling markets across the globe. It has grown from Rs 8,308 crore in 2015-16 to Rs
13,080 crore in 2018-19
–Wellness tops the chart of leading segments in Direct Selling followed by cosmetics and personal care.
-The Indian Direct Selling Industry’s contribution to the exchequer stood at around Rs 2,500 crore in 2018-19.

Report Errors in the PDF - ebooks@affairscloud.com Copyright 2014-2020 @ AffairsCloud.Com 55


Banking & Economy PDF
Click Here for Official Link
About World Federation of Direct Selling Associations (WFDSA):
Headquarter– Washington DC, United States (US)
WFDSA Advocacy Chair– Ms. Kim Drabik

Ashwini Kumar Tewari was appointed as MD & CEO of SBI Cards and payment services
SBI (State Bank of India) cards and payment services appointed, Mr. Ashwini Kumar Tewari as its new Managing
Director (MD) and Chief Executive officer (CEO) who is intended to be in charge from August 1, 2020. His new
appointment will be valid for 2 years.
Highlights:
i.Tewari replaces Mr. Hardayal Prasad, the MD and CEO who will quit the directorship of the company on July 31,
2020 due to voluntary retirement.
ii.Prasad succeeds Mr. Vijay Jasuja who retired on January 31, 2018.
iii.Tewari was Country Head of SBI’s US (United States operation) at New York.
iv.He will hold all the necessary approvals of the shareholders at the Annual General Meeting which was decided by
the Board meeting on 15 July 2020.
About State Bank of India(SBI):
SBI is the Largest bank in India with 23% market share asset.
SBI Cards and payment services are the country’s second-largest credit card issuer.
Headquarters– Mumbai, Maharashtra
Chairman– Rajnish Kumar (25th Chairman, appointed on 7th October 2017)
About SBI cards:
Headquarter – Gurgaon, Haryana.

Tamil Nadu Government Signed 8 MoUs worth Rs 10,399 crore with Industrial Groups for
Employment Creation
On July 20, 2020 The Tamil Nadu government signed 8 Memorandums of Understandings (MoUs) worth Rs 10,399
crore with companies to create employment opportunities for over 13,507 people. The MoUs were signed in the
presence of Chief Minister Edappadi K. Palaniswami in the Secretariat at Fort St George in Chennai.
• Out of these 8 projects, MoUs were signed for 5 projects directly and for 3 projects through video display.
• Industry Minister M.C. Sampath, Chief Secretary Shanmugam, senior government officials and representatives
of industry among others were also present on the occasion.
Proposed Projects– The proposed projects include the following: production of solar cells and modules, air
compressors, ductile iron foundry, industrial parks, cashew nut processing, data centre and a foundry expansion
project.
Companies signed MoU:
i.Vikram Solar, to invest in a solar cell and module manufacturing project with an investment of Rs. 5,423 crore at the
Indospace Industrial Park in Oragadam, Kanchipuram District and provide employment to 7,542 persons.
ii.Hiranandani Group’s Yotta Infrastructure to invest Rs 4,000 crore and provide jobs to 2,500 people by setting up an
information data center park in Chengalpattu district.
iii.Coimbatore based ELGi Equipments to invest Rs 250 crore in Coimbatore and provide employment to about 600
persons by setting up an air compressor unit.
iv.CGD Satharai Private Limited to invest Rs 250 crore in Walajabad, Kanchipuram district and provide jobs to
around 1,500 persons by setting up an industrial park.
v.NDR Integrated Industrial Park to invest Rs 200 crore in Ranipet district to develop an industrial park. This
provides about 500 jobs.
vi.Coimbatore-based Aqua Group to invest Rs. 200 crore for Aquasub’s ductile iron foundry project in Coimbatore
District which will provide employment to about 400 persons.
vii.Coimbatore-based firm, JS Autocast, to invest Rs 40 crore to expand Kudranesala project at Chipkot Perundurai
Industrial Park, Erode District,

Report Errors in the PDF - ebooks@affairscloud.com Copyright 2014-2020 @ AffairsCloud.Com 56


Banking & Economy PDF
viii.GI Agro Tech to invest Rs 36 crore to set up a cashew nut processing unit at Villupuram. This will provide
employment to about 465 persons .
About Tamil Nadu:
Governor– Banwarilal Purohit
National Parks(NP)– Gulf of Mannar Marine NP, Guindy NP, Mudumalai NP, Mukurthi NP.
Recent Related News
i.Tamil Nadu government inks 17 investment MoUs worth Rs 15,128 cr with corporates to provide employment
opportunities to 47,150 people.
ii.Petroleum Minister, Dharmendra Pradhan & CM Edappadi K.Palaniswami Inaugurates Compressed Bio-Gas(CBG)
Plants at Namakkal and CBG Fuel Stations at Pudhuchattiram and Rasipuram over a virtual platform.

Harsh Vardhan Launched PNB’s Nationwide CSR Campaign to Fight COVID-19 Pandemic
On July 20, 2020 Union Minister of Health and Family Welfare, Harsh Vardhan launched Punjab National
Bank(PNB)’s nationwide Corporate Social Responsibility(CSR) campaign to prevent the spread of COVID-19 by
distributing face masks and sanitizers.
• The bank will be providing COVID-19 related prevention materials to 662 districts throughout the country.
Key People
S.S. Mallikarjuna Rao, Managing Director(MD) and Chief Executive Officer(CEO) and other senior officials of PNB were
also present. In addition to them, PNB’s 22 zonal offices across the nation also participated through video
conferencing.
About Ministry of Health and Family Welfare:
Minister of State– Ashwini Kumar Choubey
About PNB:
Headquarters– New delhi, India
Recent Related News:
i.Indian Farmers Fertiliser Cooperative (IFFCO) organized a social awareness campaign,‘Break the CORONA
Chain’ on Ground zero in different states across India.
ii.Surakshit Dada-Dadi & Nana-Nani Abhiyan campaign launched virtually by NITI Aayog to protect senior citizens
during COVID-19.

Ramesh Babu Boddu Appointed as MD and CEO of Karur Vysya Bank


On July 21, 2020 Karur Vysya Bank (KVB)’s Board of Directors has co-opted Ramesh Babu Boddu as an Additional
Director and appointed him as the Managing Director(MD) & Chief Executive Officer(CEO) for three years tenure. He
replaces P R Seshadri, who resigned in January 2020 citing personal reasons.
• The tenure is effective from the date of taking charge, on the terms and conditions approved by the Reserve
Bank of India(RBI) under Section 35B of the Banking Regulation Act, 1949.
About Ramesh Babu Boddu
Career in SBI
i.Prior to this appointment he was serving as a Deputy Managing Director(DMD) in State Bank of India(SBI). He
retired from the post in April 2020. He also worked as Chief Operating Officer of the bank.
ii.Earlier to his appointment as DMD he served as Chief General Manager in SBI.
About KVB:
Headquarters– Karur, Tamil Nadu
Non-Executive Independent (Part-time) Chairman– N. S. Srinath
Tagline– Smart Way to Bank.
Recent Related News:
i.Arun Singhal appointed as CEO of Food Safety and Standards Authority of India (FSSAI).
ii.Govinda Rajulu Chintala appointed as NABARD chairman, Shaji KV and PVS Suryakumar appointed as Deputy
Managing Directors (DMD) of NABARD

Report Errors in the PDF - ebooks@affairscloud.com Copyright 2014-2020 @ AffairsCloud.Com 57


Banking & Economy PDF
Puducherry Chief Minister V Narayanasamy Presented Rs 9000 Crore Tax Free Budget for FY 20-21
Puducherry Chief Minister V Narayanasamy, who also holds the finance portfolio presented Rs 9000 crore tax free
budget for the financial year 2020-2021 in the assembly. The budget is Rs 575 crore more than the previous year’s
outlay of Rs 8,425 crore and Rs 1073 crore more than the revised budget outlay of Rs 7927 crore in 2019-2020.
Note– The budget was presented amid objections from Lt Governor Kiran Bedi and without her customary address.
Highlights of the budget:
i.Resources of the budget
• Rs 5257 crore of the budget is from the state’s own resources.
• Rs 2023 crore the budget from central assistance and centrally sponsored schemes
• Rs 1710 crore of the budget will be from open market borrowings and negotiated loans from central financial
institutions (National Bank for Agriculture and Rural Development- NABARD, Housing and Urban
Development Corporation Ltd (HUDCO) and others).
ii.Major Expenses
The major expenditure will be the following:
• Rs 1966 crores for salaries,Rs 1177 crores for pension, Rs 1625 crores for repayment of loans and interest and
Rs 1525 crores for power purchase.
iii.Old age pension scheme– Rs 896 crores has been earmarked for the old age pension scheme.
iv.Grants– Rs 864 crores earmarked for grants to educational institutions, public sector undertakings and
cooperative institutions.
v.Bank Loans– Bank loans of Rs 87 crore will be arranged for students to pursue their education.
vi.Free breakfast scheme
The free breakfast scheme which is now implemented in Puducherry schools will be named after former Tamil Nadu
Chief Minister late M Karunanidhi. Nutritious food will be included in it, presently only milk is supplied to students.
vii.To implement infrastructure development scheme
The government will implement a massive infrastructure development scheme with a loan of Rs 746.53 crore from
JICA (a Japanese International Cooperative Agency) to meet the UT’s power requirements.
About Puducherry
Capital– Pondicherry
Recent Related News:
i.Gujarat Deputy Chief Minister and Finance Minister Nitin Patel presented a Rs 2,17,287 crore budget for the
financial year 2020-21.
ii.Andhra Pradesh becomes the 1st Indian State to present its Budget amid COVID-19. The budget of Rs 2.24 lakh
crore for the FY20-21 was presented.

India ranked among 10 worst countries for working people: ITUC Global Rights Index 2020
In accordance with the 7th edition of “International Trade Union Confederation (ITUC) Global Rights Index
2020-The World’s Worst Countries for Workers”, India became the new entrant in the 10 worst countries for
working people in the world, with a rating of 5 that states “No guarantee of rights”. The index ranked 144 countries
on the basis of respect for workers’ rights.
• The other countries among the worst ten are- Bangladesh, Brazil, Colombia, , Kazakhstan, Philippines, Turkey
and Zimbabwe along with Egypt & Honduras being the other two new entrants apart from India.
• On the region front, Middle East and North Africa (MENA) is the worst region with 4.44 rating followed by
Asia-Pacific with 4.09 in the world for working people for seven years (2014-2020).
• India’s ranking was based on three factors viz. Mass dismissals, Brutal repression of strikes, and Regressive
laws.
Index perspective with respect to India
In 2020, the police in India used disproportionate violence against workers protesting to call for the payment of due
wages and better working conditions citing incidents from Tamil Nadu and Telangana. Following are the companies
mentioned in index that Violates Workers’ Rights in India
• Pricol in Coimbatore, Tamil Nadu

Report Errors in the PDF - ebooks@affairscloud.com Copyright 2014-2020 @ AffairsCloud.Com 58


Banking & Economy PDF
• Tamil Nadu Generation and Distribution Corporation (TANGEDCO), Tamil Nadu
• Telangana State Road Transport Corporation (TSRTC), Telangana
• Motherson Automotive Technologies & Engineering (MATE), Chennai, Tamil Nadu
Index perspective on global front
-85% of countries violated the right to strike and 80% of countries violated the right to collectively bargain.
-The number of countries that denied or constrained freedom of speech increased from 54 in 2019 to 56 in 2020.
-Workers were exposed to violence in 51 countries.
-Workers had no or restricted access to justice in 72% of countries.
-Workers experienced arbitrary arrests and detention in 61 countries.
The number of countries which impeded the registration of unions increased from 86 in 2019 to 89 countries in
2020.
Click Here for Official Link
About ITUC Global Rights Index:
It depicts the world’s worst countries for workers by rating countries on a scale from 1 to 5+ with 1 being the best
rating and 5+ the worst rating. Violations in each country are recorded annually from April to March against a list of
97 indicators.
About International Trade Union Confederation (ITUC):
General Secretary– Sharan Leslie Burrow
President– Ayuba Philibus Wabba
Headquarter– Brussels, Belgium
• The ITUC affiliates in India are the Confederation of Free Trade Unions of India (CFTUI), the Hind Mazdoor
Sabha (HMS), the Indian National Trade Union Congress (INTUC) and the Self-Employed Women’s Association
(SEWA).
Recent Related News:
On May 13, 2020 According to the World Economic Forum(WEF)’s global Energy Transition
Index(ETI) 2020, India has moved up two places from rank 76th(2019) to 74th with a score of 51.5%.
Sweden(74.2%) topped for the 3rd consecutive time followed by Switzerland(73.4%) and Finland(72.4%).
LIC-UBI announced corporate agency pact
Union Bank of India (UBI), after its amalgamation with Corporation Bank and Andhra Bank, has announced a
corporate agency agreement with Life Insurance Corporation of India (LIC) to distribute latter’s policies.
• As per the pact, UBI can act as a financial supermarket by offering LIC products of term insurance, pension,
plan, children plan, ULIPs, and endowment schemes to its account holders, whereby they can select products
according to their choice.
About Life Insurance Corporation of India (LIC):
Chairman– MR Kumar
Headquarter– Mumbai, Maharashtra
Virtual Assistant– LIC Mitra
About Union Bank of India (UBI):
Managing Director (MD) & Chief Executive Officer (CEO)– Rajkiran Rai G.
Headquarter– Mumbai, Maharashtra
Virtual Assistant– UVA
Tagline– Good People to Bank with
Recent Related News:
On May 26, 2020, Vakrangee Limited (VL), a technology company based in Mumbai (Maharashtra), has entered into
the Corporate Agency (Registration Code CA0249) partnership with Life Insurance Corporation of India (LIC) to
distribute latter’s micro insurance products in unserved and underserved areas through its Nextgen Vakrangee
Kendra network spread across India.

Report Errors in the PDF - ebooks@affairscloud.com Copyright 2014-2020 @ AffairsCloud.Com 59


Banking & Economy PDF
FamPay launched India’s 1st numberless card, Famcard
FamPay, India’s first Neobank for teenagers, in partnership with IDFC First Bank launched India’s 1st numberless
card, FamCard. FamPay with National Payments Corporation of India(NPCI) developed this card. This card is
accepted across the RuPay payment network of merchants. It is similar to a debit card which enables teenagers to
transact in a cashless manner.
• Teenagers can use the card independently without the intervention of the parents.
• In addition to the card, teens are provided with their own unique Unified Payment Interface(UPI) ID.
Highlights of Famcard
Offers a combination of parental control and independence for teens
It allows the parent to create a FamPool account which is a group account for the family. Through this account
parents will be able to track where the money is spent from this account. As for teens, they get their own wallets.
Online and Offline payments
Minors can make online(UPI & Peer to Peer- P2P) & offline payments without even having to set up a bank account.
Card’s details saved on FamPay App
The details of the card are saved on the FamPay app. There is no need to refer the physical card to fill in details during
online transactions
Flash PIN- For every transaction a ‘Flash PIN’ is generated. Teens can just flash this PIN to the cashier for offline
payments without the need of entering the PIN by themselves on the POS machines.
Can maintain zero balance & no hidden cost– They can maintain a zero balance and there are no hidden charges or
costs attached to any transactions.
Secure and safe
All the transactions are protected with device locks like fingerprint, Face ID, pattern lock or PIN. If the card info is
stolen or lost, the card can be paused, blocked and managed at fingertips through the app.
Order
The card can be ordered after the account is set up on the app, after both the parent and teen complete their Know
Your Customer(KYC) online.
About FamPay app
The app was launched recently. It is available for Android on the Google Play Store and iOS on the Apple Store
Note– RuPay, the first-of-its-kind India’s domestic Debit and Credit payment network, was launched by NPCI.
About FamPay:
Headquarters– Bengaluru, Karnataka
Founders– Kush Taneja & Sambhav Jain
About NPCI:
Registered Office– Mumbai, Maharashtra
Non Executive Chairman– Biswamohan Mahapatra
MD & CEO– Dilip Asbe
Recent Related News:
i.Karur Vysya Bank has launched India’s 1st prepaid wallet card ‘Enkasu’ (My cash in Tamil) in Karur,Tamil Nadu
(TN) under its Quit cash movement.
ii.Empays Payment Systems partners with Mastercard to launch “Cardless ATM powered by Mastercard” in India.
With thisIndia becomes 2nd nation to have contact-free ATM cash withdrawals.

India’s GDP to contract by 6.1% in FY21: Nomura


On July 21, 2020 As per the report of Nomura Holdings, Inc, a japanese brokerage firm, India’s Gross domestic
Product(GDP) is likely to contract by 6.1%(-6.1%) in FY21 as economic activity continues to be weak due to COVID-
19 pandemic.

Report Errors in the PDF - ebooks@affairscloud.com Copyright 2014-2020 @ AffairsCloud.Com 60


Banking & Economy PDF
Other Projections of Nomura
Quarter
i.India’s economy to contract by 15.2% in the June quarter. For the rest of this fiscal year, GDP will never be positive.
It is to be noted that this quarter will be ”nadir”(the lowest or most unsuccessful point in a situation) from a growth
perspective.
ii.It estimated contractions of 5.6% in the September quarter, 2.8% for the December quarter and 1.4% in the March
quarter.
Inflation
It suggests that there will be an increase in inflation, which will further pull down the GDP in real terms.
RBI
i.The Reserve Bank of India(RBI) is expected to suspend its policy review in August and cut rates by 25 basis points
each in the October and December reviews.
ii.On the monetary policy front, the RBI has already cut rates by a cumulative 1.15% since the beginning of the
pandemic.
Growth Estimates- After analysing ”ultra” high frequency indicators such as various mobility indices, employment
and electricity demand, the growth estimates are calculated.
Note– All the analysts expect a contraction in the GDP due to the COVID-19 pandemic, which has impacted both
supply and demand forces in the economy since March 2020.
About Nomura Holdings, Inc:
Headquarters– Tokyo, Japan
President and Group CEO– Kentaro Okuda
Recent Related News:
i.As per the India Ratings and Research (Ind-Ra), India’s Gross domestic Product (GDP) is likely to shrink by 5.3% in
FY20-21, the lowest GDP growth in the Indian history.
ii.S&P Global Ratings predicted that Indian economy will shrink 5 per cent in FY21.

India’s HIL supplied 20.60 MT of DDT 75 % WP to South Africa for malaria control
On July 21, 2020, HIL (formerly Hindustan Insecticides Limited), a PSU under the Ministry of Chemicals and
Fertilizers has supplied 20.60 Metric tonne of DDT (dichloro-diphenyl-trichloroethane) 75 % Wettable powder (WP)
to South Africa for their Malaria control program in three province adjoining Mozambique. The region is highly
affected with Malaria and it has reported maximum morbidity and mortality due to the disease in recent years.
• It is to be noted that HIL is the sole manufacturer of DDT globally.
HIL supplies to other countries:
-The company is in process of supplying DDT 75% WP to Zimbabwe (128 MT) and Zambia (113 MT) in the current FY
2020-21.
-It has recently exported 25 MT of Malathion Technical 95% to Iran under Government-to-Government initiative for
the Locust Control Programme and also exported Agrochemical-fungicide (32 MT) to Latin American region.
Malaria- A global Disease
In 2018, an estimated 228 million cases of malaria occurred globally and most cases and deaths (93%) were reported
from African Region. In the South East Asia Region, India accounts for the majority of cases and death.
• Usage of DDT: Spraying of Indoor Residual Spraying (IRS) in houses is an effective mosquito control tool. The
World Health Organisation (WHO) recommends DDT as one of the efficient IRS chemicals to curb malaria and
it is widely used by Southern African countries like South Africa, Zimbabwe, Zambia, Namibia, Mozambique
etc. and India.
About HIL:
It was incorporated in 1954 to manufacture and supply DDT to the Government of India’s Ministry of Health and
Family Welfare for malaria control programme. In the year 2019-20, the product was supplied to 20 States in the
country.
Chairman and Managing Director (CMD)– Dr. SP Mohanty
Headquarter– New Delhi

Report Errors in the PDF - ebooks@affairscloud.com Copyright 2014-2020 @ AffairsCloud.Com 61


Banking & Economy PDF
About South Africa:
Capitals– Cape Town, Pretoria, Bloemfontein
Currency– South African rand
President– Matamela Cyril Ramaphosa

Prakash Chandra Kandpal Appointed as MD & CEO of SBI General Insurance


Non-life insurer SBI General Insurance Company Limited announced that it has appointed Prakash Chandra(P C)
Kandpal as its Managing Director(MD) and Chief Executive Officer(CEO), effective from July 20. He succeeds Pushan
Mahapatra, who has been appointed as a director (strategic investments & digital initiatives), effective from 18th
July, 2020.
• Pushan Mahapatra will also continue to be the whole time director of the insurance company.
About Prakash Chandra Kandpal
i.He started his career as a Probationary Officer at State Bank of India(SBI) in 1987.
ii.Prior to this, he was serving as deputy CEO of SBI General Insurance Company Limited from June 6, 2019 till July
18, 2020.
iii.He also served as Chief General Manager at SBI and chief operating officer(COO) of SBI Mutual Fund.
iv.He has more than 33 years of experience with SBI i.e., has a wide experience in areas like HR, Retail and
Subsidiaries of SBI.
About Pushan Mahapatra
i.He joined the Company as the General Manager & Chief Operating Officer in July 2014.
ii.Prior to that, he worked as an Officer at SBI since 1985, he worked in areas of retail banking operations, IT, Risk and
Compliance.
About SBI General Insurance Company Limited
Headquarters– Mumbai, Maharashtra
Tagline– ‘Suraksha aur Bharosa Dono’
Recent Related News:
i.The Finance Ministry appointed PR Jaishankar as MD of the India Infrastructure Finance Company Limited (IIFCL)
for 3 years.
ii.The Board of Unit Trust of India(UTI )Mutual Funds Appointed Imtaiyazur Rahman as CEO of the UTI Asset
Management Company (AMC)

Airtel Payments Bank Partnered with NSDC to Skill Rural Youth for Financial Services Sector Jobs
Airtel Payments Bank Ltd has partnered with National Skill Development Corporation (NSDC) to train and skill
the youth in rural India.This will enable them to find employment opportunities and become entrepreneurs in the
financial services sector.
• The knowledge and training imparted to youth is to render financial and banking services more accessible to
rural and semi-urban India.
Highlights of the partnership
Leverage-The partnership will leverage NSDC’s experience, training infrastructure and wide network with Airtel
Payment Bank’s industry insights.
Skill development programmes-This leverage will enable them to design and conduct targeted skill development
programmes for rural youth, which will impart knowledge and skills for entry-level jobs, such as business
correspondent and field sales executive.
Focus of the programme- The skill development programmes will focus on adoption of tools to improve online
banking as well as digital financial services, given the growing penetration of affordable smartphones and 4G
networks in the country due to stay-at-home restrictions.
Conduct awareness campaign using NSDC’s training centres- Airtel Payments Bank will also use NSDC’s training
centres to conduct awareness campaigns on the importance of digital banking and its benefits.
Extend to e-Skill India- This partnership will also extend to e-Skill India, NSDC”s digital skilling initiative.

Report Errors in the PDF - ebooks@affairscloud.com Copyright 2014-2020 @ AffairsCloud.Com 62


Banking & Economy PDF
Special Learning Modules– Special learning modules will be built and introduced on the platform to raise
awareness about digital financial services.
Key Info
i.After the completion of the entry-level jobs such as business correspondent, field sales executive, among others, the
Airtel Payments Bank and NSDC will leverage their industry connections to help participants find jobs.
ii.They will also encourage trainees to take on entrepreneurial roles like, distributors or resellers of financial services.
This will create more jobs in their local geographies.
Note– India has the largest youth population in the world and a large proportion is residing in small towns and
villages of the country.
About Airtel Payments Bank:
Headquarters– New Delhi, India
Managing Director(MD) and Chief Executive Officer(CEO)– Anubrata Biswas
Non Executive Chairman– Sunil Bharti Mittal
About NSDC:
Headquarters– New Delhi, India
Chairman– Mr. A M Naik
MD & CEO- Manish Kumar
Recent Related News:
i.Airtel Payments Bank joined hands with Mastercard to develop a customised payment solutions for farmers, SMEs
ii.Airtel Payments Bank joined hands with Bharti AXA to offer COVID-19 insurance plans namely “Bharti AXA Group
Health Assure and “Group Hospital Cash” .

Bank of Baroda Launched ‘Insta Click Savings Account’, 100% Paperless Digital Self-Assisted Online
Savings Account
On July 23, 2020 India’s third-largest public sector bank, Bank of Baroda(BoB) launched ‘Insta Click Savings
Account’, a 100% paperless digital self-assisted online savings account. It can be activated in real-time i.e; the
customer can transact using the Baroda M Connect Plus application with the MPIN received on the mobile number.
• It also offers an option to subscribe to a wide range of digital channels like mobile banking, Internet banking,
UPI (unified payments interface), and debit card.
Features of ‘Insta Click Savings Account’:
Authentication of the customer– It uses a new form of digital KYC (know-your-customer) and Aadhaar-based
OTP(one-time password) authentication of the customer which can be operated from the bank’s website.
Instant Account Opening- The account can be opened instantly at any convenient time through a browser which is
accessible on Mobile,Tablet among others.
No Minimum Balance– There is no requirement to maintain a minimum balance.
Documents Required- The documents required to open the account is Aadhar no with registered mobile number,
Permanent Account Number(PAN)Card, E mail ID and Valid Mobile No.
Cash withdrawal– Cash withdrawal can be done using Card less withdrawal facility.
Account Funding- The account can be funded using Virtual ID/UPI ID or IMPS/NEFT.
Maximum balance and cumulative credit/deposit– The maximum permissible balance is Rs 1 Lakh, on any day.
The Maximum cumulative credit/deposit is Rs 2 Lakhs in a Financial Year.
Note– BoB is working on digitizing all its products and expects to be 100% paperless by 2023.
About BoB:
Headquarters- Vadodara, Gujarat
MD and CEO– Sanjiv Chadha
Non-Executive Chairman– Hasmukh Adhia
Recent Related News:
i.Airtel Payments Bank launches ‘Suraksha Salary Account’ for the micro, small and medium enterprises (MSMEs).
ii.IndusInd Bank rolls out India’s 1st mobile app based facility for opening current accounts through its ‘Indus
Corporate’ mobile app.

Report Errors in the PDF - ebooks@affairscloud.com Copyright 2014-2020 @ AffairsCloud.Com 63


Banking & Economy PDF
India’s GDP to contract by 6% in FY21: DBS
As per the report of DBS(Formerly, Development Bank of Singapore), Singaporean brokerage, India’s Gross domestic
Product(GDP) will contract by 6%(-6%) in FY21 as the economy is yet to stabilise the COVID -19 infection curve and
due to the caseload in the economically key states. This will affect the economy badly. In its earlier forecast, the
brokerage had forecasted the growth as -4.8%.
Key Projections of the report
Quarter
The infection curve, which is yet to stabilised reflects a deep double-digit contraction in Q1 and a flat pickup in
Q2 and a return to growth
Granular Analysis
i.As per the granular analysis, about 7% of the districts account for 70% of the caseload in economically key states
like Maharashtra (14% of national GDP), Tamil Nadu (8.5% of national GDP), Gujarat (8% of the national GDP),
accounting to 30.5% of the national economic output.
ii.Karnataka and Andhra also contribute significantly to the national economic output and accounts for about 70% of
the caseload.
Disruption in the renewed supply chain and uncertainty for manufacturers
There will be a disruption in the renewed supply chain and uncertainty for manufacturers, including automakers and
electronics firms if the domestic flights are restricted by more states.
Rural demand and farm output
Rural demand and farm output, which are projected to be 2% in FY21 will act as a counter-balancing buffer for a
decline in non-farm output
Meaning of Granular Analysis– A data that is in pieces, as small as possible, in order to be more defined and
detailed.
About DBS Group:
Headquarters– Singapore
CEO– Piyush Gupta
Non-Executive Chairman (DBS Group Holdings & DBS Bank)- Peter Seah
Recent Related News:
i.The latest report by CRISIL predicted that the gross domestic product (GDP) of Indian economy will contract by 5%
in fiscal 2021.
ii.As per the United Nations (UN) Economic and Social Commission for Asia and the Pacific (ESCAP) report India’s
FY21 GDP projected at 4.8% and recommended Asia-Pacific needs $880 mn/year health emergency fund amid
COVID19.

CBDT launched revised Form 26AS; Details of Specified Financial Transactions included
In order to enhance the flow of information between taxpayers and tax authorities, the Central Board of Direct Taxes
(CBDT) has launched a revised auto-generated Form 26AS namely “Annual Information Statement (AIS)” from this
assessment year of 2020-2021 w.e.f. June 1. The key difference between the new and old form is that the former will
also reflect the details of “specified financial transactions (SFT)” i.e. all high-value transactions of individuals,
companies, partnership firms, etc. which are filed with the tax department under Section 285BA of the Income-tax
Act, 1961.
• The SFTs are included in the Part E of the improved Form 26AS.
• Form 26AS is a consolidated annual tax statement that includes information on tax deducted/collected at
source (TDS), advance tax, other taxes paid, refunds, and self-assessment that is available on the Income Tax
website against a taxpayer’s Permanent Account Number (PAN).
• The new form will enable the taxpayer to have all important information available while preparing the tax
return so that the chances of committing any error will reduce significantly.
Point to be noted:
– As per the new format of Form 26AS, the tax department will update its contents within three months of receiving
the information from third parties like banks, Registrar of Properties, mutual funds, etc.

Report Errors in the PDF - ebooks@affairscloud.com Copyright 2014-2020 @ AffairsCloud.Com 64


Banking & Economy PDF
–For Knowledge: The details recorded in SFTs are provided by banks and financial institutions.
– Form 26AS earlier had an Annual Information Report (AIR) but now it has AIS.
What are SFT Transactions?
There are around 16 SFT transactions which have been introduced in Form 26AS include:
• Cash deposit to saving bank accounts, purchase of shares, debentures, mutual funds, buyback of shares worth
more than RS 10 lakh on an aggregate basis during a financial year
• Sale or purchase of immovable property worth more than Rs 30 lakh.
• Credit card payments of Rs1 lakh or more in cash or in any mode of Rs 10 lakh or more during a financial year.
Background:
The Budget for 2020-21 had announced the revised Form 26AS giving a more comprehensive profile of the taxpayer
going beyond the details of tax collected and deducted at source.
About Central Board of Direct Taxes (CBDT):
CBDT, a part of the Department of Revenue in the Ministry of Finance works as the policy making body for the Income
Tax department.
Chairperson– Pramod Chandra Mody
Headquarter– New Delhi
Recent Related News:
Union Finance &Corporate Affairs Minister Nirmala Sitharaman publicized necessitated relief measures for statutory
and regulatory compliance matters related to Income Tax, GST, Customs & Central Excise, Corporate Affairs,
Insolvency & Bankruptcy Code (IBC) Fisheries, Banking Sector and Commerce. In this regard, 3 month Extensions
(From 31st March, 2020 to 30th June, 2020) on Filing financial year 2018-19 (assessment year 2019-20) return was
provided.

Mukesh Ambani, fifth-richest Man in the World with Net Worth $75 billion: Forbes Real-Time
Billionaires List
Reliance Industries Limited (RIL) Chairman & CEO Mukesh Ambani (63), became the world’s fifth-richest man in the
world with an estimated wealth of 75 billion USD, according to the real-time ranking of billionaires by Forbes.
• Mukesh Ambani is just behind Mark Zuckerberg net worth of 88.3 billion USD.
• He sustains as the wealthiest Indian
• He is the only Asian Tycoon in the elite list of the world’s top 10 richest people as of July 2020.
Forbes Real-Time Billionaires List; Top three:
Rank Name Source Wealth
1 Jeff Bezos Amazon Founder & CEO 183.6 billion USD
2 Bill Gates Microsoft Co-founder 113.8 billion USD
3 Bernard Arnault LVMH Moet Hennessy- Louis Vuitton SE 112.4 billion USD
Note – The list will be updated regularly on every trading day.The above mentioned above is written based on July 22,
2020.
Recent Related News:
1. Mukesh Ambani became 9th richest in the world: Bloomberg Billionaires Index.
2. RIL Becomes India’s 1st Firm to Hit Rs 11 Tn Market Cap on June 11, 2020 after Valuing at Rs 11.15 tn; Mukesh
Ambani became world’s 9th richest.

Ravi Shankar Prasad Represented India in the G20 Digital Economy Ministers Virtual Meeting
Hosted by Saudi Arabia
On 22nd July 2020, Union Minister of Electronics and Information Technology, Shri Ravi Shankar Prasad represented
India in the virtual meeting of the G20 Digital Economy Minister under the presidency Saudi Arabia.
Key Points:
i.Ravi Shankar Prasad highlighted the need to build a strong global supply chain and the vision of Modi to make India
a destination for investments integrated with the global supply chain.

Report Errors in the PDF - ebooks@affairscloud.com Copyright 2014-2020 @ AffairsCloud.Com 65


Banking & Economy PDF
ii.He mentioned the importance of digital technologies like Direct Benefit Transfers and digital payments which
supported the government in providing relief to the economically weaker section of the society amid global pandemic
crisis.
iii.He focused on the need for building trustworthy Artificial intelligence systems for growth and development in
healthcare and education to support the society to transform.
iv.He discussed the issues related to data and sovereign rights of the countries in protecting the privacy and security
of its citizens.
v.He stated the importance of acknowledging the digital platform and to take responsive and accountable towards the
concerns of the countries defence and citizens privacy and security.
vi.He stated that India will engage a robust personal data protection law to address the concerns of the citizens
related to data privacy and economic development.
About G20 or Group of Twenty:
Members– Argentina, Australia, Brazil, Canada, China, France, Germany, India, Indonesia, Italy, Japan, Mexico,
Republic of Korea, Russia, Saudi Arabia, South Africa, Turkey, United Kingdom (UK), United States (US), and European
Union (EU).
Spain is a permanent guest invitee to the G20 meetings.
Chairman– Saudi Arabian King Salman bin Abdulaziz Al Saud
Current Presidency– Saudi Arabia
Recent Related News:
i.Union Minister of Petroleum and Natural Gas and Steel Dharmendra Pradhan has attended the G20 Extraordinary
Energy Ministers’ virtual meeting 2020 on behalf of India through video conferencing.
ii.Minister of Health and Family Welfare Dr. Harsh Vardhan participated in the G20 Health Ministers Meeting held via
video conference to control COVID-19.

Rs 8767 crore Approved for 81 projects under Special Window for Affordable and Mid Income
Housing Investment Fund I
Union Minister for Finance & Corporate Affairs, Nirmala Sitharaman informed that so far Rs 8767 crore has been
approved for 81 projects under Special Window for Affordable and Mid Income Housing (SWAMIH) Investment
Fund I. The approved projects will enable the completion of almost 60,000 homes across India.
• The announcement was made when the performance of SWAMIH was reviewed with Secretaries of Ministries
of Finance and senior management team of the State Bank of India, SBI Capital Markets Limited and SBICAPS
Ventures Limited (SVL)
Key Info
The Finance Minister asked the Department of Economic Affairs to closely monitor the performance of the SWAMIH
Investment Fund I so as to ensure that the capital raised by the Fund is committed towards resolving stressed
projects and remove any hindrance that arises in this process.
About SWAMIH investment fund I
i.It is a Special Window for Completion of Construction of Affordable and Mid-Income Housing Projects.
ii.The Union Cabinet has cleared a proposal in November 2019 to set up a ‘Special Window’ in the form of an
alternative investment fund (AIF) to provide priority debt financing for the completion of stalled housing projects.
SVL will be the Investment Manager to the first AIF set-up under this special window.
iii.The fund will be set up as a Category-II AIF (Alternate Investment Fund) debt fund registered with SEBI and would
be professionally run.
Highlights of the project
i.The approved projects include 26 projects in NCR, 27 projects in Mumbai, 10 projects in Bengaluru, 7 projects in
Pune and 3 projects in Chennai. The remaining are spread across Tier 2 locations.
ii.Among these projects, the investments in 18 projects have been given final clearance and disbursement is at
various stages across 7 residential projects. Applications from 353 stressed projects are under examination for
provision of assistance.

Report Errors in the PDF - ebooks@affairscloud.com Copyright 2014-2020 @ AffairsCloud.Com 66


Banking & Economy PDF
iii.Employment opportunities for various skilled and semi-skilled labourers will be provided on the activation of
these construction sites by the Special Window.
iv.The fund is also evaluating options to provide relief to approximately 15,000 home buyers in certain long-
discontinued projects.
Recent Related News:
i.Government will launch a scheme under Pradhan Mantri Awas Yojana (PMAY) for migrant labour/urban poor to
provide ease of living at affordable rent by Converting government funded housing in the cities into Affordable Rental
Housing Complexes.
ii.Rs 6.16 lakh crore (trillion) has been approved so far under Pradhan Mantri Awas Yojana (Urban)– PMAY(U).

ADB and GCF join hands towards ‘green recovery’ on the impacts of COVID-19 disease
On July 22, 2020, The Asian Development Bank (ADB) and the Green Climate Fund (GCF) agreed to partner toward
the “Green Recovery”. This is to help hands for the people who are facing the harsh economic impact due to COVID-19
pandemic.
They agreed to strengthen collaboration on non-sovereign operations while ensuring GCF’s processes are as flexible
as possible to foster at-scale financing.
Significance:
i.This collaboration focuses on enhancing the steps to reduce the carbon emission and for climate resilient
development in Asia and the pacific.
ii.ABD has offered a support package of USD 20 billion for COVID-19.
Highlights:
i.In 2019, ADB committed more than USD 6.5 billion (Rs. 4,866 crores) in climate financing.
ii.Under Strategy 2030, ADB will ensure that 75% of its project will support climate change mitigation and adaptation
by 2030.
iii.The total climatic finance support from ABD will reach USD 80 billion (Rs.1497 crores) from 2019 to 2030.
iv.GCF has approved total funding of USD 473 million for 10 ADB projects, thus ADB stands as the fourth largest
accredited entity of GCF.
v.The CGF is the world’s largest dedicated fund helping developing countries to reduce their greenhouse emission
that affects the climatic change.
Recent Related News:
ADB forms 8-member panel to find solutions for speedy recovery of COVID-19 in Southeast Asia.
On June 10, 2020, Asian Development Bank (ADB), a regional development bank, constituted an 8-member high-level
panel of leading experts in economics, finance, and health, moderated by ADB Vice-President Ahmed M. Saeed, to help
ministers, central bank governors, and other senior officials identify solutions for Southeast Asian economies to tap
into to quickly recover after the novel coronavirus (COVID-19) pandemic.
Indian Economy to contract by 4% in 2020-21: ADB Forecast
In accordance with the Asian Development Outlook (ADO) by Asian Development Bank (ADB), for FY20-21,
Indian economy is projected to contract by 4% in fiscal year (FY) 2020, ending on 31 March 2021, before growing
5.0% in FY2021
About ADB
Headquarters– Ortigas Center, Mandaluyong, Metro Manila. Philippines
Membership – 68 countries
President – Masatsugu Asakawa
About GCF
Headquarters – Songdo International Business District, Yeonsu-gu, Incheon, South Korea
Executive Director – Mr. Yannick Glemarec

Report Errors in the PDF - ebooks@affairscloud.com Copyright 2014-2020 @ AffairsCloud.Com 67


Banking & Economy PDF
World’s largest 1st Global Fintech Fest held virtually; UPI AutoPay, RuPay commercial card & OCEN
protocol launched
The 1st edition of two days long (July 22-23, 2020) world’s largest Global Fintech Fest (GFF) was held virtually, which
was organized by Payments Council of India (PCI) and Fintech Convergence Council (FCC) at National Payments
Corporation of India (NPCI) under the theme “Fintech : With and Beyond COVID”. The aim of the fest was to bring
together the global FinTech and Banking, financial services and insurance (BFSI) sector.
• Notably, both organizers of the fest work under the aegis of Internet and Mobile Association of India (IAMAI).
• The event was hosted by 6Connex Inc., a provider of virtual event solutions.
• The Department of Economic Affairs – Ministry of Finance, Government of India and Reserve Bank of India
(RBI) were the Presenting Partners for this event.
• This event was also powered by Amazon Pay and brought to you by WhatsApp and Google Pay, it is also
supported by the World Bank & United Nations Capital Development Fund (UNCDF).
The fest saw various launched for BFSI sector which are as follows:
–NPCI launched UPI AutoPAy to ease online recurring payments
In order to ease online recurring payments in India, NPCI introduced a Unified Payments Interface (UPI) AutoPay
feature. It is a one-stop fintech payment solution that allows customers to create e-mandate through their UPI ID or
QR Scan. For transactions less than Rs. 2,000, customers have to authenticate their account through UPI PIN for one-
time and subsequent monthly payments would be debited automatically. For an amount greater than Rs. 2,000, a UPI
PIN authentication will be required for every mandate.
• Mandates can be set for one-time, daily, weekly, fortnightly, monthly, bi-monthly, quarterly, half yearly and
yearly as well.
• This feature can be used for multiple financial purposes such as utility payments, booking bus pass, train
tickets, paying DTH subscriptions among others.
• In January 2020, the Reserve Bank of India (RBI) allowed the NPCI to develop a recurring payments feature on
UPI through electronic mandates.
UPI in India:
-As per, India Digital Payments Report 2019, there were UPI facilitated transactions worth Rs. 18,36,000 crores in
2019, up 214% from 2018.
-UPI is also the fastest product to hit 1 billion transactions-a-month in 2019 since its inception in -August 2016.
–NPCI launched SBM EnKash RuPay Business Card for small business
NPCI launched RuPay commercial card named SBM EnKash RuPay Business Card in partnership with State Bank of
Mauritius (SBM) India, EnKash, YAP (API- Application Programing Interface provider) and RuPay for young
entrepreneurs and small business owners. The card will be disbursed through SBM touchpoints.
• The card is aimed to streamline business expenditure and finances by offering expense tracking, and also provides
a 30-day credit period on business purchases, bill payments, travel expenses, automated GST, rental payments,
among others.
• Users can also get access to EnKash’s Whatsapp- based expense management service to manage and control all
employee and business related expenses.
SBM Bank (India) Limited is the first bank to receive universal banking license from the Reserve Bank of India to
set up and operate as a Scheduled Commercial Bank under Wholly Owned Subsidiary (WOS) mode offering
banking services in India.
Click Here for Official Notification
–Nandan Nilekani launched open credit protocol network
At the Global Fintech Festival, 2020, founding architect of Aadhaar and co-founder and non-executive chairman of
Infosys Ltd, Nandan Nilekani launched a new credit protocol infrastructure called the ‘Open Credit Enablement
Network (OCEN) protocol, which has been developed by Indian Software Product Industry RoundTable (iSpirit), a
think-tank of the software industry.
• OCEN will act as a mediator between loan service providers (LSPs), usually fintechs and mainstream lenders,
including all large banks and NBFCs (Non-Banking Financial Companies) with marketplaces. It will
democratise credit in the country, helping small businesses and entrepreneurs get loans.

Report Errors in the PDF - ebooks@affairscloud.com Copyright 2014-2020 @ AffairsCloud.Com 68


Banking & Economy PDF
• iSpirit is partnering with key lenders such as State Bank of India, HDFC Bank Ltd., ICICI Bank Ltd., IDFC First
Bank Ltd., Axis Bank Ltd. and Bajaj Finserv Ltd. for this new credit rail.
• This launch is on the lines of the recommendation of former SEBI Chairman Upendra Kumar Sinha Micro, Small
& Medium Enterprises (MSME) in a report suggested new technological interventions for MSME lending
–SEBI formed committee for financial and regulatory technology
The market regulator Securities and Exchange Board of India (Sebi) has formed a high level committee to look at
financial and regulatory technology at Global Fintech Summit. This committee will examine the aspects of technology
and unstructured data analysis for surveillance functions and market development.
• The committee will also make recommendations on the adoption of technology for markets as well as for SEBI.
About Internet and Mobile Association of India (IAMAI):
President– Dr. Subho Ray
Headquarter– Mumbai, Maharashtra
About National Payments Corporation of India (NPCI):
Managing Director and Chief Executive Officer (CEO)– Dilip Asbe
Headquarter– Mumbai, Maharashtra
Recent Related News:
On May 27, 2020 National Payment Corporation of India(NPCI) has launched an artificial intelligence(AI) based
chatbot, PAi, to create awareness around its products like FASTag, RuPay, Unified Payments Interface(UPI), Aadhaar
Enabled Payment System(AePS) on a real time basis & to improve digital financial inclusion in India. It is developed
by Bengaluru based startup CoRover Private Limited.

RBI Releases Report of Committee For Analysis of QR Code headed by Prof. Deepak B Phatak
On July 22, 2020, the Reserve Bank of India (RBI) publicized the “Report of the Committee for Analysis of QR (Quick
Response) Code” under the chairmanship of Prof. Deepak B. Phatak on its website for comments / suggestions of
various industry players currently involved in or associated with the use of QR codes before August 10, 2020. The
report has recommended making QR codes more interoperable. QR code is used to pay utility bills, fuel, grocery, food,
travel, and several other categories.
Before heading towards the recommendations of the report let’s have a look at QR in Indian scenario.
Usage of QR in India:
In India, the factors of post demonetization, increased penetration of smartphones and high-speed internet
connectivity led to the increased adoption of QR codes for digital payments. There are three different types of QR
Codes in India viz.
• Bharat QR– It is the world’s first fully interoperable QR code payment system. Its specifications have been
jointly developed by RuPay, Visa, MasterCard, and Amex.
• Unified Payments Interface (UPI) QR– It was launched by the National Payments Corporation of India (NPCI)
in August 2016.
• Proprietary QR (Closed loop) – It is provided by RBI authorized companies as Prepaid Payment Instruments
(PPI).
For Knowledge: Closed loop mobile payments are those that enable consumers to load money into a spending
account that is linked to a payment device while open platforms are connected to a personal account. For example
credit card which doesn’t require a prepaid amount to exist in the system that needs to be topped up when the money
runs out.
Key recommendations of Report:
-The committee has recommended phasing out of proprietary, closed loop QR codes as it requires customers to
manage separate apps and is creating hindrance in this payment system. Instead RBI must encourage the use of
interoperable QR codes such as the Bharat QR and UPI QR to enable faster on-boarding of all types of merchants for
digital payments.
-Government / RBI should allow a controlled interchange instead of zero Merchant Discount Rate (MDR) on QR code
/ UPI / RuPay Debit card transactions,
-Tax incentives should be provided to merchants who accept payments through electronic mode.

Report Errors in the PDF - ebooks@affairscloud.com Copyright 2014-2020 @ AffairsCloud.Com 69


Banking & Economy PDF
-For consumers also, the government should provide incentive schemes to ensure popularity of QR code transactions.
-All bank and non-bank applications must go through a process of standardisation to offer seamless experience to
customers.
-Small value merchants may be allowed to access the Bharat QR ecosystem to allow them more payments acceptance
options.
-Non-banks and fintechs could be allowed to participate in the Bharat QR ecosystem to increase penetration.
Background:
The Committee was constituted by RBI on December 23, 2019 under the Chairmanship of Prof. D.B. Phatak (Professor
Emeritus, IIT-Bombay) to review the prevalent system of QR Codes in India for facilitating digital payments and
submit recommendations. Its members were:
• Arvind Kumar – Director General (DG), Standardisation Testing and Quality Certification (STQC), Ministry of
Electronics and Information Technology
• Sunil Mehta– Chief Executive, Indian Banks’ Association (IBA)
• AS Ramasastri– Director, Institute for Development of Research in Banking Technology
• Dilip Asbe– Managing Director and Chief Executive Officer (CEO), National Payments Corporation of India
(NPCI)
• Vishwas Patel– Chairman, Payments Council of India (PCI)
Click Here for Official Link
About Reserve Bank of India (RBI):
Headquarters– Mumbai, Maharashtra
Formation– 1 April 1935
Governor– Shaktikanta Das
Deputy Governors– 4 (Bibhu Prasad Kanungo, Mahesh Kumar Jain, Michael Debabrata Patra, one is yet to be
appointed).
Recent Related News:
On June 13, 2020, RBI made some changes to the monitoring structure of Financial Markets Infrastructure
(FMI) and Retail Payment Systems (RPSs) with the aim to ensure the security and stability of the payment structure.
The new framework treats NPCI (National Payments Corporation of India) and National Electronic Fund Transfer
(NEFT) as FMIs.

Amazon Pay partnered with Acko General Insurance Ltd to offer auto insurance in India
Amazon Pay, the payments arm of Amazon India has partnered with Acko General Insurance Ltd to offer auto
insurance for two and four-wheelers in India, a 100% paperless insurance plan. Customers can avail this auto
insurance from Amazon Pay page, Amazon app or mobile website and can get a quote for their car or bike’s insurance
by providing basic details.
About auto insurance
Quick Paperless insurance plan– The customers can get claims with one-hour pick-up, three-day assured claim
servicing, and one-year repair warranty in select cities. It can be bought within 2 minutes.
Instant cash settlements– The policyholders will have an option for instant cash settlements for low-value claims
Select from a list of add-ons– The customers can select from a list of add-ons like zero-depreciation and engine
protection.
Additional benefit for amazon prime members– Amazon Prime members will get extra benefits, including
additional discounts on premiums on its insurance category.
Payment of insurance– The customers can pay for the insurance using Amazon Pay balance, UPI, or any saved card.
A copy of the policy can also be downloaded from customers’ Orders page.
Key Info– Amazon partnered with digital lending startup Capital Float and Karur Vysya Bank and launched its instant
credit line service, ‘Amazon Pay Later’ in the end of April 2020
Note– Amazon India announced that it has added 10 new warehouses, and expanded seven existing buildings.
Recent Related News:
i.Flipkart ties up with Bajaj Allianz General Insurance for digital motor insurance policy

Report Errors in the PDF - ebooks@affairscloud.com Copyright 2014-2020 @ AffairsCloud.Com 70


Banking & Economy PDF
ii.Edelweiss General Insurance launched ‘Edelweiss SWITCH’, an app-based motor insurance own damage (OD)
floater policy under IRDAI’s sandbox regulations.
About Amazon:
Headquarters– Washington, United States
Founder & CEO– Jeff Bezos
About Acko General Insurance Ltd
Headquarters– Mumbai, Maharashtra
MD & CEO– Varun Dua

ICICI Lombard in partnership with PhonePe launched Hospital Daily cash benefit
ICICI Lombard in partnership with PhonePe has launched Hospital Daily cash benefit under its Group Safeguard
Insurance policy, a sachet-based group insurance product.
It is a customized hospitalisation policy that allows PhonePe users to get an assured amount if they are hospitalised
due to injury or illnesses(including COVID-19).
i.Claim
• The hospital bills are not required for the claim, a discharge certificate will be sufficient in most cases for the
claim processing.
• To be eligible for the claim, a minimum of 48 hours hospitalisation is required along with a deductible of one
day for each claim.
• Customers who already have mediclaim policies, employer health insurance, self-funded means of paying
hospital bills will also be eligible for the claim payout benefit.
ii.Cover
Through this cash benefit, customers can choose a cover from Rs 500 per day up to Rs 5,000 per day at an annual
premium starting at Rs 130 including taxes, effectively costing Rs 0.35 per day.
iii.Policy can be bought through the app– The users of PhonePe can buy the policy through the app, from the “My
Money” section The policy documents can be viewed by the customers instantly on the app.
iv.Hospitals covered & double insurance cover– All government and private hospitals come under the insurance
cover. The patients who are admitted in the ICU will be qualified for double the insurance cover.
Recent Related News:
i.PhonePe Partnered with ICICI Bank on Unified Payments Interface(UPI) multi-bank model.
ii.PhonePe along with ICICI Lombard launches ‘first of its kind’ domestic trip travel insurance.
About PhonePe:
HeadQuarter– Bengaluru, Karnataka
Chief Executive Officer(CEO)– Sameer Nigam
About ICICI Lombard:
HeadQuarter– Mumbai, Maharashtra
Managing director(MD) & CEO– Bhargav Dasgupta

Mr. Partha Pratim Sengupta appointed as MD and CEO of IOB


i.On July 24, 2020 Indian Overseas Bank (IOB) has appointed Mr. Partha Pratim Sengupta as its new Managing
Director (MD) and Chief Executive Officer (CEO), and his tenure ends by December 31, 2022 or until further orders.
ii.Mr. Partha Pratim Sengupta succeeds Shri. Karnam Sekar who was retired on June 30, 2020.
Facts about Mr. Partha Pratim Sengupta:
i.Sengupta was the deputy managing director of State Bank of India (SBI)
ii.Prior to Deputy MD of SBI, he was the chief General manager at the country’s largest lender’s Kolkata circle.
iii.He has more than three decades of experience in the Banking sector, in various domains.
Recent Related news:
Reserve Bank approves Lakshmi Vilas Bank’s MD and CEO, Subramanian Sundar extension
On June 1, 2020, The Reserve Bank of India (RBI) approved the extension of tenure for Lakshmi Vilas Bank (LVB)
Managing Director (MD) and Chief Executive Officer (CEO), Subramanian Sundar by 6 months from May 31, 2020 to

Report Errors in the PDF - ebooks@affairscloud.com Copyright 2014-2020 @ AffairsCloud.Com 71


Banking & Economy PDF
November 30, 2020.
About IOB
Founded – 10 February 1937
Founder – Mr. M.Ct.M Chidambaram Chettyar
Headquaders – Chennai, Tamil Nadu
Chairman -Mr. T. C. A Ranganathan (Non-Exe Chairman)

Punjab Cabinet Approved a USD 285.71 million project for canal-based water supply
The Punjab Cabinet chaired by Chief Minister Amarinder Singh approved a USD 285.71 million(over Rs 2,130
crore) project for canal-based water supply for Amritsar and Ludhiana cities through video conferencing. The
funding of these projects will be offered by the World Bank(70%) and Punjab gov(30%). In addition to this, the
cabinet also approved an amendment to the Land Pooling Policy.
About USD 285.71 million project
The composition of the financing
The International Bank for Reconstruction and Development (IBRD) would finance USD 200 million (70%) and the
Punjab government USD 85.71 million (30%).
Components of the project
The component of the project includes the following:
USD 11.61 million to strengthen urban water supply service management; USD 240.38 million to improve water
supply infrastructure; USD 15.62 million for land acquisition and rehabilitation; USD 10 million for COVID crisis
response; USD 7.6 million for project management
Background
The Punjab government in June 2018 through the Department of Economic Affairs (DEA), Government of India
requested the World Bank to support Punjab for this purpose.
Amendment to the Land Pooling Policy
The amendment to the Land Pooling Policy is made for the residential Sector and its extension to the Industrial Sector
in the jurisdiction of the Greater Mohali Area Development Authority (GMADA).
As per the amended policy:
Residential and Commercial Plots
1000 square yards of developed residential plots and 200 square yards of the commercial plot, excluding parking, will
be given in lieu of cash compensation for every one acre of land to be acquired from landowners for the upcoming
Aerotropolis Residential Estate.
Sahuliyat Certificate
i.The validity of the ‘Sahuliyat Certificate’ will be given to the landowners opting for land pooling policy.
ii.They will be provided with certain benefits during the purchase of equivalent value of land and will be counted
from the date of allotment of plot to the landowner.
iii.The certificate holders will be entitled to get an exemption from Stamp Duty among others.
Recent Related News:
i.Odisha cabinet approved the proposal to give the status of State anthem to “Bande Utkala Janani”, a poem written by
Kantakabi Laxmikanta Mohapatra in 1912.
ii.Madhya Pradesh Cabinet approved to form trust for ‘Ram Van Gaman Path’ project.
About Punjab:
Capital– Chandigarh
Governor– Vijayender Pal(V.P.) Singh Badnore

DST launched Rs 15 Crore fund to support India-Russia collaboration joint R&D and cross-country
technology adaptation
On July 24, 2020, The Department of Science and Technology (DST) under the Ministry of Science and Technology
launched India-Russia Joint Technology Assessment and Accelerated Commercialization Programme in partnership
with Russia’s Foundation for Assistance to Small Innovative Enterprises (FASIE). On the Indian side, the

Report Errors in the PDF - ebooks@affairscloud.com Copyright 2014-2020 @ AffairsCloud.Com 72


Banking & Economy PDF
programme will be implemented & funded by Federation of Indian Chambers of Commerce & Industry (FICCI) on
behalf of the DST.
• The objective of this bilateral initiative is to strengthen the Science, Technology and Innovation ties between
the two countries by connecting Indian, and Russian Science & Technology (S&T) led Small and medium-sized
enterprises (SMEs) and Start-ups for joint development through Research & Development (R&D) along with
technology adaptation.
Implementation of programme; DST to fund up to Rs 15 cr
The programme will run through two annual cycles and top five selected Indian and Russian partners will be
provided partial funding support. DST will fund up to Rs 15 Crores to Indian SMEs/Start-ups and FASIE will provide
similar funding to the Russian projects. The remaining cost will be borne by partners.
• In addition to the financial support, the teams will also be supported through capacity building, mentorship
and business development.
The programme is accepting applications under two broad categories, i.e. Joint Partnership Projects and Technology
Transfer/Adaptation. The last date to apply for the first round is September 30, 2020
at www.indiarussiainnovate.org.
Static Points:
Secretary Department of Science and Technology (DST)– Prof. Ashutosh Sharma
Indian Ambassador to Russia – D. Bala Venkatesh Varma
Foundation for Assistance to Small Innovative Enterprises (FASIE) General Director– Sergey Polyakov
About FICCI:
Secretary General– Dilip Chenoy
President– Sangita Reddy
Headquarter– New Delhi
Recent Related News:
Minister of Defence (Raksha Mantri) Rajnath Singh paid a 3-day visit to Russia from June 23- 25, 2020 on the
invitation of Defence Minister of the Russian Federation Sergey Shoigu to attend the 75th Anniversary of Victory Day
Parade and to strengthen the longstanding special and privileged strategic partnership between India and Russia.

India extended medical assistance worth about USD 1 million to North Korea under WHO’s anti-
tuberculosis programme
On July 24, 2020, on the request of the World Health Organisation (WHO), the Government of India (GoI) has
extended medical assistance in the form of anti-tuberculosis medicines worth about US$ 1 million to Democratic
People’s Republic of Korea (DPRK).
• It is to be noted that New Delhi’s medical assistance is under the aegis of an ongoing WHO’s anti-
tuberculosis programme in North Korea and hence exempted from the sanctions imposed by the United
Nations Security Council (UNSC).
The consignment of medicines was handed over by Indian Ambassador to DPRK, Atul Malhari Gotsurve to North
Korean Vice Minister for Foreign Affairs, Pak Myong Ho along with a floral basket and a congratulatory letter
addressed to the Supreme Leader, Kim Jong Un, on the occasion of the 8th anniversary of conferment of the title of
‘marshal” on him.
Indo-DPRK Relation:
-India had earlier provided North Korea two consignments of food worth $ 1 million each in 2011 and 2016 under the
World Food Programme (WFP).
-In May 2018, Vijay Kumar Singh, the then Minister of State for External Affairs (MEA), had embarked on a rare visit
to North Korea after two decades when Mukhtar Abbas Naqvi visited in September 1998. He was the Minister of State
(MoS) for Information and Broadcasting in Prime Minister Late Atal Bihari Vajpayee’s Government.
About Democratic People’s Republic of Korea (DPRK):
Capital– Pyongyang
Currency– North Korean won

Report Errors in the PDF - ebooks@affairscloud.com Copyright 2014-2020 @ AffairsCloud.Com 73


Banking & Economy PDF
Recent Related News:
On May 5, 2020, Russian President Vladimir Putin has awarded North Korean Leader Kim Jong-un a commemorative
war medal marking the 75th anniversary of the victory over Nazi Germany for recognizing his role in preserving the
memory of Soviet soldiers who died on North Korean territory.

India ranked 3rd in gaining forest area in world; global employment in forestry led by India: FAO
In accordance with the latest Global Forest Resources Assessment (FRA) 2020 brought out by the Food and
Agriculture Organization (FAO) of the United Nations (UN), India has ranked 3rd among the top 10 countries that
have gained in forest areas during 2010-2020. FRA 2020 examined the status of, and trends in, more than 60 forest-
related variables in 236 countries and territories in the period 1990–2020.
• India reported 0.38% annual gain in forest, or 266,000 ha of forest increase every year at an average. It’s
naturally regenerating forest rate was also 0.38% in 2010-20.
• As per the report, the world has a total forest area of 4.06 billion hectares (ha) in 2020, which is 31% of the
total land area where India accounts for 2% of total global forest area. Russia has a maximum forest area
accounting 20% of global forest area (815 312 ha) followed by Brazil (496 620 ha-12%) & Canada (346 928
ha- 9%).
Top ten countries for average annual net gain in forest area, 2010–2020
Rank Country
3 India
1 China
2 Australia
4 Chile
5 Viet Nam
6 Turkey
7 United States of America
8 France
9 Italy
10 Romania
Findings with respect to India:
-The FRA 2020 has credited the government’s Joint Forest Management programme for the significant increase in
community-managed forest areas in the Asian continent. India has been taking up massive afforestation and
plantation schemes.
-The forest area managed by local, tribal and indigenous communities in India increased from zero in 1990 to about
25 million ha in 2015.
India’s employment in forestry sector is maximum in world
The assessment examined employment in the forestry sector (including logging) with data from 136 countries that
represent 91% of the world’s forests where India reported the maximum employment in the forestry sector in the
world.
• Globally, 12.5 million people were employed in the forestry sector. Out of this, India accounted for 6.23 million,
or nearly 50%.
Key Points:
-The world has lost 178 million ha of forest since 1990, which is an area about the size of Libya. Africa had the largest
annual rate of net forest loss in 2010–2020, at 3.9 million ha.
–The South Asia sub-region reported net forest losses during 1990-2020.
-The Asian continent reported the highest net gain in forest area in 2010-2020 as it recorded 17 million hectares (ha)
per year net increase in forests.

Report Errors in the PDF - ebooks@affairscloud.com Copyright 2014-2020 @ AffairsCloud.Com 74


Banking & Economy PDF
About Food and Agriculture Organization (FAO):
Director-General (DG)– Qu Dongyu
Headquarter– Rome, Italy
Recent Related News:
In accordance with the Food and Agriculture Organization (FAO) report, released in March 2020, Peatlands are facing
degradation and need to be urgently monitored. Peatlands are wetlands with a thick layer of organic soil and cover
only 3% of the global land area, but they store 30% of the world’s soil carbon. In India, peatlands occupy 320–1,000
square kilometres area.

RBI Signed USD 400 mn Currency Swap Facility for Sri Lanka
Reserve Bank of India (RBI) has agreed to a USD 400 million currency swap facility for Sri Lanka, valid till
November 2022. This will help Sri Lanka to meet its short-term international liquidity requirements and to boost its
foreign exchange reserves following the economic crisis resulting from COVID-19.This Currency swap agreement was
made under the South Asian Association for Regional Cooperation (SAARC) framework.
Technical discussion
i.Recently, India and Sri Lanka held a “technical discussion” on the rescheduling of the bilateral debt repayment
through video conference.
ii.Key People: From India, senior officials from the Ministry of External Affairs, Ministry of Finance, and the EXIM
Bank interacted with representatives from the Department of External Resources of Sri Lanka.
iii.The next round of technical discussions between the two sides on the rescheduling of debt repayment is expected
to be held soon.
Note: Sri Lanka owes USD 960 million to India.
Webinar on ‘Deepening Economic Collaboration between India and Sri Lanka’
i.The government and industry representatives from India and Sri Lanka participated in a webinar on ‘Deepening
Economic Collaboration between India and Sri Lanka’.
ii.The webinar is organized by the Federation of Indian Chambers of Commerce and Industry (FICCI) in association
with the Lakshman Kadirgamar Institute of International Relations and Strategic Studies (LKIIRSS)
Key People- Secretary, Ministry of Foreign Affairs, Ambassador Ravinatha Aryasinha and the President of FICCI, Dr.
Sangita Reddy and representatives from the Sri Lankan Board of Investment, Tourism Promotion Board, and the
Institute of Nanotechnology among others participated in the webinar.
Relationship between India and Sri Lanka
i.India is among the top 5 foreign investors in Sri Lanka, and accounts for 20 % of total Sri Lankan imports. India is
the 3rd largest export destination for Sri Lanka
ii.Sri Lanka is one of India’s largest trading partners in SAARC. With India-Sri Lanka Free Trade Agreement in March
2000 in force the trade grew between two countries.
iii.Sri Lanka is among the major recipients of development assistance from the Government of India.
iv.India’s overall commitment to Sri Lanka is about USD 3 billion, out of which around USD 560 million are in grants.
Recent Related News:
i.India and Denmark signed an MOU for developing cooperation in the power sector.
ii.India and Maldives inks 5 MoUs to establish Addu tourism Zone on 5 islands of Addu Atoll in Maldives with a
project cost of $2.49 million. In addition, a 6th MoU to set up a bottled water plant at Hoarafushi, an inhabited island
in Maldives was also signed.
About Sri Lanka
Capital – Colombo, Sri Jayawardenepura Kotte
Currency– Sri Lankan Rupee
President– Gotabaya Rajapaksa
Prime Minister– Mahinda Rajapaksa

Report Errors in the PDF - ebooks@affairscloud.com Copyright 2014-2020 @ AffairsCloud.Com 75


Banking & Economy PDF
Gross NPA of banks may rise to 12.5% by March 2021: RBI Financial Stability Report
In accordance with the 21st issue of Financial Stability Report (FSR), July 2020 by Reserve Bank of India (RBI)
released on July 24, 2020, the gross nonperforming assets (GNPA) ratio of all scheduled commercial banks (SCBs)
may increase from 8.5% in March 2020 to 12.5% by March 2021 under the baseline scenario; the ratio may escalate
to 14.7% under a very severely stressed scenario.
• While Public sector banks’ (PSBs) gross bad loans ratio of 11.3% at the end of March, 2020 may increase to
15.2% by March 2021 under the baseline scenario, on the other hand, Private banks may see an increase from
4.2% to 7.3%.
Key Points:
–Capital to risk-weighted assets ratio (CRAR) of SCBs declined to 14.8% in March 2020 from 15% in September
2019. This ratio could slide to “13.3% in March 2021 under the baseline scenario and to 11.8% under the very severe
stress scenario.
–Gross non-performing asset (GNPA) ratio of SCBs declined to 8.5% from 9.3%.
–Provision coverage ratio (PCR) of SCBs improved to 65.4% from 61.6% over this period.
–Gross Domestic Product (GDP) will decline to 8.9% in 2020-21. In the baseline case, the GDP is assumed to
contract 4.4%.
–System-level capital adequacy ratio (ratio of capital to advances) may drop from 14.6% in March 2020 to 13.3%
by March 2021 under the baseline scenario and to 11.8% under the very severe stress scenario.
-Bank credit which had considerably weakened during the first half of 2019-20, slid down further to 5.9% by March
2020 to early June 2020.
-Five banks may fail to meet the minimum capital level by March 2021 in a very severe stress scenario.
-Large borrowers (with exposure of over Rs 5 crore) accounted for 51.3% and 78.3% of the -aggregate loan portfolio
and GNPAs, respectively, of SCBs in March 2020. Both these shares have declined since March 2018.
About FSR:
It reflects the collective assessment of the Sub-Committee of the Financial Stability and Development Council (FSDC)
on risks to financial stability, and issues relating to development and regulation of the financial sector.
About Reserve Bank of India (RBI):
Headquarters– Mumbai, Maharashtra
Formation– 1 April 1935
Governor– Shaktikanta Das
Deputy Governors– 4 (Bibhu Prasad Kanungo, Mahesh Kumar Jain, Michael Debabrata Patra, one is yet to be
appointed).
Recent Related News:
On June 8, 2020, after exercising the powers conferred by the Banking Regulation Act, 1949 and the Reserve Bank of
India (RBI) Act, 1934, RBI has proposed two draft documents titled “Draft Comprehensive Framework for Sale of
Loan Exposures” and “Draft Framework for Securitisation of Standard Assets” consisting of guidelines applicable to
all Scheduled Commercial Banks (excluding Regional Rural Banks); All India Financial Institutions; and, all Non-
Banking Financial Companies (NBFCs) including Housing Finance Companies (HFCs).

TN government signed 16 MoUs worth Rs 5,137 crore with companies for employment creation
The Tamil Nadu(TN) government has signed 16 Memorandum of Understanding (MoUs) worth Rs 5,137 crore to
create employment opportunities for over 6,555 people. The MoUs were signed by TN Chief Minister Edappadi K.
Palaniswami. The government has signed three tranches of MoUs worth Rs 30,664 crore within 2 months, which will
provide employment to over 67,212 persons.
• Out of 16 MoUs, 6 were signed directly and 10 through virtual link.
• 7 MoUs have been signed in the technology space. Of these, 6 will come up in Chennai and one in Coimbatore.
A new portal HYPERLINK Launched
The Chief Minister launched a new portal HYPERLINK(https://investingintamilnadu.com/) for the government’s
investment promotion agency.

Report Errors in the PDF - ebooks@affairscloud.com Copyright 2014-2020 @ AffairsCloud.Com 76


Banking & Economy PDF
Highlights
i.Adani Enterprises Limited to invest Rs 2,300 crore and provide 1,000 jobs by setting up a data centre at Siruseri
ii.Princeton Digital Group to invest Rs 750 crore and provide 100 jobs at Siruseri
iii.The Super Auto Forge, manufacturer of cold and warm forged steel and aluminium parts will set up a forged steel
and aluminium parts manufacturing facility at an investment of Rs 500 crore at SIPCOT Vadakal industrial park in
Kancheepuram district and provide about 500 jobs.
Key People:
Industries Minister M.C. Sampath, N. Muruganandam, Principal Secretary, Department of Industries and Neeraj Mittal
, MD and CEO, Guidance, Industries Department also participated in the occasion.
Other MoU’s signed by TN govt
i.MoUs for Rs 10,399 crore was signed with eight firms to bring jobs to over 13,507 people on July 20, 2020.
ii.MoUs worth Rs 15,128 crore were signed with 17 companies to provide employment opportunities to 47,150
people on May 27, 2020.
Recent Related News:
i.UP government signs MoU with industry associations to create 10 Lakh jobs for migrants.
ii.Maharashtra signs 12 MoU’s with domestic and foreign companies for FDI worth over Rs 16,000 crores. It is a part
of the Magnetic Maharashtra 2.0 initiative.
About Tamil Nadu:
Governor– Banwarilal Purohit
National Parks(NP)– Gulf of Mannar Marine NP, Guindy NP, Mudumalai NP, Mukurthi NP.

INDIA-UK’s 14th Joint Economic and Trade Committee meeting 2020 held virtually
On July 24, 2020, the 14th Joint Economic and Trade Committee (JETCO) meeting 2020 was held virtually for the first
time amid COVID-19 between India and United Kingdom (UK), which was co-chaired by Minister for Commerce and
Industry Piyush Vedprakash Goyal and UK Secretary of State for International Trade, Elizabeth Mary Truss. They
were assisted by Minister of State (MoS) for Commerce and Industry Hardeep Singh Puri and his counterpart, UK
Minister of State for International Trade, Ranil Jayawardena.
• Both nations have affirmed their shared commitment towards Free Trade Agreement (FTA). In this regard, the
two sides will engage in early harvest schemes or limited trade agreements in a staged manner as steps
towards FTA.
• It should be noted that the Early Harvest Programme basically consists of those projects which are to be
implemented within a short timeline.
Key Points:
-As the UK leaves European Union (EU), both sides are exploring the building blocks that would allow for more
ambitious trade arrangements in the future.
–Both sides have identified five key sectors: food and drink, information communications technology (ICT), life
sciences and services, and chemicals to address non-tariff barriers to trade.
-It was also decided that a deliberation led by Mr Goyal and Ms Truss will be held in Autumn 2020 in New Delhi to
carry forward the dialogue. Apart from this, Hardeep Singh Puri and Ranil Jayawardena will conduct monthly
meetings for the same.
-Both sides also resolved to cooperate in the health sector especially in view of the COVID-19 pandemic.
-The Co-chairs of Business led Joint Working Groups on Life Science and Health, ICT and Food and Drink set up during
the last JETCO made their recommendations to the Ministers.
About United Kingdom (UK):
Capital– London
Currency– Pound sterling
Prime Minister– Alexander Boris de Pfeffel Johnson
Recent Related News:
On June 25, 2020, According to the ‘Annual banking statistics, 2019’ released by Switzerland’s central bank Swiss
National Bank (SNB), India is at the 77th position in terms of money parked by its citizens and enterprises with Swiss

Report Errors in the PDF - ebooks@affairscloud.com Copyright 2014-2020 @ AffairsCloud.Com 77


Banking & Economy PDF
banks at the end of the year 2019,compared to 74th the previous year i.e 2018. United Kingdom (UK) holds the first
place in this list with a share of 27 % of the total deposits.

Bank of Maharashtra joined hands with Uniken for Omni-channel security


The state-owned Bank of Maharashtra has joined hands with Uniken, a pioneer in the field of digital security in India
to secure its Internet banking channels on mobile, card, call centres and ATMs, among others, against cyber fraud. In
this regard, the bank will use Uniken’s REL-ID technology for Omni-channel security for its online banking
app MahaSecure .
• MahaSecure has made mandatory for all its customers who perform Internet banking activities to ensure data
security and keeping cyber fraud at bay for their customers
• The app provides convenience, security and uniform experience across all devices, including desktops, laptops,
smartphones and tablets, through the app’s medium to deliver services.
The REL-ID technology by Uniken has been modelled for defence-in-depth functions that will help secure the bank’s
mobile apps by protecting them against phishing, pharming and malware attacks by delivering world class
transaction capability.
About Bank of Maharashtra:
Managing Director & Chief Executive Officer (CEO)– A S Rajeev
Headquarter– Pune, Maharashtra
Tagline– One Family One Bank (Ek Parivar Ek Bank)
About Uniken:
Chief Executive Officer (CEO)– Bimal Gandhi
Headquarter– Chatham, New Jersey, United States (US)
Recent Related News:
In June 2020, The Reserve Bank of India (RBI) raised the withdrawal limit from Punjab and Maharashtra Cooperative
(PMC) Bank to Rs 1 lakh from Rs 50,000 but extended the moratorium on the bank for another six months i.e. until
December 22, 2020.PMC is under All-Inclusive Directions under sub-section (1) of Section 35 A read with Section 56
of the Banking Regulation Act, 1949 with effect from September 23, 2019.

Income Tax Day 2020 – July 24


24th July 2020 is marked as the 160th Income Tax day or Aaykar Diwas by the Income Tax Department. The Income
Tax Day has been celebrated in India on 24th July since 2010.
In 2010, The Income Tax Department declared July 24th as the annual Income Tax Day to mark the 150th year of this
levy, the income tax was first imposed as a duty and came into force on 24th July 1860.
History:
Income tax was introduced in India by Sir James Wilson on 24th July 1860, to compensate for the losses of the British
government due to the First war of Independence in 1857
Events:
i.The regional offices of the Income Tax department conducts various activities in the week preceding the Income tax
day.
ii.The best performing Indian Revenue Service and other officials of the Income tax department will be awarded on
the income tax day.

Central Govt. with World Bank decides to convert ravines of Chambal region into arable land
On 26th July 2020, Narendra Singh Tomar, Union Minister of Agriculture and Farmers Welfare announced that the
Central Government in collaboration with the World Bank has decided to convert the area of ravines in the region of
Gwalior-Chambal belt of Madhya Pradesh into cultivable land.
The decision was made through a virtual meeting with Adarsh Kumar, representative of the World Bank.
Key People:
Vivek Aggarwal, Joint Secretary, K K SinghAgriculture Production Commissioner, S K Rao Vice-Chancellor of Rajmata
Vijayaraje Scindia Agricultural University.

Report Errors in the PDF - ebooks@affairscloud.com Copyright 2014-2020 @ AffairsCloud.Com 78


Banking & Economy PDF
About the Project:
i.A preliminary report for the conversion of ravines into arable land will be prepared in a month.
ii.Following the initial reports, the further actions will be discussed with the Chief Minister of Madhya Pradesh in the
subsequent meetings.
iii.The aspects related to the technology, infrastructure, capital cost, investment and other will be considered before
the initiation of the project with minimum budget allocation.
Benefits:
i.Around 3 lakh hectares land of this region are non-cultivable, this conversion will improve and support the
integrated development of the Bihad area in the region of Gwalior-Chambal.
ii.In addition to the agricultural and environmental development this project will also create employment
opportunities and will support the substantive development of this region.
iii.The overall development of this region will be made possible through the constitution of the Chambal Expressway
through this region which will increase the development scope of the Beehad area in the Gwalior-Chambal region.
About World Bank:
President– David Robert Malpass
Headquarters– Washington, D.C., United States
Motto– Working for a World Free of Poverty
About Ministry of Agriculture and Farmers Welfare:
Union Minister– Narendra Singh Tomar
Recent Related News:
i.The board of executives of the World Bank (WB) has approved loan of $ 500 million (about Rs 3,700 crore) under
‘Strengthening Teaching-Learning and Results for States Program (STARS)’project to improve the quality and
governance of school education across six states (Himachal Pradesh, Kerala, Madhya Pradesh, Maharashtra, Odisha,
and Rajasthan) in India.
ii.The Government of India and West Bengal Government have inked two loan agreements with Asian Infrastructure
Investment Bank(AIIB) and World Bank (WB) for the “West Bengal Major Irrigation and Flood Management Project”.

India’s GDP boosts to 2.5% by AI : Report by ICRIER, Google & NASSCOM


According to The report titled, ’Implications of AI on the Indian Economy’ by National Association of Software and
Services Companies (NASSCOM) along with the Indian Council for Research on International Economic
Relations (ICRIER) and Google, a unit increase(measured as the ratio of AI to total sales) in Artificial Intelligence(AI)
intensity can result in a 2.5% increase in India’s Gross Domestic Product(GDP) in the immediate term.
Key Info
• The report results find a positive and significant relation between AI using firms and TFP (total factor
productivity) growth.
• The business as usual growth in AI investments is unlikely to increase current levels of AI intensity
• To trigger a positive growth shock, AI intensities must be sharply increased.
HIghlights of the report
Investment of Rs 7,000 crore
i.The report noted that the Investment of Rs 7,000 crore, announced in the Union Budget towards AI program will
increase AI investments at rates higher than the business-as-usual-rates.
ii.The increase in investment will improve the AI intensity by approximately 1.3 times, resulting in benefits of $85.77
billion or increase in 3.2% of the GDP.
Recommended policy measures
The following are the recommended policy measures of the report to support AI’s wider adoption in India:
• Identify a nodal agency for the development and diffusion of AI.
• Build collaborative frameworks for engagement between governments, industry and academia.
• Build an all-encompassing data strategy for India, addressing India’s skill gap in AI and promoting the
development of AI safety standards among others

Report Errors in the PDF - ebooks@affairscloud.com Copyright 2014-2020 @ AffairsCloud.Com 79


Banking & Economy PDF
Key Facts of the report
i.As per estimates, Indian AI market for start-ups attracted USD 762.5 million in 2019. Even Though companies
recognise the benefits and potential of AI, the cost and the doubts on return on investments in AI are acting as a
barrier.
ii.Once AI goes mainstream its positive impact would become more noticeable in GDP
iii.The report recognises the role of the government to create institutions and provide public goods that enable an AI
ecosystem.
Note-The report has used investments in software, databases and computer machinery as a proxy for AI.
Recent Related News:
i.According to the RBI Governor Shaktikanta Das, India’s gross domestic product (GDP) growth will be in negative
territory in 2020-21 as the outbreak of coronavirus has disrupted economic activities. He also stated that the inflation
outlook is “highly uncertain”.
ii.The Federation of Indian Export Organisations(FIEO), apex body of exporters informed that Indian exports are
expected to fall by 20% in the current fiscal, in value terms it will be around USD 50 to 60 billion & also imports due
to COVID-19 pandemic.
About NASSCOM:
Headquarters– New Delhi, India
President– Debjani Gosh.
About ICRIER
Headquarters– New Delhi, India
Chairperson, Board of Governors – Isher Judge Ahluwalia
Director and Chief Executive– Rajat Kathuria

IRCTC and SBI Card launched a Co-branded Contactless Credit Card on RuPay Platform
On July 28, 2020 Indian Railway Catering and Tourism Corporation Limited(IRCTC) and SBI Card launched a co-
branded contactless credit card, IRCTC -SBI Platinum Card on RuPay Platform to reward the frequent
railway(train) travellers. The card is equipped with Near Field Communication (NFC) technology by which users can
facilitate their transactions at the point of sale(POS) machines by Tap and Pay Option. This enables consumers to
transact safely and securely online.
• The card offers exclusive benefits for rail passengers on retail, dining, entertainment besides transaction fee
waivers. It also offers maximum savings proposition on their travel
Key Benefits of the cardholders
i.When the cardholders make bookings on IRCTC website, they will receive up to 10% value back on 1st AC, 2nd AC,
3rd AC, Executive Chair Car & AC Chair Car bookings
ii.It offers online transaction fee waiver (1% of transaction amount), 1% fuel surcharge waiver and 4 premium lounge
free access at Railway stations in a year (one per quarter).
iii.Upon activation of the card the cardholders will receive 350 Bonus Reward Points.
iv.The accumulated reward points can be redeemed against purchase of train tickets on IRCTC’s ticketing website.
v.It also offers several benefits for online shopping portals. The customers can avail discounts when shopping on e-
commerce sites.
Recent Related News:
i.Yes Bank & Affordplan Jointly Launched Co-Branded Healthcare Card Called ‘Swasth Card’ under Swasth program
with the aim to enable families plan and manage their finances for their healthcare needs.
ii.SOLV partnered with Standard Chartered Bank to Launch Credit Card for MSMEs
About IRCTC:
Headquarters– New Delhi, India
Chairman & Managing Director– Mahendra Pratap Mall

Report Errors in the PDF - ebooks@affairscloud.com Copyright 2014-2020 @ AffairsCloud.Com 80


Banking & Economy PDF
About SBI Card:
Headquarters– Gurgaon, Haryana.
Managing Director(MD) and Chief executive Officer(CEO)– Hardayal Prasad(will retire on July 31, 2020) Ashwini
Kumar Tewari to take charge from August 1, 2020.

Bharti AXA General Insurance Launched Crop Insurance Campaign, ‘Bohot Zaroori Hai’ for Farmers
Bharti AXA General Insurance has launched its crop insurance campaign,’Bohot Zaroori Hai’ for farmers in
Maharashtra and Karnataka to encourage them about their yield protection and financial security. It aims to create
awareness to farming communities about the relevance and significance of crop insurance, which can alleviate rural
distress caused by crop failure or damage due to unseasonal rains, monsoon failure among others.
• Bharti AXA General Insurance had received Rs 800 crore worth crop insurance mandate from the governments
of Maharashtra and Karnataka to insure their farmers under the Pradhan Mantri Fasal Bima Yojana (PMFBY)
About the campaign
The awareness drive describes that buying crop insurance as a very important aspect of farming and also signifies
that the protection of crops will eventually secure the aspirations of farmers.
Rs 800 crore worth crop insurance mandate
i.Bharti AXA General Insurance has secured authorization for a period of 3 years from both the state governments to
implement the PMFBY in six districts of Maharashtra and three districts of Karnataka.
ii.The PMFBY offers insurance cover to farmers against losses of crops during the entire cycle from preparation of
sowing to harvesting and post-harvest, due to poor yield.
iii.PMFBY is open for both lenders and non lenders. The scheme covers food crops (cereals, millets and pulses),
oilseeds as well as horticultural crops.
iv.The farmers pay 2% of sum insured as the premium for Kharif crops and 1.5% of the sum insured for Rabi crops.
Recent Related News:
i.NITI Aayog Launched Behaviour Change Campaign, ‘Navigating the New Normal’,and its Website.
ii.Power Minister RK Singh initiated ‘#iCommit’ campaign on the occasion of World Environment Day (WED) on June
5, 2020.
About Bharti AXA General Insurance
Headquarters– Mumbai, Maharashtra
Chief Executive Officer & Managing Director– Sanjeev Srinivasan

SBI provided USD 16.20 million liquidity support for local businesses in Maldives as COVID-relief
On the occasion of Independence Day of the Maldives on July 26, 2020, the largest Indian public sector bank, State
Bank of India (SBI) provided liquidity support of USD 16.20 million as a COVID-relief for the Government of Maldives
to overcome liquidity shortage. The fund will support economic relief measures by utilizing it for local businesses and
defers loan repayment for over 200 retail accounts.
• It should be noted that the year 2020 marks the 55 years of establishment of diplomatic ties between India
and the Maldives.
Key Points:
-An Assistance of USD 400 million has been made available already to Maldives through an extended currency swap
arrangement which was inked in July 2019 by both nations. Notably, Maldives is the only country, other than Bhutan,
which has been extended with the USD 400 million currency swap facility.
-India will soon announce another substantial financial assistance package to support the Maldivian economy and
assist in economic recovery post-COVID-19.
-The growing relationship between both nations is premised upon the ‘India First’ policy of Maldives
and ‘Neighbourhood First’ policy of India.
Recent Related News:
On April 2, 2020, Indian Air Force (IAF) launched ‘Operation Sanjeevani’ and air-lifted 6.2 tonnes of essential medical
supplies to Maldives via transport aircraft C-130J as an assistance to fight against COVID-19.

Report Errors in the PDF - ebooks@affairscloud.com Copyright 2014-2020 @ AffairsCloud.Com 81


Banking & Economy PDF
About Maldives:
Capital– Male
Currency– Maldivian rufiyaa
President– Ibrahim Mohamed Solih
About SBI:
Formed– July 1955 as SBI
Headquarter– Mumbai, Maharashtra
Chairman– Rajnish Kumar
Digital banking platform– Yono
Tagline– The Banker to Every Indian

Tata AIA Life Insurance Appointed Naveen Tahilyani as its new MD & CEO
On July 23, 2020, Mr. Naveen Tahilyani was appointed as the new Managing Director (MD) and Chief Executive
officer (CEO) of the Tata American International Assurance (AIA). His appointment was under regulatory approval
from Insurance Regulatory and Development Authority of India (IRDAI).
Tahilyani succeeds Rishi Srivastava who has been moved to the Group Agency Distribution, AIA Group, Hong Kong,
as CEO.
About Naveen Tahilyani
i.He was the Senior Partner and Director of McKinsey & Company from July 1977 to November 2014.
ii.Later he joined as the CEO, Managing Director of Tata AIA, Hong Kong from January 2015 to July 2018.
iii.He was the group CEO, Group Partnership District Distribution at AIA Hong Kong till the 2019.
iv.He worked as the group head in-charge of transformation at Axis-Bank.
About Tata AIA Life Insurance:
It is the joint venture of Tata Sons Private Ltd. and AIA Group Ltd. (AIA).
Headquarters – Mumbai, Maharashtra

5th Annual Meeting of Board of Governors of AIIB held virtually; Jin Liqun Reelected President of
AIIB
On July 28, 2020, the 5th Annual Meeting of the Board of Governors of Asian Infrastructure Investment Bank (AIIB)
was held virtually from Beijing, China, was originally planned to be held in-person there, focusing on “Connecting for
Tomorrow”. The meet saw the re-election of Jin Liqun as AIIB President, and a roundtable discussion on the
theme “AIIB 2030-Supporting Asia’s Development over the Next Decade”. Notably, India’s Union Minister of
Finance & Corporate Affairs Nirmala Sitharaman was the Lead Speaker at the roundtable discussion.
• This 5th annual meet was the first virtual for AIIB and also in response to the COVID-19 pandemic.
• During the meet AIIB’s efforts were appreciated for fast tracking financial assistance of about $10 Billion to its
member countries including India to combat COVID-19 pandemic.
• The objective of the annual meeting of the Board of Governors is to take key decisions that impact the Bank’s
future.
Jin Liqun Reelected President of AIIB
During the meet, Jin Liqun was elected President of AIIB for a second five-year term commencing January 16, 2021.
The election result was announced by chair of AIIB’s Board of Governors, Liu Kun, who is a Minister of Finance,
People’s Republic of China.
• Under President Jin’s leadership, AIIB has grown from 57 founding members to 103 approved members from
around the world.
AIIB to Hold its First Annual Meeting in the Middle East in the UAE in 2021
During the meet it was also decided that the sixth Annual Meeting will be held in Dubai, United Arab Emirates (UAE)
on Oct. 27-28, 2021 which will be the first annual meeting in the Middle East. It will take place in parallel with the
Dubai World Expo.
• UAE was also one of the founding members of AIIB.

Report Errors in the PDF - ebooks@affairscloud.com Copyright 2014-2020 @ AffairsCloud.Com 82


Banking & Economy PDF
AIIB’s Membership grew to 103; Liberia joined
During the meet, the Board of Governors approved the application of Liberia to join the Bank. With this AIIB now has
103 approved members from around the world.
• Also, now AIIB has 10 members and nine prospective members from Africa.
Suggestions to AIIB from India
During the round table discussion, Nirmala Sitharaman suggested for introducing new financing instruments,
mobilising private sector finance, providing financing for social infrastructure to achieve the Sustainable
Development Goals (SDGs) 2030, among the following:
• Integrating development of climate resilient and sustainable energy access infrastructure into the recovery
response to the COVID-19 crisis.
• Establishment of Regional Presence which would aid in effective project management and implementation.
Indian Government efforts highlighted in meet
Towards World:
—COVID-19 Emergency Fund for SAARC: The Indian government has created a COVID-19 Emergency Fund for
South Asian Association for Regional Cooperation (SAARC) Nations and supplied critical medical health kits to tackle
the COVID-19.
—India’s participation in G20 Debt Service Suspension Initiative (DSSI): In April, 2020 the World Bank’s
Development Committee and the G20 (Group of Twenty) Finance Ministers endorsed the Debt Service Suspension
Initiative in response to a call by the World Bank and the International Monetary Fund (IMF) to grant debt-service
suspension to the poorest countries to help them manage the severe impact of the COVID-19 pandemic.
–India is now supporting global efforts of COVID-19 vaccine trials.
Towards India:
–The GoI took various measures to respond the COVID-19, including the $23 Billion Pradhan Mantri Garib Kalyan
Yojana (PMGKP) and $295 Billion Atma Nirbhar Bharat Package (ANBP), which aim at protecting all sectors and
sections of the economy.
–Reserve Bank of India (RBI) eased the monetary policy and introduced liquidity in the economy to the extent of
almost 3.9% of Gross Domestic Product (GDP).
–To give a boost to the infrastructure development, India has launched the National Infrastructure Pipeline (NIP)
2020-2025 with estimated expenditure of $1.4 trillion
Recent Related News:
On May 8, 2020, Asian Infrastructure Investment Bank (AIIB), a multilateral development bank, has approved a
loan worth of $500 million for India’s ‘COVID-19 Emergency Response and Health Systems Preparedness Project’ to
assist nation’s efforts to stop, detect, and respond to the risk posed by coronavirus (Covid-19).This is the first-ever
health sector support from AIIB to India.
About Asian Infrastructure Investment Bank (AIIB):
Initiated– 2016
Headquarter– Beijing China
Currently, India has a 7.65% vote share in the AIIB, while China holds 26.63% stake in the organisation.

ADB Approves 3 Million USD Grant to India to Support COVID-19 Response


On 29th July 2020, The Asian Development Bank (ADB) approved the 3 Million USD (about Rs.22 Crore) from the
ADB’s Asia Pacific Disaster Response Fund (APDRF) to India to support the Government’s emergency response to
COVID-19.The amount was financed by Japanese government.
ADB’s Grant:
i.The grant will be utilised to procure thermal scanners and other essential commodities to strengthen the Indian
Governments COVID-19 response.
ii.The grant enhances the ADB’s support to the Government of India and this grant will support the surveillance of the
infection and will help in early detection, contact tracing and treatment of the infection.
iii.This will boost the other public health measures.

Report Errors in the PDF - ebooks@affairscloud.com Copyright 2014-2020 @ AffairsCloud.Com 83


Banking & Economy PDF
Other ADB funds and initiatives:
i.As a part of the 20 million USD expanded assistance for the developing member countries’ pandemic response
announced on 13th April 2020, COVID-19 pandemic response option (CPRO) was established under the ADB’s
Countercyclical Support Facility.
ii.To Support India’s immediate COVID-19 response efforts like disease containment and prevention ADB approved a
1.5 Billion USD COVID-19 Active Response and Expenditure Support (CARES) Program on 28th April 2020 which is
funded through CPRO.
About ADB:
President– Masatsugu Asakawa
Headquarters– Metro Manila, Philippines
Motto– Committed to achieving a prosperous, inclusive, resilient and sustainable Asia & the Pacific, while sustaining
its efforts to eradicate extreme poverty.
Recent Related News:
i.The World Bank‘s Board of Executive Directors has approved a USD 400 million multi-year financial support to help
India enhance its coastal & marine resources, protect marine resources & coastal populations from pollution, erosion,
sea-level rise, and improve livelihood opportunities for coastal communities over the next decade.
ii.World Bank approves Rs 7500 crore (1 billion USD) to increase India’s COVID-19 Social Protection response
Program to help the nation’s efforts.

CSIR, UBA-IIT Delhi and VIBHA Signs a Tripartite MoU to Adopt CSIR Technologies for Rural
Development
On 29th July 2020, Council of Scientific and Industrial Research(CSIR), Unnat Bharat Abhiyan-Indian Institute of
Technology, Delhi (UBA-IITD) and Vijnana Bharati (VIBHA), New Delhi signed a tripartite Memorandum of
Understanding (MoU) to provide access to CSIR rural technologies for Unnat Bharat Abhiyan (UBA) and is expected
to lay the foundation for cooperation and joint action in the area of UBA for rural development of India.
Key People:
Shekhar Mande, Director General of CSIR & Secretary of Department of Scientific and Industrial
Research(DSIR), Virendra Kumar Vijay, National Coordinator, Unnat Bharat Abhiyan, Jayant Jahasrabudhe and
Praveen Ramdas from VIBHA and other professors at the Centre for Rural Development and Technology (CRDT), Dr.
Sunil. K. Khare, Dean, CRDT, IIT Delhi; Professor Vivek Kumar and Dr Priyanka Kaushal, CRDT, IIT Delhi, Prof. Ranjana
Aggarwal, Director CSIR-NISTADS were present during the signing of the MoU.
Objective of MoU:
To adopt the CSIR rural technologies and related educational institutes in line with the targets of the people to
support the initiatives like UBA and VIBHA.
Features of MoU:
i.The MoU enables an adequate structural network, an essential requirement for the implementation of UBA across
the nation.
ii.The network is also essential to achieve the collaborative synergy among the concerned ministries, district
administration, local Panchayat Raj Institutes (PRIs), voluntary organizations and the institutions that participate
with UBA.
iii.The MoU enables the access of the technologies and products developed by CSIR in collaboration with various
stakeholders for the people across the nation.
Unnat Bharat Abhiyan:
Unnat Bharat Abhiyan is a flagship programme under the Ministry of Human Resource Development (MHRD) with a
vision for the transformational change in the process of rural development with the support of educational institutes
to build the framework of an inclusive India.
Vijnana Bharati:
Vijnana Bharati is a science body linked with the Rashtriya Swayamsevak Sangh.
About CSIR:
President- Narendra Modi

Report Errors in the PDF - ebooks@affairscloud.com Copyright 2014-2020 @ AffairsCloud.Com 84


Banking & Economy PDF
Vice President- Harsh Vardhan
Director General– Shekhar C. Mande
Headquarters- New Delhi
Recent Related News:
i.Council of Scientific and Industrial Research (CSIR) and Atal Innovation Mission (AIM) have signed a letter of intent
(LoI) to promote innovation in the country.
ii.CSIR-CFTRI (Council of Scientific & Industrial Research- Central Food Technological Research Institute) and APEDA
(Agricultural & Processed Food Products Export Development Authority) Signs MoU for Liaison Office in Guwahati to
Promote Food Processing & Organic Agri Exports in North-East States.

BOB Financial Solutions Limited ropes Fiserv, Inc for Digital Transformation
BOB Financial Solutions Limited(BFSL) has chosen Fiserv, Inc. to enable the digitization of their end-to-end card
issuance and processing cycle and support the launch of several new and high-tech products including contactless
credit cards, virtual credit cards among others.
• For this purpose, BFSL will utilize Fiserv’s FirstVisionTM, an end-to-end managed services solution.
About FirstVisionTM
i.It is an end-to-end managed services solution that enables card issuance and processing with global economies of
scale and integrated capabilities that span the card lifecycle.
ii.The platform will help BFSL to advance its digital transformation through a fully integrated suite of card
management tools (including digital card and membership management, advanced fraud modules, risk management,
and analysis solutions).
iii.The FirstVision’s service-oriented architecture and open API facilitates rapid application development, allowing
new features to be brought to market faster at a lower cost.
iv.The software as a service (SaaS) solution is hosted locally in India and it can handle card processing for major card
issuers across the country, ensuring compliance with local payment and customer data regulations.
About BFSL
It is a Non Banking Financial Company that is wholly owned by Bank of Baroda.
Managing Director(MD) and Chief executive Officer(CEO)– Manish Banerjea
About Fiserv, Inc.
Headquarters– Brookfield, United States
Executive Chairman– Jeffery W. Yabuki

Praj Signed MoU with ARAI to Jointly Develop Advanced Biofuels Application technology
Praj Industries (Praj) and Automotive Research Association of India (ARAI) signed a Memorandum of
Understanding (MoU) under which they will jointly develop advanced biofuels application technology for the
usage in industry and transportation. This technology will help to reduce the health risks posed by air pollution in the
transportation sector.
About the agreement
i.This collaboration will enable them to jointly address technologies to expand the use of biofuels in a variety of
applications, including the use of Internal Combustion Engines (ICEs) in the transportation sector.
ii.As per the MoU, Praj will provide biofuel technology solutions through its TEMPO(Technology, Engineering,
Manufacturing, Project management, Operations) business model and ARAI will contribute its vast experience in the
field of alternative fuels, green and sustainable mobility.
iii.The projects that are jointly undertaken will reinforce the Praj’s Bio-Mobility TM platform that offers technology
solutions globally to produce carbon neutral transportation fuel from bio-based feedstock for all modes of mobility
Note– The collaboration also aims to strengthen the position of India as a technology leader in the global biofuels
industry.
Benefits of Biofuels
i.Biofuels developed for the mobility sector will have a potential to reduce carbon footprint and improve tailpipe
emissions.

Report Errors in the PDF - ebooks@affairscloud.com Copyright 2014-2020 @ AffairsCloud.Com 85


Banking & Economy PDF
ii.They are derived by processing bio-based feedstock such as agri residue, molasses, cane syrup among others.
iii.They facilitate energy self-reliance and also help conserve the environment.
Relationship b/w Transportation sector & GHG Emission
i.The transportation sector is the largest consumer of imported crude oil and natural gas.
ii.As per the world energy statistics data, it is the 3rd largest source of Greenhouse gases(GHG) emissions after the
power sector & industry.
iii.As a signatory to the Paris summit to reduce GHG emissions by 30-35%, India has to redefine its transportation
fuel mix.
Recent Related News:
i.The Power Finance Corporation(PFC) signed an agreement to fund Rs. 22,000 crore with Narmada Basin Projects
Company limited(NBPCL) of Government of Madhya Pradesh for the 225 MW hydro-electric projects and
multipurpose projects in the state.
ii.Indian Air Force signed MoU with Savitribai Phule Pune University to establish a ‘Chair of Excellence’
named “Marshal of the Air Force Arjan Singh Chair of Excellence”.
About Praj:
Headquarters– Pune, Maharashtra
Founder & Executive Chairman– Pramod Chaudhari
Managing Director(MD) and Chief executive Officer(CEO)– Shishir Joshipura
About ARAI
HeadOffice- Pune, Maharashtra
Officiating Director– Nilkanth V Marathe

Banking, Finance & Economy Q&A: July 2020


1. The world bank and Government of India has signed agreement along with which state to increase access
to affordable housing to low income group?
1) Kerala
2) Tamil Nadu
3) Karnataka
4) Andhra Pradesh
5) Maharashtra
Answer-2) Tamil Nadu
Explanation:
The World Bank and the government of India signed an agreement along with the Government of Tamil Nadu to help
the low-income groups in the state of Tamil Nadu to get access to affordable housing. Agreements signed: – Legal
agreements of two projects were signed to strengthen the state’s housing sector policies, institutions, and regulations.
First Tamil Nadu Housing Sector Strengthening Programme worth $200 million (Rs 1,510 crore), Tamil Nadu Housing
and Habitat Development Project worth $50 million (Rs 377 crore).

2. Find the bank which has partnered with Swiggy to launch industry first instant digital wallet ‘Swiggy
Money’?
1) HDFC Bank
2) RBL Bank
3) Axis Bank
4) IndusInd Bank
5) ICICI Bank
Answer-5) ICICI Bank
Explanation:
Swiggy has launched its own digital wallet, ‘Swiggy Money’ in partnership with ICICI Bank to enable a ‘single-click
checkout experience’ on its platform. It is an industry- first instant digital wallet. The Swiggy Money is powered by

Report Errors in the PDF - ebooks@affairscloud.com Copyright 2014-2020 @ AffairsCloud.Com 86


Banking & Economy PDF
ICICI Bank’s ‘insta wallet service’ which is built on a cloud platform with Application Programming Interface
(API)integration.

3. China signed $2.4 billion worth agreement with Pakistan for hydro power project on which river?
1) Jhelum river
2) Ravi river
3) Beas river
4) Chenab river
5) Sutlej river
Answer-1) Jhelum river
Explanation:
A tripartite agreement was signed between a Chinese company and the governments of Pakistan and China on
Thursday for construction of a 1,124-megawatt hydropower project at Kohala on the Jhelum river at a cost of USD 2.4
billion.

4. What is the currency of Iceland?


1) Rupiah
2) Ringgit
3) Krone
4) Krona
5) Won
Answer-4) Krona
Explanation:
The capital and currency of Iceland are Reykjavik & Icelandic Krona respectively.

5. The WTO settlement body has set up panel against which country based on complaint of European Union
regarding import duties on ICT (information and communication technology)?
1) Australia
2) Russia
3) United States
4) China
5) India
Answer-5) India
Explanation:
The World Trade Organization (WTO) Dispute Settlement Body (DBS) has established a panel after the European
Union (EU) has dragged India into the Dispute Settlement System of WTO for the second time against import duties
on ICT (information and communication technology) products, including mobile phones, cameras, headphones and
earphones, imposed by the country.

6. PM Modi has announced the extension of Pradhan Mantri Garib Kalyan Ann Yojana till November 2020
recently. What is the outlay towards extension of the scheme?
1) 80,000 crores
2) 70,000 crores
3) 60,000 crores
4) 90,000 crores
5) 50,000 crores
Answer-4) 90,000 crores
Explanation:
Prime Minister Shri Narendra Modi, during televised address to the nation, has announced the extension of Pradhan
Mantri Garib Kalyan Ann Yojana (PMGKAY) from July, 2020 till the end of November 2020, with an outlay of more
than Rs 90,000 crore. In this regard, more than 80 crore people will be provided 5 kg free wheat or rice per month

Report Errors in the PDF - ebooks@affairscloud.com Copyright 2014-2020 @ AffairsCloud.Com 87


Banking & Economy PDF
along with 1 kg free whole chana (pulses) to each family per month. The scheme was initially rolled out for three
months i.e. April-June.

7. What is the amount that was provided by World bank to India for MSME Emergency Response Program?
1) USD 750 million
2) USD 850 million
3) USD 500 million
4) USD 700 million
5) USD 800 million
Answer-1) USD 750 million
Explanation:
The World Bank approved a USD 750 million(about Rs.5600 Crore) Micro, Small, and Medium Enterprises(MSMEs)
Emergency Response Program to support severely impacted MSMEs in India due to COVID-19 crisis. The
International Bank for Reconstruction and Development(IBRD) will provide the loan.

8. What is the increase in Foreign Direct Investment (FDI) in India during 2019-20 as per India’s
International Investment Position (IIP) data released by RBI?
1) $ 23 billion
2) $15 billion
3) $ 21 billion
4) $19 billion
5) $17 billion
Answer-4) $ 19 billion
Explanation:
In accordance with the India’s International Investment Position (IIP) data as at end-March 2020, released by Reserve
Bank of India (RBI), Foreign Direct Investment (FDI) in the country has raised by $19 billion while portfolio
investment declined by $13.7 billion during 2019-20.

9. Find the bank which has launched ‘e-Kisaan Dhan’ app for farmers.
1) HDFC Bank
2) IDBI Bank
3) RBL Bank
4) ICICI Bank
5) DBS Bank
Answer-1) HDFC Bank
Explanation:
HDFC Bank has launched ‘e-Kisaan Dhan’ App for Farmers in India. This app provides information and knowledge to
farmers which will help them to meet the needs of the rural ecosystem.

10. Which bank is setting up an e-commerce portal named ‘Bharat Craft’ for marketing of products
manufactured by MSMEs?
1) State Bank of India
2) Punjab National Bank
3) Canara Bank
4) United Bank of India
5) Indian Bank
Answer-1) State Bank of India
Explanation:
State Bank of India is working on setting up an e-commerce portal for marketing of products manufactured by micro,
small and medium enterprises (MSMEs) in the country, its chairman Rajnish Kumar said on Saturday. The portal
called Bharat Craft would be jointly run by the bank and the government.

Report Errors in the PDF - ebooks@affairscloud.com Copyright 2014-2020 @ AffairsCloud.Com 88


Banking & Economy PDF
11. As per Fitch ratings Inc, India’s growth forecast for FY22 is _____.
1) 6%
2) 7%
3) 8%
4) 5%
5) 4%
Answer-3) 8%
Explanation:
Fitch Ratings Inc in its June 2020 update of Global Economic Outlook (GEO), has lowered India’s growth forecast for
FY 2021-22 to 8% from 9.5 % projected in May 2020 due to the toughest lockdown imposed by the central
government to prevent the outbreak of Coronavirus (COVID-19). However, Fitch has not made any change in its
previous estimate of a 5% decline in the economy during the current financial year (FY 2020-21).

12. Rajkiran Rai G’s tenure as MD&CEO of which bank was extended for 2 years?
1) Canara Bank
2) Indian Bank
3) Punjab National Bank
4) State Bank of India
5) Union Bank of India
Answer-5) Union Bank of India
Explanation:
Government extends the term of Rajkiran Rai G as the Managing Director(MD) and Chief Executive Officer(CEO) of
Union Bank of India(UBI) by 2 years till May 31, 2022.

13. Where is the HQ of Union Bank of India (UBI) located?


1) Mumbai
2) Chennai
3) Bengaluru
4) Kolkata
5) New Delhi
Answer-1) Mumbai
Explanation:
The HQ of Union Bank of India (UBI) located at Mumbai, Maharashtra.

14. What is the % increase in marine fish production 2019 when compared to 2018, as per report released by
Central Marine Fisheries Research Institute (CMFRI)?
1) 2.9%
2) 3.2%
3) 1.7%
4) 2.1%
5) 0.9%
Answer-4) 2.1%
Explanation:
In accordance with the report “Annual Marine Fish Landings in India for 2019” by Central Marine Fisheries Research
Institute (CMFRI), India’s marine fish production has increased marginally by 2.1% to 3.56 million tonnes (mt) in
2019 as compared to 3.49 mt in 2018.

15. Which state topped in fish production based on volume, as per “Annual Marine Fish Landings in India for
2019” report of CMFRI?
1) Kerala
2) Tamil Nadu

Report Errors in the PDF - ebooks@affairscloud.com Copyright 2014-2020 @ AffairsCloud.Com 89


Banking & Economy PDF
3) Gujarat
4) Maharashtra
5) West Bengal
Answer-2) Tamil Nadu
Explanation:
In accordance with the report “Annual Marine Fish Landings in India for 2019” by Central Marine Fisheries Research
Institute (CMFRI), India’s marine fish production has increased marginally by 2.1% to 3.56 million tonnes (mt) in
2019 as compared to 3.49 mt in 2018. As per the report, Tamil Nadu grabbed the top in annual fish production with
7.75 lakh tonnes followed by Gujarat (7.49 lakh tonnes), which had been holding the first position earlier, and Kerala
(5.44 lakh tonnes) which retained the third position.

16. Which state has amended Fiscal Responsibility and Budget Management Act which will enable the state to
increase the borrowing limit to 5% from 3% of GSDP?
1) Uttar Pradesh
2) Bihar
3) Gujarat
4) Telangana
5) Himachal Pradesh
Answer-4) Telangana
Explanation:
The Governor of Telangana Dr.TamilisaiSounderarajan has promulgated an ordinance No.3 of 2020, amending the
Telangana Fiscal Responsibility and Budget Management (FRBM) Act-2005 which will enable the State to increase the
borrowing limit by 2% to 5% from earlier 3%. With the new Telangana FRBM (Amendment) Act 2020, state
government’s additional borrowing limit can go up to Rs 20,000 crore now, without any hassles. With this
amendment, the Telangana State government has accepted the recent decision of the Central government to expand
the borrowing limit of the States under FRBM Act, 2003 from 3% to 5% allowing the States to borrow more as there
is fall in state’s revenues due to COVID-19 induced lockdown.

17. What is the estimated outlay of Pradhan Mantri Garib Kalyan Ann Yojana (PMGKAY) between April-
November 2020 as per Department of Food and Public Distribution?
1) 2,48,938 crores
2) 1,24,938 crores
3) 1,48,938 crores
4) 1,98,938 crores
5) 3,28,938 crores
Answer-3) 1,48,938 crores
Explanation:
Department of Food and Public Distribution (DFPD) under Ministry of Consumer Affairs, Food and Public Distribution
has worked out estimated cost for distribution of food grains (Rice and Wheat) and pulses under PMGKAY, which will
be Rs 1,48,938 crore approx for a period between 8 months from April to November 2020. Notably, Govt. of India is
bearing the entire expenditure towards this Scheme.

18. The central government has approved special liquidity scheme (SLS) for NBFCs and HFCs through a
Special Purpose Vehicle (SPV) recently. Name the company which set up a SPV called SLS Trust to manage the
operations.
1) IDBI Capital Markets Limited
2) HDFC Capital Markets Limited
3) PNB Capital Markets Limited
4) IOB Capital Markets Limited
5) SBI Capital Markets Limited

Report Errors in the PDF - ebooks@affairscloud.com Copyright 2014-2020 @ AffairsCloud.Com 90


Banking & Economy PDF
Answer-5) SBI Capital Markets Limited
Explanation:
Government of India (GoI) has approved a scheme for improving the short-term liquidity position of non-banking
finance companies (NBFCs)/housing finance companies (HFCs) through a Special Purpose Vehicle (SPV) in the form
of Special Liquidity Scheme (SLS) Trust set up by SBI Capital Markets Limited (SBICAP), a subsidiary of the State Bank
of India (SBI). Means, SBICAP will manage this operation. In the Budget Speech of 2020-21, it was announced that a
mechanism would be devised to provide additional liquidity facility to NBFCs/HFCs over that provided through the
Partial Credit Guarantee Scheme (PCGS). It should be noted that the finances provided to the NFBCs/HFCs under this
scheme should be used to repay existing liabilities and not to expand assets.

19. Which bank has launched ‘Insta Loans against Mutual Funds’ facility in partnership with Computer Age
Management Services (CAMS)?
1) HDFC Bank
2) IDFC Bank
3) ICICI Bank
4) RBL Bank
5) IDBI Bank
Answer-3) ICICI Bank
Explanation:
ICICI Bank in partnership with Computer Age Management Services(CAMS) launched ‘Insta Loans against Mutual
Funds’, a facility for retail customers to avail loans upto Rs 1 crore instantly by pledging their holdings in both debt
and equity Mutual Funds(MFs).

20. KarnamSekar has been retired from the post of MD&CEO of which bank?
1) State Bank of India
2) Union Bank of India
3) Dena Bank
4) Punjab National Bank
5) Indian Overseas Bank
Answer-5) Indian Overseas Bank
Explanation:
KarnamSekar, The Managing Director (MD) and Chief Executive Officer(CEO) of Indian Overseas Bank(IOB), retired
on 30th June 2020.

21. The US based Carlyle Group decided to acquire _____% stake in Nxtra Data Ltd, an arm of Bharti Airtel for
$235 million.
1) 25
2) 20
3) 30
4) 15
5) 35
Answer-1) 25
Explanation:
American multinational private equity company Carlyle Group has decided to acquire about 25% stake in India’s
second largest telecom company Bharti Airtel’s data centre business arm–Nxtra Data Ltd– for $235 million (about
₹1,780 crore) through a company called Comfort Investments II, which is an affiliated entity of CAP V Mauritius Ltd.,
an investment fund managed and advised by affiliated firms of the Carlyle. At the same time, Airtel will continue to
hold the remaining 75 % stake.

Report Errors in the PDF - ebooks@affairscloud.com Copyright 2014-2020 @ AffairsCloud.Com 91


Banking & Economy PDF
22. Where is the HQ of Indian Overseas Bank located?
1) Mumbai
2) Lucknow
3) Hyderabad
4) Bengaluru
5) Chennai
Answer-5) Chennai
Explanation:
The HQ of Indian Overseas Bank located at Chennai, Tamil Nadu.

23. Which bank has joined as the observer in Paris based Network for Greening the Financial System (NGFS)
recently?
1) World Bank
2) Asian Development Bank
3) Reserve Bank of India
4) New Development Bank
5) None of the Above
Answer-2) Asian Development Bank
Explanation:
Asian Development Bank (ADB), a regional development bank, has joined the Paris (France)-based Network for
Greening the Financial System (NGFS) as an observer. The NGFS is a global forum that brings together central banks
and supervisors committed to understanding and managing the financial risks and opportunities associated with
climate change.

24. Under which operation the RBI has conducted open market operation (OMO) of purchase and sale of G-
secs for Rs 10,000 crores each recently?
1) Operation reserve
2) Operation twist
3) Operation Borrow
4) Operation liquidity
5) Operation lending
Answer-2) Operation twist
Explanation:
Reserve Bank of India (RBI) conducted the simultaneous purchase of Government Securities (G-Secs), maturing
between May 2027 and October 2033, and sale of 182 day and 364 day treasury bills (DTBs) under Open Market
Operations (OMO) for Rs 10,000 crore each. This OMO is a part of “Operation Twist” to ease pressure evolving
liquidity and market conditions. The main purpose of the OMO is to bring down the yields at the longer end.

25. Name the bank which offers an online instant auto loan to its customers named ‘ZipDrive’
1) CSB Bank
2) RBL Bank
3) IDBI Bank
4) ICICI Bank
5) HDFC Bank
Answer-5) HDFC Bank
Explanation:
HDFC Bank announced that it will offer ‘ZipDrive’, an online instant auto loan to its customers in 1,000 cities. The
auto loan disbursal product is only for customers with pre-approved offers. It is a technology-enabled loan product
where credit assessment is done by banking software.

Report Errors in the PDF - ebooks@affairscloud.com Copyright 2014-2020 @ AffairsCloud.Com 92


Banking & Economy PDF
26. Which private sector bank collaborated with Innoviti Payment Solutions recently?
1) Axis Bank
2) YES Bank
3) Kotak Mahindra Bank
4) IndusInd Bank
5) ICICI Bank
Answer-3) Kotak Mahindra Bank
Explanation:
Kotak Mahindra Bank (Kotak) and Innoviti Payment Solutions Pvt. Ltd. announced that they have collaborated to
provide Equated Monthly Instalments(EMI) billing options on Kotak Debit Cards which are swiped on Innoviti Point
of Sale(POS) terminals.

27. What is the increase in % of external debt of India in March 2020 when compared to March 2019, as per
RBI data?
1) 2.1%
2) 4.9%
3) 2.8%
4) 1.2%
5) 0.6%
Answer-3) 2.8%
Explanation:
In accordance with the Reserve Bank of India (RBI) data, India’s external debt stood at $558.5 billion in March 2020,
an increase of $15.4 billion or 2.8% as compared to the March 2019. The largest component of the external debt is
Commercial borrowings with a share of 39.4%, followed by non-resident deposits at 23.4% and short-term trade
credit at 18.2%. In terms of currency, United States (US) dollar-denominated debt continued to be the largest
component of India’s external debt, with a share of 53.7% at end-March 2020, followed by the Indian rupee (31.9%),
yen (5.6%), SDR (4.5%) and the euro (3.5%).

28. India’s GDP will contract by _____%in FY21 as per Care ratings.
1) 3.2%
2) 4.5%
3) 6.4%
4) 4.9%
5) 7.1%
Answer-3) 6.4%
Explanation:
CARE Ratings Limited ( formerly Credit Analysis and Research Limited) in its report, ‘Revised GDP growth projections
for FY21’ has projected a 6.4 % contraction (or -6.4%) in India’s GDP (Gross Domestic Product) growth for the
current financial year FY 21 (2020-21) due to the continuous lockdown imposed for prevention of coronavirus
(COVID-19) epidemic.

29. The US based Intel capital planned to buy _____% stake in Jio platforms for Rs 1,894.50 crore.
1) 0.87%
2) 1.21%
3) 0.39%
4) 2.42%
5) 0.61%
Answer-3) 0.39%
Explanation:
US based Electronic chip maker Intel’s investment arm, Intel Capital to buy 0.39% stake in Reliance Industries

Report Errors in the PDF - ebooks@affairscloud.com Copyright 2014-2020 @ AffairsCloud.Com 93


Banking & Economy PDF
Limited(RIL)’s Jio Platforms for Rs 1,894.50 crore. This is the 12th deal of Jio Platforms. With this investment, Jio
Platforms has raised Rs 1,17,588.45 crore from global investors since April 22.

30. How much was the amount approved by the Defence Acquisition Council for capital acquisition of
equipment for armed forces (July 2020)?
1) Rs 29,200 crore
2) Rs 41,800 crore
3) Rs 32,500 crore
4) Rs 38,900 crores
5) Rs 36,700 crore
Answer-4) Rs 38,900 crores
Explanation:
Defence Acquisition Council (DAC) has given its consent for the capital acquisition of various platforms and
equipment required by the Indian Armed Forces, Army, Navy and the Air Force, worth Rs 38,900 crore. This decision
was taken during the DAC meet held on July 2, 2020 under the chairmanship of Raksha Mantri (Defence Minister)
Rajnath Singh. Notably, these approvals include acquisitions of Rs 31,130 crore from Indian industry signifying Prime
Minister (PM) Narendra Modi’s clarion call for ‘Atmanirbhar Bharat’ focused on indigenous design and development.

31. Where is the HQ of HDFC bank located?


1) Lucknow
2) New Delhi
3) Pune
4) Mumbai
5) Gurugram
Answer-4) Mumbai
Explanation:
The HQ of HDFC bank located at Mumbai, Maharashtra.

32. Who is the present chairman of SEBI?


1) Amitabh Kant
2) AjitDoval
3) Rajiv Kumar
4) Ajay Tyagi
5) VK Paul
Answer-4) Ajay Tyagi
Explanation:
Ajay Tyagi is the present chairman of SEBI.

33. How much loss will be incurred in tourism sector because of 4-month lockdown due to COVID-19 as per
UNCTAD’s report?
1) USD 2.2 trillion
2) USD 1.2 trillion
3) USD 1.8 trillion
4) USD 0.9 trillion
5) USD 2.4 trillion
Answer-2) USD 1.2 trillion
Explanation:
The United Nations Conference on Trade and Development (UNCTAD) published the report, COVID-19 and Tourism:
Assessing the Economic Consequences stating that the World Tourism sector could lose around 1.2 trillion USD or
1.5% of the Global Gross Domestic Product (GDP) because of the 4months lockdown due to global pandemic situation.

Report Errors in the PDF - ebooks@affairscloud.com Copyright 2014-2020 @ AffairsCloud.Com 94


Banking & Economy PDF
34. Which Indian state has launched ‘Balaram’ scheme (designed in collaboration with NABARD) to provide
agricultural credit to landless farmers?
1) Odisha
2) Jharkhand
3) Bihar
4) Assam
5) Sikkim
Answer-1) Odisha
Explanation:
Government of Odisha launched 1st of its kind scheme, ‘Balaram’ to provide agricultural credit of Rs 1,040 crore to
landless farmers facing hardships during COVID 19.The Scheme is designed in collaboration with National Bank for
Agriculture and Rural Development (NABARD).

35. Find the Asset Management Company (AMC) which is going to launch second tranche of the Bharat Bond
ETF to raise Rs 14,000 crore.
1) Kotak Mahindra Asset Management Company
2) Edelweiss Asset Management Company
3)ICICI Prudential Asset Management Company
4) L&T Investment Management Limited
5) HDFC Asset Management Company
Answer-2) Edelweiss Asset Management Company
Explanation:
Edelweiss Asset Management Company (AMC), a subsidiary of Edelweiss Capital Limited (ECL) has decided to bring
the second instalment of the Bharat Bond ETF (Exchange Traded Fund) with two new series to raise Rs 14,000 crore
in July 2020.Through the launch of these two new ETF series, which have maturities of 5 years and 11 years,
Edelweiss aims to raise an initial amount of Rs. 2,000 crores from institutional players with a greenshoe option of Rs.
6,000 crores 5-year maturity period (April 2025) and initial amount of Rs. 1,000 crores with a green shoe option of
Rs. 5,000 crores in 11 years (April 2031).

36. Which insurance company has partnered with BOB financial to offer COVID-19 insurance cover?
1) Oriental Insurance
2) Tata AIA Life Insurance
3) IndiaFirst Life Insurance
4) Max Life Insurance
5) Future Generali India Life Insurance
Answer-3) IndiaFirst Life Insurance
Explanation:
IndiaFirst Life Insurance Company Limited(IndiaFirst Life) associated with BOB Financial Services Limited (BOB
Financial), a wholly owned subsidiary of Bank of Baroda(BOB) to offer a group insurance cover. The cover will
provide financial support to the BOB credit card customers upon their hospitalisation or on COVID-19 diagnosis.

37. Which PSU signed a pact with Coal India to form 50:50 Joint Venture to develop 5000MW solar and
thermal power assets?
1) GAIL India
2) PGCIL
3) NLC India Limited
4) NTPC Limited
5) SAIL India
Answer-3) NLC India Limited
Explanation:
NLC India Limited signed a pact with Coal India Ltd (CIL) to form a joint venture (JV) to develop 5,000 megawatts

Report Errors in the PDF - ebooks@affairscloud.com Copyright 2014-2020 @ AffairsCloud.Com 95


Banking & Economy PDF
(MW) of solar and thermal power assets across the country. There will be equal equity participation of both
companies in the proposed JV i.e. in the ratio of 50:50. Notably, both undertakings are under the administrative
control of the Ministry of Coal.

38. The US based KKR has agreed to buy _____% stake in Mumbai-based JB Chemicals & Pharmaceuticals for Rs
3100 crore (approx).
1) 60
2) 51
3) 29
4) 54
5) 75
Answer-4) 54
Explanation:
US private equity giant KKR has agreed to acquire a 54% stake(41.7 million or 4.17 crore fully paid up equity shares)
in Mumbai-based drug manufacturer JB Chemicals & Pharmaceuticals for approximately Rs 3100 crore. KKR will
make the investment through its subsidiaries, Tau Investments Holdings Pte. Ltd., Tau Holdco Pte. Ltd., and KKR Asia
III Fund Investments Pte.

39. Who is the present chairman of NABARD?


1) P.V.S Suryakumar
2) Govinda Rajulu Chintala
3) Shaji K V
4) Harsh Kumar Bhanwala
5) Ajay Tyagi
Answer-2) Govinda Rajulu Chintala
Explanation:
Govinda Rajulu Chintala was appointed as the chairman of NABARD in May 2020.

40. Find the bank which partnered with Mastercard to facilitate real-time domestic & cross-border payments
& remittances through ‘Mastercard Send’.
1) CSB Bank
2) IndusInd Bank
3) RBL Bank
4) ICICI Bank
5) SBM Bank
Answer-5) SBM Bank
Explanation:
SBM Bank India has partnered with Mastercard to enable its customers to make real-time domestic & cross-border
payments(Business to Consumer- B2C) and remittances quickly and efficiently through ‘Mastercard Send’. This
partnership allows the bank’s customer base to offer quick, convenient and secure payments experiences.

41. Which private sector bank launched ‘Loan in seconds’ digital solution for instant disbursement of retail
loans?
1) HDFC Bank
2) Axis Bank
3) Yes Bank
4) City Union Bank
5) Karur Vysya Bank
Answer-3) Yes Bank
Explanation:
YES Bank launched a digital solution, ‘Loan in Seconds’ for instant disbursement of retail loans. It is available for the

Report Errors in the PDF - ebooks@affairscloud.com Copyright 2014-2020 @ AffairsCloud.Com 96


Banking & Economy PDF
bank’s pre-approved liability account holders. The algorithm evaluates the loan application in real-time and
eliminates the need for documentation.

42. What is India’s rank in SDG index 2020 (Sweden ranked No.1)?
1) 89
2) 121
3) 117
4) 62
5) 51
Answer-3) 117
Explanation:
In accordance with the “Sustainable Development Report 2020- The Sustainable Development Goals and Covid-19”,
comprising of SDG index 2020, there will be severe negative impacts on most of the United Nations (UN)-mandated
Sustainable Development Goals (SDGs) due to COVID-19 pandemic. India, which has ranked at 117th position with a
score of 61.92 is also facing major challenges in 10 of the 17 SDGs including zero hunger, good health, gender
inequality among others. The index has been topped by Sweden.

43. India ranked _____ in INFORM risk index 2020 for humanitarian crises & disasters (Somalia – 1).
1) 22nd
2) 31st
3) 29th
4) 51st
5) 78th
Answer-2) 31st
Explanation:
In accordance with the “INFORM Report 2020:Shared evidence for managing crises and disasters” released by
research Centre INFORM, operating under European Commission (EU), India has ranked at 31st place with an inform
risk of 5.4, means risk of humanitarian crises and disasters. The report ranked the countries based on “INFORM Risk
Index” which has been topped by Somalia with an inform risk of 8.9. The report has analyzed 191 countries. Notably,
this report is produced by the United Nations Office for the Coordination of Humanitarian Affairs (UN-OCHA) on
behalf of all INFORM Partners.

44. Which payment platform company (with QorQl) is set to acquire Raheja QBE General Insurance Company
Limited for $76 million?
1) Paytm
2) CashFree
3) Paykun
4) Paypal
5) PayU
Answer-1) Paytm
Explanation:
Indian e-commerce payment system Paytm &QorQl Pvt. Ltd, a Healthcare startup company with majority
shareholding of Paytm founder Vijay Shekhar Sharma, is set to acquire Mumbai (Maharashtra)-based private sector
general insurer Raheja QBE General Insurance Company Limited in a deal worth around $76 million (or Rs 570
crore). Raheja QBE is a joint venture of India’s Prism Johnson (51%) and QBE Australia (49%).

45. Which edition of GST day was celebrated on July 1, 2020?


1) 1st
2) 2nd
3) 3rd

Report Errors in the PDF - ebooks@affairscloud.com Copyright 2014-2020 @ AffairsCloud.Com 97


Banking & Economy PDF
4) 4th
5) 5th
Answer-3) 3rd
Explanation:
Goods and Service Tax(GST) day is observed annually on July 1. This year marks the 3rd anniversary of the launch of
the day and was marked by the Central Board of Indirect Taxes and Customs(CBIC) and all its field offices across
India. Most of the interactions with stakeholders to mark this day were made in virtual mode due to COVID-19.The
day was 1st celebrated on July 1, 2018.

46. What is the currency of Israel?


1) Yen
2) Pound
3) Dinar
4) Tomar
5) Shekel
Answer-5) Shekel
Explanation:
The capital and currency of Israel are Jerusalem & Israeli Shekel respectively.

47. How much amount was allocated to Himachal Pradesh by central government for the implementation of
Jal Jeevan Mission (for FY21)?
1) 512.80 Crore
2) 815.25 Crore
3) 326.20 Crore
4) 128.96 Crore
5) 239.10 Crore
Answer-3) 326.20 Crore
Explanation:
On 4th July 2020, during a video conference between Union Minister of Jal Shakti, Gajendra Singh Shekhawat and
Chief Minister of Himachal Pradesh (HP) Jairam Thakur, the Union Government allocated Rs. 326.20 Crore for the
effective implementation of Jal Jeevan Mission (JJM) in Himachal Pradesh for FY 2020-21. Including the state’s share,
the total funds available for providing Functional Household Tap Connection (FHTC) is Rs. 371 Crore.

48. Name the country which has inaugurated a new school infrastructure developed with the India’s
assistance of Rs 1.94 crore.
1) Bhutan
2) Nepal
3) Thailand
4) Hong Kong
5) Tibet
Answer-2) Nepal
Explanation:
A new four-storey school building, developed with the Indian assistance of Rs 1.94 crore (Nepalese Rupees (NPR)
31.13 million) under the Nepal-Bharat Maitri Development Cooperation programme has been inaugurated in
ShreeSaptmai Gurukul Sanskrit Vidyalaya, located in Ilam district of Nepal to boost the learning environment for
students. The project was implemented by the District Coordination Committee (DCC), Ilam.

49. Find the payments bank which has launched ‘Bhavishya’ savings account scheme for minors aged
between 10-18 years.
1) India Post Payments Bank
2) Jio Payments Bank

Report Errors in the PDF - ebooks@affairscloud.com Copyright 2014-2020 @ AffairsCloud.Com 98


Banking & Economy PDF
3) Fino Payments Bank
4) Paytm Payments Bank
5) Airtel Payments Bank
Answer-3) Fino Payments Bank
Explanation:
Fino Payments Bank Limited launched ‘Bhavishya‘, a savings account scheme for minors aged between 10-18 years.
The subscription-based savings account aims to inculcate banking and saving habits in children.

50. Which health insurance company has partnered with Karur Vysya bank for bancassurance?
1) SBI Life Insurance Company
2) Oriental Insurance Company
3) Religare Health Insurance Company
4) Star Health and Allied Insurance Company
5) All the above
Answer-4) Star Health and Allied Insurance Company
Explanation:
Karur Vysya Bank (KVB), a Scheduled Commercial Bank in India has joined hands with Star Health and Allied
Insurance Co Ltd for bancassurance (selling of life assurance and other insurance products and services by
bank)through which KVB will market wide range health insurance products of the Star Health through its 780
branches.

51. Name the bank which has planned to enter into strategic partnership with Google Cloud.
1) HSBC Bank
2) BNP Paribas
3) Deutsche Bank
4) Standard Chartered Bank
5) Bank of China
Answer-3) Deutsche Bank
Explanation:
Deutsche Bank and Google Cloud have proposed to enter into a strategic, multi-year partnership. The partnership
focuses on redefining how the bank develops and offers its financial services.

52. The Liberty general insurance company launched “Liberty Assure” service recently. Where does the HQ of
Liberty general insurance located?
1) New Delhi
2) Mumbai
3) Gurugram
4) Kolkata
5) Bengaluru
Answer-2) Mumbai
Explanation:
The Mumbai headquartered Liberty General Insurance Launches, ‘Liberty Assure’, an Industry first service. The car
insurance customers who avail this service, are not required to pay the ‘Compulsory Deductible’ cost for each claim.
In addition to this no premium will be charged from the buyers to provide this unique cost-effective feature at the
time of raising a claim/loss.

53. Name the digital payments company which has entered into partnership with Ola recently.
1) PhonePe
2) PayTm
3) Google Pay

Report Errors in the PDF - ebooks@affairscloud.com Copyright 2014-2020 @ AffairsCloud.Com 99


Banking & Economy PDF
4) Freecharge
5) JioMoney
Answer-1) PhonePe
Explanation:
India based ridesharing company; Ola announced that it has entered into a strategic partnership with PhonePe to
boost its digital payments experience for its customers.

54. Who has been appointed as the 1st chairman of International Financial Services Centre Authority
(IFSCA)?
1) Injeti Srinivas
2) Ajay Tyagi
3) Nandhan Nilekani
4) TN Manoharan
5) Usha Thorat
Answer-1) Injeti Srinivas
Explanation:
Government appointed former corporate affairs secretary and IAS officer, Injeti Srinivas as the 1st chairman of the
International Financial Services Centre Authority(IFSCA) for a 3 years term. IFSCA regulates all financial services in
international financial services centres (IFSCs) in Gandhinagar.

55. What is India’s rank in 11th edition of ‘Global Real Estate Transparency Index 2020’ released by US based
JLL (UK topped the list)?
1) 78
2) 51
3) 42
4) 28
5) 34
Answer-5) 34
Explanation:
According to the 11th edition of the ‘Global Real Estate Transparency Index 2020’, which covers 99 countries and
territories and 163 cities, released by US based property consultant Jones Lang LaSalle’s (JLL), India has improved by
one place to 34th with a total score of 2.69 under ‘semi-transparent’ zone.Regulatory reforms, better market data and
sustainability initiatives led to a one-point improvement in the nation’s ranking.
Rank Country /Territory Score
34th India 2.69
1 United Kingdom 1.31
2 United States 1.35
3 Australia 1.39

56. What is the capital and currency of Indonesia?


1) Thimphu & Ngultrum
2) Jakarta & Rupiah
3) Baghdad & Dinar
4) Nicosia & Euro
5) Tehran & Rial
Answer-2) Jakarta & Rupiah
Explanation:
The capital and currency of Indonesia are Jakarta and Indonesian Rupiah respectively.

Report Errors in the PDF - ebooks@affairscloud.com Copyright 2014-2020 @ AffairsCloud.Com 100


Banking & Economy PDF
57. Where is the HQ of Star Health and Allied Insurance located?
1) Gurugram
2) Lucknow
3) Navi Mumbai
4) Chennai
5) Kochi
Answer-4) Chennai
Explanation:
The HQ of Star Health and Allied Insurance located at Chennai, Tamil Nadu.

58. How much amount was released by Asian Infrastructure Investment Bank as the 1st tranche to L&T
Infrastructure Finance for renewable energy projects in the India?
1) USD 10 million
2) USD 100 million
3) USD 75 million
4) USD 25 million
5) USD 50 million
Answer-5) USD 50 million
Explanation:
Asian Infrastructure Investment Bank(AIIB) has released USD 50 million in the first tranche of assistance to L&T
Infrastructure Finance Ltd(LTIF), to fund renewable Energy Projects in the country. The released amount is part of a
sanctioned USD 100 million External Commercial Borrowing(ECB) loan. This is the 1st loan of AIIB to a non-banking
financial company (NBFC) in India.

59. Find the bank which has entered into Bancassurance agreement with Bajaj Allianz Life Insurance
recently.
1) Axis Bank
2) Karur Vysya Bank
3) RBL Bank
4) Indian Overseas Bank
5) ICICI Bank
Answer-2) Karur Vysya Bank
Explanation:
Karur Vysya Bank (KVB), a Scheduled Commercial Bank in India has joined hands with Bajaj Allianz Life Insurance for
bancassurance, under which KVB will market all retail and group products of Bajaj Allianz including savings,
retirement, investment, protection and critical illness life insurance products through its 780 branches.

60. Who is the president of the Beijing based Asian Infrastructure Investment Bank (AIIB)?
1) Andrey Kruglov
2) Adel AlHosani
3) Muhammad Aslam
4) Dinanath Dubhashi
5) Jin Liqun
Answer-5) Jin Liqun
Explanation:
JinLiqun is the present president of the Asian Infrastructure Investment Bank(AIIB). The HQ of AIIB located at Beijing,
China.

Report Errors in the PDF - ebooks@affairscloud.com Copyright 2014-2020 @ AffairsCloud.Com 101


Banking & Economy PDF
61. The Union cabinet has approved 10-year long pan India Central Sector Scheme namely “Agriculture
Infrastructure Fund” recently. What is the outlay of the scheme?
1) 50,000 Crore
2) 2 Lakh Crore
3) 1 Lakh Crore
4) 25,000 Crore
5) 75,000 Crore
Answer-3) 1 Lakh Crore
Explanation:
As a part of the Rs 20 lakh crore “Atmanirbhar Bharat Abhiyan” announced in response to the COVID-19 crisis, the
Union Cabinet has given its approval to a 10 years long (FY2020 to FY2029) pan India Central Sector Scheme namely
“Agriculture Infrastructure Fund” with an outlay of Rs. 1 Lakh Crore in order to support agri-entrepreneurs, start-ups,
agri-tech players and farmer groups for infrastructure and logistics facilities. Under this, the total outflow as
budgetary support from Government of India (GoI) will be Rs.10,736 crores.

62. The cabinet has approved the capital infusion of 12,450 crores to which public sector general insurance
company/companies??
1) Oriental Insurance Company Limited
2) National Insurance Company Limited
3) United India Insurance Company Limited
4) Both 1) and 2)
5) All the 1), 2) and 3)
Answer-5) All the 1), 2) and 3)
Explanation:
The Union Cabinet has approved the capital infusion for an overall value of Rs.12,450 crores; (including Rs. 2,500
crore infused in FY 2019-20) in the three Public Sector General Insurance Companies (PSGICs) namely Oriental
Insurance Company Limited (OlCL), National Insurance Company Limited (NICL) and United India Insurance
Company Limited (UIICL). Of this, Rs.3,475 crores will be released immediately; while the balance Rs,6475 crores will
be infused later.

63. The union cabinet approved development of affordable rental housing Complexes for urban migrants /
poor worth ________ under PMAY-U.
1) 100 Crores
2) 250 Crores
3) 600 Crores
4) 400 Crores
5) 500 Crores
Answer-3) 600 Crores
Explanation:
Asa part of the Rs 20 lakh crore economic package to deal with the COVID-19 pandemic, Union Cabinet has given its
approval for developing of Affordable Rental Housing Complexes (AHRCs) for urban migrants / poor as a sub-scheme
under Pradhan MantriAwasYojana – Urban (PMAY – U). The outlay of the scheme is estimated at Rs 600 crore in the
form of Technology Innovation Grant and will initially cover three lakh beneficiaries.

64. Find the state which topped in market borrowings in FY 21 as per RBI data?
1) Andhra Pradesh
2) Tamil Nadu
3) Rajasthan
4) Maharashtra
5) Uttar Pradesh

Report Errors in the PDF - ebooks@affairscloud.com Copyright 2014-2020 @ AffairsCloud.Com 102


Banking & Economy PDF
Answer-2) Tamil Nadu
Explanation:
According to data from the Reserve Bank of India (RBI), Tamil Nadu (TN) has topped market borrowings among
States in India with a collection of Rs.30,500 crores in the fiscal 2020-21(FY 21).

65. What will be the contraction in world economy in 2020 as per ‘Country Risk and Global Outlook Report’
released by D&B?
1) 5%
2) 4.8%
3) 6%
4) 5.2%
5) 7.1%
Answer-4) 5.2%
Explanation:
In accordance with the Dun and Bradstreet’s (D&B) Country Risk and Global Outlook Report, which has covered 132
countries, the global economy is likely to contract by 5.2% in 2020. This is the biggest decline since the Second World
War and a far stronger contraction than the 1.7% recorded in 2009 during the global financial crisis. As per report,
the global economy will not reach pre-pandemic levels of activity again before 2022.

66. Which organization has signed MoU with Central Board of Direct Taxes (CBDT) for data exchange?
1) Forward Market Commission of India
2) Reserve Bank of India
3) Insurance Regulatory and Development Authority of India
4) Pension Fund Regulatory and Development Authority
5) Securities and Exchange Board of India
Answer-5) Securities and Exchange Board of India
Explanation:
The Central Board of Direct Taxes (CBDT) and Securities and Exchange Board of India (SEBI) signed a Memorandum
of Understanding (MOU) for the data exchange between the two organizations via video conference.

67. What is India’s rank based on cancer preparedness among Asia-Pacific countries as per Economist
Intelligence Unit’s (EIU) report?
1) 8th
2) 2nd
3) 4th
4) 9th
5) 6th
Answer-1) 8th
Explanation:
In accordance with the Economist Intelligence Unit (EIU) report on “Cancer preparedness in Asia-Pacific: Progress
towards universal cancer control”, India has been ranked eighth with an overall score of 51.6 for cancer preparedness
among 10 Asia-Pacific countries. The report sponsored by Roche, examined the findings from the EIU’s Index of
Cancer Preparedness (ICP) which was created upon the findings of the global ICP (evaluated 28 countries based on
45 separate indicators).The 10 Asia-Pacific countries included in this report were — Australia, China, India,
Indonesia, Japan, Malaysia, the Philippines, South Korea, Thailand and Vietnam.
Rank Country Score (out of 100)
8 India 51.6
1 Australia 92.4
2 South Korea 83.4

Report Errors in the PDF - ebooks@affairscloud.com Copyright 2014-2020 @ AffairsCloud.Com 103


Banking & Economy PDF

3 Malaysia 80.3

68. Who is the current chairperson of Central Board of Direct Taxes (CBDT)?
1) M Ajit Kumar
2) Rajnish Kumar
3) P C Mody
4) Rahul Chaudhry
5) Sanjay Dutt
Answer-3) P C Mody
Explanation:
Pramod Chandra Modyis the current chairperson of Central Board of Direct Taxes (CBDT).

69. Where is the HQ of Economist Intelligence Unit (EIU) located?


1) Geneva
2) London
3) Paris
4) New York
5) New Delhi
Answer-2) London
Explanation:
The HQ of Economist Intelligence Unit (EIU) located at London, United Kingdom.

70. What is the capital and currency of Egypt?


1) Cairo & Pound
2) Jerusalem & Rial
3) Cairo & Rial
4) Beirut & Pound
5) Jerusalem & Dollar
Answer-1) Cairo & Pound
Explanation:
The capital and currency of Egypt are Cairo &Egyptian Pound respectively.

71. N. Biren Singh, the Chief Minister of which Indian state has inaugurated 5 Water Supply projects worth Rs
48.39 crore recently?
1) Manipur
2) Mizoram
3) Meghalaya
4) Maharashtra
5) Madhya Pradesh
Answer-1) Manipur
Explanation:
Manipur Chief Minister(CM) N. Biren Singh inaugurates 5 Water Supply projects worth Rs 48.39 crore, which have a
total capacity of more than 26.15 million of liters per day(MLD) through video conferencing from the Chief Minister’s
Secretariat, Imphal. With addition to the projects one office building of Weight and Measures Department and Rapid
Antigen Detection Testing Centre at Jiribam were also inaugurated. The New Development Bank has sanctioned Rs
3,000 crore to provide drinking water to every household of the state and work orders for the projects have been
issued.

Report Errors in the PDF - ebooks@affairscloud.com Copyright 2014-2020 @ AffairsCloud.Com 104


Banking & Economy PDF
72. Find the bank which renewed MoU with International Energy Agency (IEA) to increase the collaboration
on sustainability with a focus on energy sector.
1) Asian Infrastructure Investment Bank (AIIB)
2) New Development Bank (NDB)
3) Asian Development Bank (ADB)
4) World Bank (WB)
5) None of these
Answer-3) Asian Development Bank (ADB)
Explanation:
Asian Development Bank (ADB) has renewed a memorandum of understanding (MOU) with International Energy
Agency (IEA) during IEA’s Clean Energy Transitions Summit 2020 with the aim to further increase the collaboration
on sustainability with a focus on energy sector resilience in Asia and the Pacific.

73. Asian Development Bank has entered into agreement with Qatar-based Education Above All (EAA)
Foundation to work jointly for project worth of __________.
1) $100 million
2) $200 million
3) $150 million
4) $250 million
5) $500 million
Answer-1) $100 million
Explanation:
The Asian Development Bank (ADB) will help India and 9 other Asian nations to ensure quality education for the out-
of-school and at-risk children in the primary level. In its preliminary effort, ADB has entered into a 5-year agreement
with Qatar-based Education Above All (EAA) Foundation to work jointly for project worth of $100 million to put at
least 3.2 lakh out-of-school children, including refugees and internally displaced children into quality primary
education.

74. The IRDAI has removed the _______ maximum cap on sum insured for Arogya Sanjeevani policies recently.
1) Rs. 1 Lakh
2) Rs. 2 Lakh
3) Rs. 50,000
4) Rs. 25,000
5) Rs. 5 Lakh
Answer-5) Rs. 5 Lakh
Explanation:
The Insurance Regulatory and Development Authority of India (IRDAI) has allowed all general and health insurance
companies to increase the sum insured on the Standard Individual Health Insurance Product namely “Arogya
Sanjeevani” beyond Rs 5 lakh. This decision has been taken in view of the increasing cost of healthcare in the country
amid the covid-19 pandemic. Arogya Sanjeevani, is a product for which all insurers are mandated to offer had a cap of
Rs 5 lakh on the sum insured and the minimum cover for it was of Rs 1 lakh. The regulator has now allowed insurers
to offer sum insured as low as Rs 50,000 with no upper limit. The sum insured has to be in multiples of Rs 50,000.

75. Maharashtra government along with which bank planned to set up fund to revive Slum Rehabilitation
Authority projects?
1) Bank of Baroda
2) Indian Overseas Bank
3) Canara Bank
4) Punjab National Bank
5) State Bank of India

Report Errors in the PDF - ebooks@affairscloud.com Copyright 2014-2020 @ AffairsCloud.Com 105


Banking & Economy PDF
Answer-5) State Bank of India
Explanation:
Maharashtra Housing Minister Jitendra Awhad announced that the state govt and State Bank of India(SBI) will set up
a stress fund to revive Slum Rehabilitation Authority(SRA) projects(about 370 projects) in the Mumbai Metropolitan
Region (MMR) and other urban areas of the state.

76. How much was the additional amount Mastercard plans to invest to support Indian Small and Medium
Enterprises (SMEs)?
1) Rs 1000 crore
2) Rs 750 crore
3) Rs 500 crore
4) Rs 250 crore
5) Rs 100 crore
Answer-4) Rs 250 crore
Explanation:
Mastercard has announced that it will invest Rs 250 crore(USD 33 million) to support Indian Small and Medium
Enterprises(SMEs) to enable the SMEs to restart itself and help them in business recovery. SMEs have been hit
hardest during the lockdown caused by COVID-19 due to their dependence on the cash-economy and disruption of
the supply chain.

77. Who chairs the advisory committee constituted by SEBI on matters related to regulation and development
of the mutual funds?
1) M. S. Kamath
2) Motilal Oswal
3) Ashish Chauhan
4) Usha Thorat
5) Nitin Vyakaranam
Answer-4) Usha Thorat
Explanation:
Securities and Exchange Board of India (SEBI), the regulator of the securities and commodity market in India, has
reconstituted the committee advising it on matters related to regulation and development of the mutual fund
industry by including five new members. The new 20-member committee, will be chaired by Usha Thorat, former
deputy governor of Reserve Bank of India (RBI).

78. Which country is the largest Foreign Direct Investor in United Kingdom in 2019 (India ranked No.2)?
1) Germany
2) United states
3) France
4) Hong Kong
5) China
Answer-2) United states
Explanation:
According to the United Kingdom’s Department for International Trade (DIT) inward investment statistics for 2019-
2020, India has become the second largest Foreign Direct Investor in the United Kingdom (UK) after the United States
in 2019, by investing in 120 projects and creating 5,429 new jobs in the UK. The United States of America remains as
the top source of Foreign Direct Investment (FDI) of the UK, delivering 462 projects and 20,131 jobs. The List is
followed by India, Germany, France, China and Hong Kong.

Report Errors in the PDF - ebooks@affairscloud.com Copyright 2014-2020 @ AffairsCloud.Com 106


Banking & Economy PDF
79. Which state has signed MoU with HDFC bank to provide digital payment solutions to the state’s transport
department?
1) West Bengal
2) Karnataka
3) Odisha
4) Bihar
5) Jharkhand
Answer-5) Jharkhand
Explanation:
Jharkhand government signed Memorandum of Understanding(MoU) with HDFC Bank to provide digital payment
solutions to the state’s transport department.

80. Which Indian organization was conferred with ‘Development Marketplace Award 2020’ from World Bank
Group and Sexual Violence Research Initiative (SVRI)?
1) RAHI Foundation
2) Institute for Human Development
3) Indian Cancer Society
4) Citizens Foundation India
5) Indiability Foundation
Answer-2) Institute for Human Development
Explanation:
Institute for Human Development (IHD), an Indian non-profit institution conferred with ‘Development Marketplace
Award 2020:Innovations to Address Gender-Based Violence’ from World Bank Group and Sexual Violence Research
Initiative (SVRI) for the project alcohol, intimate partner violence and household economy: an assessment of liquor
ban in Bihar. Apart from India, other 8 non-profit organizations from various countries have received this award.

81. Which organization has presented India’s second Voluntary National Review (VNR) at the United Nations
High-level Political Forum on sustainable development 2020?
1) Finance Commission (FC)
2) Reserve Bank of India (RBI)
3) National Institution for Transforming India (NITI) Aayog
4) National Development Council (NDC)
5) None of these
Answer-3) National Institution for Transforming India (NITI) Aayog
Explanation:
NITI (National Institution for Transforming India) Aayog presented India’s second Voluntary National Review (VNR)
at the United Nations High-level Political Forum (HLPF) on Sustainable Development, 2020, which is being held
virtually from July 7, 2020 to July 16, 2020, under the aegis of the Economic and Social Council (ECOSOC). The theme
of HLPF is “Accelerated action and transformative pathways: realizing the decade of action and delivery for
sustainable development”. The India’s 2nd VNR 2020 report titled ‘Decade of Action: Taking SDGs from Global to
Local’.

82. Which bank partnered with Visa to deploy visa secure to the bank’s cardholders.
1) Bandhan Bank
2) Federal Bank
3) Dhanlaxmi Bank
4) IndusInd Bank
5) Catholic Syrian Bank
Answer-2) Federal Bank
Explanation:
Visa, enters into partnership with Federal Bank to deploy Visa Secure to the bank’s cardholders. Visa Secure will

Report Errors in the PDF - ebooks@affairscloud.com Copyright 2014-2020 @ AffairsCloud.Com 107


Banking & Economy PDF
provide additional authentication layers for issuers and merchants to combat online frauds. In addition to this, the
upgraded technology provides a quick and smooth checkout experience to consumers. This partnership will also
enhance the focus of the bank on customer-centric digital solutions that make online shopping experiences
seamless.Visa Secure is a global authentication program that uses the latest standards of the EMV (Europay,
Mastercard, and Visa) 3DS (3D Secure) protocol.

83. What will be India’s annual median Gross Domestic Product (GDP) growth in FY 21 as per Economic
Outlook Survey of FICCI?
1) (-) 4.5%
2) (-) 5%
3) (-) 3.5%
4) (-) 1.5%
5) (-) 5.5%
Answer-1) (-) 4.5%
Explanation:
In accordance with the latest round of Economic Outlook Survey of FICCI (Federation of Indian Chambers of
Commerce and Industry), India’s annual median Gross Domestic Product (GDP) growth forecast for FY2020-21 has
been declined to (-) 4.5% with a minimum and maximum growth estimate of (-) 6.4% and 1.5% respectively.As per
the survey, there is a possibility of V-shaped recovery with government expenditure as in the current situation.

84. Which country is the India’s top trading partner in 2019-20, according to the data of the Ministry of
Commerce and Industry?
1) United Kingdom
2) Mauritius
3) United States
4) Germany
5) China
Answer-3) United States
Explanation:
In accordance with the data of the Ministry of Commerce and Industry, the United States (US) remained India’s top
trading partner for the second consecutive fiscal in 2019-20 as bilateral trade between the US and India stood at USD
88.75 billion as against USD 87.96 billion in 2018-19.

85. How much percentage of Jio platforms’ stake was bought by Qualcomm Ventures for Rs. 730 Crores?
1) 0.15%
2) 0.45%
3) 0.73%
4) 1.21%
5) 2.42%
Answer-1) 0.15%
Explanation:
US-based Qualcomm Ventures, the investment arm of chipmaker Qualcomm Incorporated to invest upto Rs 730 crore
in Reliance Industries Limited(RIL)’s Jio Platforms for 0.15% stake. With this investment Jio Platforms has raised a
total of Rs 1,18,318.45 crore.

86. What is the capital and currency of Poland?


1) Berlin & Euro
2) Kyiv & Hryvnia
3) Warsaw & Zloty
4) Bratislava& Euro
5) Prague & Koruna

Report Errors in the PDF - ebooks@affairscloud.com Copyright 2014-2020 @ AffairsCloud.Com 108


Banking & Economy PDF
Answer-3) Warsaw & Zloty
Explanation:
The capital and currency of Poland are Warsaw and Polish Zloty respectively.

87. What is the constituency of Union minister of Commerce and Industry Piyush Goyal?
1) Gujarat (RS)
2) Maharashtra (RS)
3) Himachal Pradesh (RS)
4) Karnataka (RS)
5) New Delhi (RS)
Answer-2) Maharashtra (RS)
Explanation:
The Union minister of Commerce and Industry& Railways,Piyush Goyal is the Rajya Sabha MP from Maharashtra.

88. The ministry of shipping has approved Rs. 107 crores for modern firefighting facilities in Haldia Dock
complex of which port?
1) Mumbai Port
2) Chennai Port
3) Kandla Port
4) Kolkata Port
5) Mormugao Port
Answer-4) Kolkata Port
Explanation:
Mansukh Laxmanbhai Mandaviya, Union Minister of State (I/C) Shipping approved Rs.107 Crore to improve the
firefighting facilities on five jetties of Haldia Dock Complex of Kolkata Port. The modern firefighting facilities will
enable safe and secure handling of petrochemical goods in the Haldia Dock Complex.

89. How much percentage of world population went hungry in 2019 as per UN annual report titled “The state
of food security and nutrition in the world 2020”?
1) 7.9%
2) 8.9%
3) 8.2%
4) 6.7%
5) 10.5%
Answer-2) 8.9%
Explanation:
In accordance with the United Nations'(UN) annual report “The state of food security and nutrition in the world-
Transforming food systems for affordable healthy diets”, nearly 690 million people, or 8.9% of the world’s
population, went hungry in 2019, an increase of 10 million since 2018 and of nearly 60 million since 2014.The report
has been prepared by 5 UN agencies, Food and Agriculture Organization (FAO), International Fund for Agricultural
Development (IFAD), United Nations Children’s Fund (UNICEF), World Food Programme (WFP) and World Health
Organization (WHO).

90. The central government provides financial aid of Rs 1500 crore to which state for crop diversification and
rejuvenation of ponds (Rs 500 crore allocated for ‘MeraPani Meri Virasat’ scheme)?
1) Punjab
2) Haryana
3) Rajasthan
4) Gujarat
5) Maharashtra

Report Errors in the PDF - ebooks@affairscloud.com Copyright 2014-2020 @ AffairsCloud.Com 109


Banking & Economy PDF
Answer-2) Haryana
Explanation:
Central government provides financial aid of Rs 1500 crore to Haryana for water conservation, which includes Rs 500
crore to expand crop diversification under the ‘MeraPani Meri Virasat’ scheme and Rs 1000 crore to rejuvenate the
ponds and treat water.

91. Which edition of foundation day of NABARD was celebrated on July 12, 2020 (NABARD organized its 1st
“digital choupal” on July 13)?
1) 39th
2) 44th
3) 37th
4) 21st
5) 29th
Answer-1) 39th
Explanation:
On July 12, 2020, National Bank for Agriculture and Rural Development (NABARD) has celebrated its 39th
Foundation day. On July 13, 2020, NABARD has organized its first “digital choupal”, a video conference which brought
together farmers from across the country who have been associated with NABARD in 7 development projects in rural.

92. How much crore worth refinance scheme was announced by NABARD for banks and financial institutions
(July 2020)?
1) Rs 1,000 crore
2) Rs 3,700 crore
3) Rs 2,500 crore
4) Rs 5,000 crore
5) Rs 10,000 crore
Answer-4) Rs 5,000 crore
Explanation:
NABARD has announced Rs 5,000 crore of refinance scheme for banks and financial institutions for providing finance
to the beneficiaries of its 2,150 watershed development projects. The concessional line of assistance will be available
for three years from 2020-21 to 2022-23. Another Rs 5,000 crwas allocated for turning PACs into multi service
centres.

93. What is the rank of Mukesh Ambani in world’s richest person list, as per Bloomberg Billionaires Index
released on July 14, 2020 (Jeff Bezos tops)?
1) 6th
2) 5th
3) 3rd
4) 4th
5) 9th
Answer-1) 6th
Explanation:
Bloomberg Billionaires Index has ranked Mukesh Ambani, chairman of Reliance Industries Ltd(RIL) as the World’s
6th richest man with a worth USD 72.4 billion after surpassing Elon Musk, co-founder & CEO of Tesla and Alphabet
Inc co-founders, Sergey Brin and Larry Page. Note– The Bloomberg Billionaires Index is a daily ranking of the world’s
richest people.
Top 3 in Global wise
Rank Name Total Net Worth
1 Jeff Bezos(US) USD 184B

Report Errors in the PDF - ebooks@affairscloud.com Copyright 2014-2020 @ AffairsCloud.Com 110


Banking & Economy PDF

2 Bill Gates(US) USD 115B


3 Bernard Arnault(France) USD 94.5B
Other Indians in Top 100
77 Azim Premji USD 16.9B
89 Shiv Nadar USD15.8B

94. NABARD disbursed Rs 221.89 crore to support 298 watershed projects in which state?
1) Punjab
2) Haryana
3) Maharashtra
4) Karnataka
5) Uttar Pradesh
Answer-4) Karnataka
Explanation:
National Bank for Agriculture and Rural Development (NABARD) has disbursed Rs 2.51 crore of grant assistance to
Self Help Groups (SHGs) for promotion, skill development and digitization of records in Karnataka. This funding is a
part of eShakti or Digitization of SHGs an initiative of Micro Credit and Innovations Department (mCID) of
NABARD.Also, in Karnataka, NABARD has extended a grant assistance of Rs 221.89 crore to support 298 watershed
projects in over three lakh hectares benefitting 66,500 families.

95. The Central Board of Direct Taxes (CBDT) launched tool for banks and post offices to ascertain TDS
applicability rates on cash withdrawals. The TDS applicability rates on cash withdrawals of above _______ in
case of a non-filer of the income-tax return.
1) Rs. 10 Lakh
2) Rs. 20 Lakh
3) Rs. 5 Lakh
4) Rs. 1 Lakh
5) Rs. 15 Lakh
Answer-2) Rs. 20 Lakh
Explanation:
A new functionality of Income Tax Department namely “Verification of applicability u/s 194N” which is available on
www.incometaxindiaefiling.gov.in since July 1, 2020 has now made accessible for banks, co-operative societies, and
post offices through web-services by Central Board of Direct Taxes (CBDT). This feature will enable banks and post
offices to ascertain the TDS (Tax deduction at source) applicability rates on cash withdrawals of above Rs 20 lakh in
case of a non-filer of the income-tax return (ITR) and that of above Rs 1 crore in case of a filer of the ITR.

96. Name the person who has been appointed as the new vice-president of Asian Development Bank (ADB).
1) Ashok Lavasa
2) Anil Kishora
3) Gita Gopinath
4) Nirmala Sitharaman
5) Diwakar Gupta
Answer-1) Ashok Lavasa
Explanation:
The Asian Development Bank(ADB) has appointed Ashok Lavasa as its Vice-President for Private Sector Operations
and Public–Private Partnerships, who is currently serving as one of the Election Commissioners of India(since
January 2018). He will succeed Diwakar Gupta, whose term will end on 31 August, 2020.

Report Errors in the PDF - ebooks@affairscloud.com Copyright 2014-2020 @ AffairsCloud.Com 111


Banking & Economy PDF
97. What is the rank of Mukesh Ambani in world’s richest person list, as per Hurun’s research ?
1) 3rd
2) 6th
3) 5th
4) 8th
5) 7th
Answer-3) 5th
Explanation:
According to the Hurun’s Research’s World Richest Man List, Reliance Industries Limited (RIL) chairman & CEO
Mukesh Ambani (63) with worth 78 billion USD ranked 5th richest man in the world.He sustains as the wealthiest
Indian and he is the only Asian Tycoon in the world’s top 10 richest people as on July 2020.

98. What is the capital and currency of Chile?


1) Santiago& Peso
2) Buenos Aires & Peso
3) Sucre& Dollar
4) Quito & Euro
5) Lima & Dollar
Answer-1) Santiago & Peso
Explanation:
The capital and currency of Chile are Santiago& Peso respectively.

99. The Defence Acquisition Council (DAC) has granted special powers to the armed forces for capital
procurement worth Rs. 300 crore. Who chairs the DAC?
1) Nitin Gadkari
2) Nirmala Sitharaman
3) Amit Shah
4) Narendra Modi
5) Rajnath Singh
Answer-5) Rajnath Singh
Explanation:
The Defence Acquisition Council (DAC) headed by Rajnath Singh, Minister of Defence has granted special financial
power to the armed forces to make emergency purchases of armed weapons worth upto Rs.300 crores to meet the
emerging operational requirement.

100. Central government plans to increase public health expenditure to ______% of GDP by 2025.
1) 2.5
2) 1.25
3) 3
4) 5
5) 1
Answer-1) 2.5
Explanation:
In a meeting with the 15th Finance Commission the Union Health Minister Harsh Vardhan highlighted that the
government aims to gradually increase the public health expenditure to 2.5% of Gross Domestic Product(GDP) by
2025.The government under the Ayushman Bharat programme proposes to establish 1.5 lakh health and wellness
centers by 2022.

Report Errors in the PDF - ebooks@affairscloud.com Copyright 2014-2020 @ AffairsCloud.Com 112


Banking & Economy PDF
101. Name the country which opens new trade route with India to improve trade and connectivity (July
2020).
1) Pakistan
2) Bhutan
3) Sri Lanka
4) Nepal
5) Bangladesh
Answer-2) Bhutan
Explanation:
India and Bhutan opened a new trade route between Jaigaon in West Bengal and Ahlay, Pasakha in Bhutan to
establish easy connectivity and to increase trade between India and Bhutan amid the COVID-19 pandemic.

102. NABARD launched development projects worth Rs. 44 Lakhs in which state/UT (July 2020)?
1) Goa
2) Chhattisgarh
3) Assam
4) Andaman and Nicobar
5) Lakshadweep
Answer-4) Andaman and Nicobar
Explanation:
National Bank for Agriculture and Rural Development(NABARD) launched development projects of Rs 44 lakhs in
Andaman and Nicobar Islands to strengthen the Atmanirbhar Bharat through rural entrepreneurship. The
development projects include formation and development of Self-Help Groups(SHGs).

103. Name the person who chairs CII’s e-commerce committee for the year 2020-21
1) Kunal Bahl
2) Chandrajit Banerjee
3) Uday kotak
4) T. V. Narendran
5) Sangeeta Verma
Answer-1) Kunal Bahl
Explanation:
Snapdeal’sKunalBahl to lead CII’s e-commerce committee. Confederation of Indian Industry recently
announcedSnapdeal’s Co-founder & CEO KunalBahl has taken over as the Chairman of its e-commerce committee for
the year 2020-21.

104. What is the currency of Tunisia?


1) Pound
2) Dollar
3) Dinar
4) Riyal
5) Tomar
Answer-3) Dinar
Explanation:
The capital and currency of Tunisia are Tunis & Tunisian Dinar respectively.

105. NABARD sanctioned Rs. 795 cr to which state to reconstruct infrastructure which was damaged by
Cyclone Amphan?
1) Odisha
2) West Bengal
3) Telangana

Report Errors in the PDF - ebooks@affairscloud.com Copyright 2014-2020 @ AffairsCloud.Com 113


Banking & Economy PDF
4) Bihar
5) Andhra Pradesh
Answer-2) West Bengal
Explanation:
National Bank for Agriculture and Rural Development (NABARD) has sanctioned Rs 795 crore to West Bengal to
reconstruct infrastructure which was damaged by Cyclone Amphan (May) under Rural Infrastructure Development
Fund (RIDF).

106. What will be the contraction in India’s GDP in FY21 as per ICRA Limited?
1) 9.5%
2) 7.5%
3) 5.5%
4) 3.5%
5) 1.5%
Answer-1) 9.5%
Explanation:
ICRA Limited (formerly Investment Information and Credit Rating Agency) projects India’s Gross Domestic Product
(GDP) to contract by 9.5% (-9.5%) in FY21 from 5% earlier forecasted as ongoing lockdowns in some states have
affected the recovery seen in May and June 2020.

107. RBI approves reappointment of Shyam Srinivasan as the MD & CEO of which bank (with effect from Sept
23, 2020)?
1) IndusInd bank
2) Federal bank
3) RBL bank
4) Axis bank
5) YES bank
Answer-2) Federal bank
Explanation:
The Reserve Bank of India (RBI) approved the reappointment of Shyam Srinivasan as the managing Director and
Chief Executive officer of Federal bank Limited with effect from September 23, 2020. He will be working for 1 year till
Sep 22, 2021.

108. What is India’s rank in the Baseline scenario as per 2020 global manufacturing risk index report
released by Cushman & Wakefield (China tops)?
1) 8
2) 5
3) 12
4) 3
5) 28
Answer-4) 3
Explanation:
In accordance with the “Cushman & Wakefield’s 2020 Global Manufacturing Risk Index (MRI)” India ranks 3rd in the
Baseline Scenario,3rd in Cost Scenario & 30th in Risk Scenario. The Index ranks the countries in the above 3 Scenarios
based on most-suitable locations for global manufacturing among 48 countries in terms of cost, competitiveness and
operating conditions.

109. India ranked _____ in Data Science Domain in Global Skills Index 2020 released by “Coursera”.
1) 11
2) 91
3) 71

Report Errors in the PDF - ebooks@affairscloud.com Copyright 2014-2020 @ AffairsCloud.Com 114


Banking & Economy PDF
4) 51
5) 21
Answer-4) 51
Explanation:
The report of Global Skills Index 2020, an in depth view at the state of the skills across the world prepared by
“Coursera”, World’s largest online learning platform shows that India Surges ahead of the peers in the business and
technology skill sets because of the growing young population.In the Data Science Domain India secured the global
rank 51(lagging) and secured 12th rank in the Asia Pacific region. In the Business Domain India secured the global
rank 34 (emerging)and secured 8th rank in the Asia Pacific region. In the Technology Domain India secured the global
rank 40 (emerging) and secured 10th rank in the Asia Pacific region.

110. How many people in India moved out of multidimensional poverty over 10 years as per Global
Multidimensional Poverty Index 2020 released by UNDP & OPHI?
1) 273 million
2) 412 million
3) 315 million
4) 216 million
5) 174 million
Answer-1) 273 million
Explanation:
The United Nations’ “Global Multidimensional Poverty Index(GMPI) 2020: Charting pathways out of
multidimensional poverty: Achieving the SDGs” states that India records the largest reduction in the number of
People living in poverty as around 273 million people moved out of the multidimensional poverty over 10 years
(2005/2006 – 2015/2016). The report released by the Human Development Report Office of the United Nations
Development Programme (UNDP) and the Oxford Poverty and Human Development Initiative (OPHI) focuses on how
multidimensional poverty has declined.

111. According to world bank’s classifications by Income level 2020-21, India was classified under which
category?
1) Low-Income
2) Upper-Middle-Income
3) Lower-Middle-Income
4) High-Income
5) Lowest-Income
Answer-3) Lower-Middle-Income
Explanation:
According to the New World Bank country classifications by income level 2020-2021, India remains in the
classification, Lower-Middle-Income Economy. In total 218 countries have been included in this classification.The
World Bank classifies countries into four income groups namely: low income, lower-middle income, upper-middle
income, and high-income economies. The classifications are updated each year on July 1 and are based on Gross
National Income(GNI) per capita in current USD(using the Atlas method exchange rates) of the previous year.

112. India’s GDP to contract by ______% in FY21 as per the estimation of India Ratings and Research.
1) 4.9%
2) 3.7%
3) 7.2%
4) 9.5%
5) 5.3%
Answer-5) 5.3%
Explanation:
As per the research report of India Ratings and Research(Ind-Ra), Centre’s fiscal deficit is estimated to increase to

Report Errors in the PDF - ebooks@affairscloud.com Copyright 2014-2020 @ AffairsCloud.Com 115


Banking & Economy PDF
7.6% in FY21, which is twice the budget estimate as the country spends more to mitigate the impact of COVID-19
pandemic, facing a shortfall in revenues. The report projects India’s Gross Domestic Product(GDP) to contract by
5.3% in FY21.

113. What is the currency of South Africa?


1) Rand
2) Dinar
3) Franc
4) Riyal
5) Dollar
Answer-1) Rand
Explanation:
The capital and currency of South Africa are Pretoria (executive), Bloemfontein (Judicial), Cape town (legislative) &
South African rand.

114. How many MoUs worth Rs.10,399 was signed by Tamil Nadu government with industrial groups to
create employment opportunities in the state?
1) 9
2) 11
3) 12
4) 10
5) 8
Answer-5) 8
Explanation:
The Tamil Nadu government signed 8 Memorandums of Understandings (MoUs) worth Rs 10,399 crore with
companies to create employment opportunities for over 13,507 people. The MoUs were signed in the presence of
Chief Minister Edappadi K. Palaniswami in the Secretariat at Fort St George in Chennai.

115. Who has been appointed as the MD&CEO of SBI cards and payment services Ltd (with effect from August
1, 2020)?
1) Rajnish Kumar
2) Mahesh Kumar Sharma
3) Hardayal Prasad
4) Vijay Jasuja
5) Ashwini Kumar Tewari
Answer-5) Ashwini Kumar Tewari
Explanation:
SBI (State Bank of India) cards and payment services Ltd. appointed, Mr. Ashwini Kumar Tewari as its new Managing
Director (MD) and Chief Executive officer (CEO) who is intended to be in charge from August 1, 2020. His new
appointment will be valid for 2 years.

116. What is India’s rank in the global direct selling industry list, as per 2019 report released by World
Federation of Direct Selling Associations (WFDSA)?
1) 15th
2) 9th
3) 8th
4) 12th
5) 6th
Answer-1) 15th
Explanation:
In accordance with the report of “The Global Direct Selling – 2019 Retail Sales” by the World Federation of Direct

Report Errors in the PDF - ebooks@affairscloud.com Copyright 2014-2020 @ AffairsCloud.Com 116


Banking & Economy PDF
Selling Associations (WFDSA), India was ranked 15th in the global direct selling industry with sales of USD 2.477
billion in 2019. This has shown a growth of 12.1% in comparison to the 19th rank in 2018. The ranking has been
topped by the United States (US) which contributed 20% of the global direct selling industry with sales of USD 35.21
billion, followed by China with 13% contribution at 2nd rank, and Korea and Germany with 10% contribution each at
3rd position.
Rank Country
15 India
1 United States (US)
2 China
3 Korea & Germany

117. What is the currency of UAE?


1) Dinar
2) Tomar
3) Riyal
4) Dirham
5) Lek
Answer-4) Dirham
Explanation:
The capital and currency of UAE are Abu Dhabi & UAE Dirham respectively.

118. Which ministry partnered with UNICEF’s YuWaah initiative to mobilize 1 crore youth volunteers in India
to achieve the goals of Atmanirbhar Bharat?
1) Ministry of Skill Development and Entrepreneurship
2) Ministry of Youth Affairs and Sports
3) Ministry of Personnel, Public Grievances and Pensions
4) Ministry of Labour and Employment
5) Ministry of Tribal Affairs
Answer-2) Ministry of Youth Affairs and Sports
Explanation:
The Ministry of Youth Affairs and Sports(MYAS) signed the statement of intent with United Nations Children’s
Fund(UNICEF)’s multi stakeholder platform – YuWaah to mobilise 1 crore youth volunteers in India to achieve the
goals of Atmanirbhar Bharat and to promote the volunteerism among the youth in India.

119. Puducherry CM V Narayanasamy presented Rs 9000 crore tax free budget for the financial year 2020-
2021 recently. Who is the Lt. governor of Puducherry?
1) V.P. Singh Badnore
2) Dineshwar Sharma
3) Praful Patel
4) Kiran Bedi
5) Devendra Kumar Joshi
Answer-4) Kiran Bedi
Explanation:
Puducherry Chief Minister V Narayanasamy, who also holds the finance portfolio presented Rs 9000 crore tax free
budget for the financial year 2020-2021 in the assembly. The budget is Rs 575 crore more than the previous year’s
outlay of Rs 8,425 crore and Rs 1073 crore more than the revised budget outlay of Rs 7927 crore in 2019-2020. Note–
The budget was presented amid objections from Lt Governor Kiran Bedi and without her customary address.

Report Errors in the PDF - ebooks@affairscloud.com Copyright 2014-2020 @ AffairsCloud.Com 117


Banking & Economy PDF
120. The Consumer Protection Act (CPA), 2019 has come into effect from which date?
1) July 20, 2020
2) June 30, 2020
3) July 1, 2020
4) August 15, 2020
5) July 31, 2020
Answer-1) July 20, 2020
Explanation:
The Consumer Protection Act (CPA), 2019, which has replaced old Consumer Protection Act 1986 has come into force
w.e.f. July 20, 2020 aiming to enforce and protect the rights of consumers, and provide an effective mechanism to
address consumer grievances. The act was briefed by Union Minister for Consumer Affairs, Food & Public Distribution
Ram Vilas Paswan through video conference with media.The Act proposes the establishment of a Central Consumer
Protection Authority (CCPA) to promote, protect and enforce the rights of consumers, and make interventions in
situations of unfair trade practices.

121. Union Minister of Health and Family Welfare, Harsh Vardhan launched the CSR campaign (to prevent
spread of COVID-19) of which public sector bank recently?
1) Indian Overseas Bank
2) Central Bank of India
3) Punjab National Bank
4) UCO Bank
5) Bank of Maharashtra
Answer-3) Punjab National Bank
Explanation:
Union Minister of Health and Family Welfare, Harsh Vardhan launched Punjab National Bank(PNB)’s nationwide
Corporate Social Responsibility(CSR) campaign to prevent the spread of COVID-19 by distributing face masks and
sanitizers.

122. Which ministry signed MoU with Central Board of Direct Taxes (CBDT) for data sharing?
1) Ministry of Culture
2) Ministry of Skill Development and Entrepreneurship
3) Ministry of Human Resource Development
4) Ministry of Environment, Forest and Climate Change
5) Ministry of Micro, Small and Medium Enterprises
Answer-5) Ministry of Micro, Small and Medium Enterprises
Explanation:
Central Board of Direct Taxes(CBDT) and the Ministry of Micro, Small and Medium Enterprises(MoMSME) signed the
formal memorandum of understanding(MoU) for sharing of data related to income tax returns by the CBDT to the
Ministry of MSME.

123. The Central Board of Direct Taxes (CBDT) signed MoU with the Central Board of Indirect Taxes and
Customs (CBIC) for data sharing. Both the CBDT & CBIC functions under which ministry?
1) Ministry of Home Affairs
2) Ministry of Commerce
3) Ministry of Heavy Industries
4) Ministry of Corporate Affairs
5) Ministry of Finance
Answer-5) Ministry of Finance
Explanation:
Two tax policy bodies under the Finance Ministry: The Central Board of Direct Taxes (CBDT) and the Central Board of

Report Errors in the PDF - ebooks@affairscloud.com Copyright 2014-2020 @ AffairsCloud.Com 118


Banking & Economy PDF
Indirect Taxes and Customs (CBIC), signed A Memorandum of Understanding (MoU) for a smoother data exchange
between the two organizations.

124. Name the person who has been appointed as the MD&CEO of Karur Vysya Bank.
1) Arun Singhal
2) PVS Suryakumar
3) Shaji KV
4) P R Seshadri
5) Ramesh Babu Boddu
Answer-5) Ramesh Babu Boddu
Explanation:
Karur Vysya Bank (KVB)’s Board of Directors has co-opted Ramesh Babu Boddu as an Additional Director and
appointed him as the Managing Director(MD) & Chief Executive Officer(CEO) for three years’ tenure. He replaces P R
Seshadri, who resigned in January 2020 citing personal reasons.

125. Where is the HQ of Punjab National Bank located?


1) New Delhi
2) Mumbai
3) Kolkata
4) Amritsar
5) Bengaluru
Answer-1) New Delhi
Explanation:
The HQ of Punjab National Bank located at New Delhi.

126. According to International Trade Union Confederation (ITUC)’s 7th edition of “ITUC Global Rights Index
2020”, India ranked among 10 worst countries for working people. Where is the HQ of ITUC located?
1) New Delhi
2) Geneva
3) Brussels
4) Vienna
5) London
Answer-3) Brussels
Explanation:
In accordance with the 7th edition of “International Trade Union Confederation (ITUC) Global Rights Index 2020-The
World’s Worst Countries for Workers”, India became the new entrant in the 10 worst countries for working people in
the world, with a rating of 5 that states “No guarantee of rights”. The index ranked 144 countries on the basis of
respect for workers’ rights.The other countries among the worst ten are- Bangladesh, Brazil, Colombia, Kazakhstan,
Philippines, Turkey and Zimbabwe along with Egypt & Honduras being the other two new entrants apart from
India.The headquarters of International Trade Union Confederation (ITUC) located at Brussels, Belgium.

127. Name the public sector bank which signed corporate agency pact with Life Insurance Corporation of
India.
1) Union Bank of India
2) Bank of Baroda
3) Oriental Bank of Commerce
4) Bank of Maharashtra
5) State Bank of India
Answer-1) Union Bank of India
Explanation:
Union Bank of India (UBI), after its amalgamation with Corporation Bank and Andhra Bank, has announced a

Report Errors in the PDF - ebooks@affairscloud.com Copyright 2014-2020 @ AffairsCloud.Com 119


Banking & Economy PDF
corporate agency agreement with Life Insurance Corporation of India (LIC) to distribute latter’s policies. As per the
pact, UBI can act as a financial supermarket by offering LIC products of term insurance, pension, plan, children plan,
ULIPs, and endowment schemes to its account holders, whereby they can select products according to their choice.

128. Fampay partnered with which bank to launch India’s 1st numberless card ‘FamCard’?
1) Nainital Bank
2) ICICI Bank
3) IDFC First Bank
4) Kotak Mahindra Bank
5) IDBI Bank
Answer-3) IDFC First Bank
Explanation:
FamPay, India’s first Neobank for teenagers, in partnership with IDFC First Bank launched India’s 1st numberless
card, FamCard. The FamPay with National Payments Corporation of India(NPCI) developed this card. This card is
accepted across the RuPay payment network of merchants. It is similar to a debit card which enables teenagers to
transact in a cashless manner.

129. India’s GDP to contract by _______ in FY21 as per estimation of Nomura.


1) 6.1%
2) 4.2%
3) 5.3%
4) 7.8%
5) 11.2%
Answer-1) 6.1%
Explanation:
As per the report of Nomura Holdings, Inc, a Japanese brokerage firm, India’s Gross Domestic Product(GDP) is likely
to contract by 6.1%(-6.1%) in FY21 as economic activity continues to be weak due to COVID-19 pandemic.

130. Who has been appointed as the MD&CEO of SBI general insurance company?
1) Prakash Chandra Kandpal
2) Pushan Mahapatra
3) Devesh Srivastava
4) V Ramasamy
5) A K Das
Answer-1) Prakash Chandra Kandpal
Explanation:
Non-life insurer SBI General Insurance Company Limited announced that it has appointed Prakash Chandra(P C)
Kandpal as its Managing Director(MD) and Chief Executive Officer(CEO), effective from July 20. He succeeds Pushan
Mahapatra, who has been appointed as a director (strategic investments & digital initiatives), effective from 18th July,
2020.

131. Who has been appointed as the chairman of Bajaj Finance (with effect from August 1, 2020)?
1) Jamnalal Bajaj
2) Rahul Bajaj
3) Sanjiv Bajaj
4) Rajiv Bajaj
5) None of the Above
Answer-3) Sanjiv Bajaj
Explanation:
Rahul Bajaj, the present non-executive chairman of Bajaj Finance has decided to step down on July 31, 2020 and he
will be replaced by his son Sanjiv Bajaj (the present Vice Chairman of the company) from August 1, 2020.

Report Errors in the PDF - ebooks@affairscloud.com Copyright 2014-2020 @ AffairsCloud.Com 120


Banking & Economy PDF
132. Who is the MD&CEO of Union Bank of India (UBI)?
1) G. Padmanabhan
2) Padmaja Chunduru
3) Pallav Mahapatra
4) T. C. A. Ranganathan
5) Rajkiran Rai G
Answer-5) Rajkiran Rai G
Explanation:
Rajkiran Rai G is the present Managing Director (MD) & Chief Executive Officer (CEO) of Union Bank of India (UBI).

133. Which country became the 25th country to receive observer status in the World Trade Organization
(WTO)?
1) Afghanistan
2) Iran
3) Kazakhstan
4) Turkmenistan
5) Uzbekistan
Answer-4) Turkmenistan
Explanation:
Turkmenistan became the 25th country who was granted observer status in the World Trade Organization (WTO)
enabling the nation to establish permanent residence at WTO headquarter in Geneva, Switzerland and monitor its
proceedings. Turkmenistan also became the last former Soviet republic to establish formal ties with the trade body.
This observer status was granted during the 5th virtual session of WTO General Council which was held from WTO
headquarters and attended by representatives of its member-states.

134. Which payments bank partnered with National Skill Development Corporation (NSDC) to train and skill
the youth in rural India for financial service sector jobs?
1) Airtel Payments Bank
2) India Post Payments Bank
3) Paytm Payments Bank
4) Fino Payments Bank
5) Jio Payments Bank
Answer-1) Airtel Payments Bank
Explanation:
Airtel Payments Bank Ltd has partnered with National Skill Development Corporation (NSDC) to train and skill the
youth in rural India. This will enable them to find employment opportunities and become entrepreneurs in the
financial services sector. The knowledge and training imparted to youth is to render financial and banking services
more accessible to rural and semi-urban India.

135. Which bank launched 100% paperless digital self-assisted online savings account named ‘Insta Click
Savings Account’?
1) ICICI Bank
2) Bank of Baroda
3) Union Bank of India
4) HDFC Bank
5) State Bank of India
Answer-2) Bank of Baroda
Explanation:
India’s third-largest public sector bank, Bank of Baroda(BoB) launched ‘Insta Click Savings Account’, a 100%
paperless digital self-assisted online savings account. It can be activated in real-time i.e; the customer can transact
using the Baroda M Connect Plus application with the MPIN received on the mobile number.

Report Errors in the PDF - ebooks@affairscloud.com Copyright 2014-2020 @ AffairsCloud.Com 121


Banking & Economy PDF
136. What will be the GDP of India in FY21 as per the estimation of Singapore based DBS Bank?
1) -7%
2) -4%
3) -5%
4) -6%
5) -9%
Answer-4) -6%
Explanation:
As per the report of DBS (Formerly, Development Bank of Singapore), Singaporean brokerage, India’s Gross Domestic
Product (GDP) will contract by 6% (-6%) in FY21 as the economy is yet to stabilize the COVID -19 infection curve and
due to the caseload in the economically key states. This will affect the economy badly. In its earlier forecast, the
brokerage had forecasted the growth as -4.8%.

137. Which form was revised by Central Board of Direct Taxes (CBDT) to enhance the flow of information
between taxpayers and tax authorities (July 2020)?
1) Form 26AS
2) Form 15IC
3) Form 10BA
4) Form 44AB
5) Form 16
Answer-1) Form 26AS
Explanation:
In order to enhance the flow of information between taxpayers and tax authorities, the Central Board of Direct Taxes
(CBDT) has launched a revised auto-generated Form 26AS namely “Annual Information Statement (AIS)” from this
assessment year of 2020-2021 w.e.f. June 1. The key difference between the new and old form is that the former will
also reflect the details of “specified financial transactions (SFT)” i.e. all high-value transactions of individuals,
companies, partnership firms, etc. which are filed with the tax department under Section 285BA of the Income-tax
Act, 1961.

138. Which is the 1st Indian company to reach the market Capitalization of Rs 13 lakh crore (48th in world)?
1) Reliance Industries Limited
2) HDFC Bank
3) Bharti Airtel
4) Tata Consultancy Services
5) Hindustan Unilever Limited
Answer-1) Reliance Industries Limited
Explanation:
As stock market data, Reliance Industries Limited (RIL) becomes the 1st Indian company to reach Market
Capitalisation (m-cap) of Rs 13 lakh crore. It becomes the world’s 48th largest company by m-cap, ahead of Abbott
Labs, Oracle, Chevron and Unilever. It also crossed the stock price of Rs 2000 for the 1st time. Globally, Saudi Aramco
is the company with the highest market cap of $1.7 trillion, followed by Apple, Microsoft, Amazon, and Alphabet.
Reliance is the 10th highest m-cap company in Asia. China’s Alibaba Group is ranked 7th globally.

139. As per Forbes real-time ranking of billionaires, what is the rank of Mukesh Ambani ($75 billion wealth)
in world’s richest man list (July 2020)?
1) 2nd
2) 8th
3) 5th
4) 7th
5) 9th

Report Errors in the PDF - ebooks@affairscloud.com Copyright 2014-2020 @ AffairsCloud.Com 122


Banking & Economy PDF
Answer-3) 5th
Explanation:
Reliance Industries Limited (RIL) Chairman & CEO Mukesh Ambani (63), became the world’s fifth-richest man in the
world with an estimated wealth of 75 billion USD, according to the real-time ranking of billionaires by Forbes.

140. India targeted to reach _____ spot in World Bank’s (WB) Ease of Doing Business ranking?
1) 75th
2) 60th
3) 50th
4) 80th
5) 100th
Answer-3) 50th
Explanation:
In October 2019, India jumped 14 places to rank 63rd among 190 nations in the World Bank’s (WB) Ease of Doing
Business ranking. Now, the target of India is to reach at the 50th spot.

141. Who represented India in G20 Digital Economy Ministers virtual meet held in July 2020?
1) Piyush Goyal
2) Raj Kumar Singh
3) Ravi Shankar Prasad
4) Giriraj Singh
5) Pralhad Joshi
Answer-3) Ravi Shankar Prasad
Explanation:
Union Minister of Electronics and Information Technology, Shri Ravi Shankar Prasad represented India in the virtual
meeting of the G20 Digital Economy Ministers under the presidency Saudi Arabia.

142. What is the amount approved for 81 projects under Special Window for Affordable and Mid Income
Housing (SWAMIH) Investment Fund I (July 2020)?
1) Rs 8776 crore
2) Rs 8112 crore
3) Rs 8767 crore
4) Rs 9357 crore
5) Rs 4527 crore
Answer-3) Rs 8767 crore
Explanation:
Union Minister for Finance & Corporate Affairs, Nirmala Sitharaman informed that so far Rs 8767 crore has been
approved for 81 projects under Special Window for Affordable and Mid Income Housing (SWAMIH) Investment Fund
I. The approved projects will enable the completion of almost 60,000 homes across India. The announcement was
made when the performance of SWAMIH was reviewed with Secretaries of Ministries of Finance and senior
management team of the State Bank of India, SBI Capital Markets Limited and SBICAPS Ventures Limited (SVL).

143. Which state government approved USD 285.71 million project for canal-based water supply (70% of
project fund will be offered by World Bank)?
1) Gujarat
2) Haryana
3) Maharashtra
4) Punjab
5) Madhya Pradesh
Answer-4) Punjab

Report Errors in the PDF - ebooks@affairscloud.com Copyright 2014-2020 @ AffairsCloud.Com 123


Banking & Economy PDF
Explanation:
The Punjab Cabinet chaired by Chief Minister Amarinder Singh approved a USD 285.71 million (over Rs 2,130 crore)
project for canal-based water supply for Amritsar and Ludhiana cities through video conferencing. The funding of
these projects will be offered by the World Bank (70%) and Punjab gov (30%). In addition to this, the cabinet also
approved an amendment to the Land Pooling Policy.

144. Which bank joined hands with Green Climate Fund (GCF) towards ‘green recovery’ on the impacts of
COVID-19 disease?
1) Asian Infrastructure Investment Bank (AIIB)
2) New Development Bank (NDB)
3) Asian Development Bank (ADB)
4) World Bank (WB)
5) None of these
Answer-3) Asian Development Bank (ADB)
Explanation:
The Asian Development Bank (ADB) and the Green Climate Fund (GCF) agreed to partner toward the “Green
Recovery”. This is to help hands for the people who are facing the harsh economic impact due to COVID-19 pandemic.
They agreed to strengthen collaboration on non-sovereign operations while ensuring GCF’s processes are as flexible
as possible to foster at-scale financing. ABD has offered a support package of USD 20 billion for COVID-19.

145. The Reserve Bank of India publicized the “Report of the Committee for Analysis of QR Code” recently.
Who chaired the committee?
1) Deepak B. Phatak
2) Arvind Kumar
3) Sunil Mehta
4) AS Ramasastri
5) Dilip Asbe
Answer-1) Deepak B. Phatak
Explanation:
Reserve Bank of India (RBI) publicized the “Report of the Committee for Analysis of QR (Quick Response) Code”
under the chairmanship of Prof. Deepak B. Phatak on its website for comments / suggestions of various industry
players currently involved in or associated with the use of QR codes before August 10, 2020.

146. Which feature of United Payments Interface (UPI) was launched by National Payments Corporation of
India (NPCI) at Global Fintech Festival 2020.
1) UPI Scan
2) UPI Cash
3) UPI AutoPay
4) UPI Transfer
5) None of the Above
Answer-3) UPI AutoPay
Explanation:
In order to ease online recurring payments in India, NPCI introduced a Unified Payments Interface (UPI) AutoPay
feature. It is a one-stop fintech payment solution that allows customers to create e-mandate through their UPI ID or
QR Scan. For transactions less than Rs. 2,000, customers have to authenticate their account through UPI PIN for one-
time and subsequent monthly payments would be debited automatically. For an amount greater than Rs. 2,000, a UPI
PIN authentication will be required for every mandate.

Report Errors in the PDF - ebooks@affairscloud.com Copyright 2014-2020 @ AffairsCloud.Com 124


Banking & Economy PDF
147. Which organization partnered with State Bank of Mauritius, Enkash, YAP & Rupay to launch commercial
card named SBM EnKash RuPay business card?
1) NPCI
2) SWITCH
3) AePS
4) IFSC
5) UPI
Answer-1) NPCI
Explanation:
National Payments Corporation of India (NPCI) launched RuPay commercial card named SBM EnKash RuPay
Business Card in partnership with State Bank of Mauritius (SBM) India, EnKash, YAP (API- Application Programing
Interface provider) and RuPay for young entrepreneurs and small business owners. The card will be disbursed
through SBM touchpoints.

148. Amazon pay partnered with which general insurance company to offer 100% paperless auto insurance?
1) Acko General Insurance
2) Bajaj Allianz General Insurance Company
3) HDFC ERGO General Insurance Company
4) SBI General Insurance
5) Tata AIG General Insurance
Answer-1) Acko General Insurance
Explanation:
Amazon Pay, the payments arm of Amazon India has partnered with Acko General Insurance Ltd to offer auto
insurance for two and four-wheelers in India, a 100% paperless insurance plan. Customers can avail this auto
insurance from Amazon Pay page, Amazon app or mobile website and can get a quote for their car or bike’s insurance
by providing basic details.

149. ICICI Lombard has partnered with which digital wallet company to offer affordable hospital daily cash
benefit?
1) BHIM
2) Jio Money
3) Amazon Pay
4) PayTM
5) PhonePe
Answer-5) PhonePe
Explanation:
ICICI Lombard in partnership with PhonePe has launched Hospital Daily cash benefit under its Group Safeguard
Insurance policy, a sachet-based group insurance product. It is a customized hospitalization policy that allows
PhonePe users to get an assured amount if they are hospitalized due to injury or illnesses (including COVID-19).

150. Who has been appointed as the MD&CEO of Indian Overseas Bank (IOB)?
1) Partha Pratim Sengupta
2) Subramanian Sundar
3) Karnam Sekar
4) M Chidambaram
5) T. C. A Ranganathan
Answer-1) Partha Pratim Sengupta
Explanation:
Indian Overseas Bank (IOB) has appointed Mr. Partha Pratim Sengupta as its new Managing Director (MD) and Chief
Executive Officer (CEO), and his tenure ends by December 31, 2022 or until further orders. He succeeds Karnam
Sekar who was retired on June 30, 2020.

Report Errors in the PDF - ebooks@affairscloud.com Copyright 2014-2020 @ AffairsCloud.Com 125


Banking & Economy PDF
151. How much crore worth fund was created by Department of Science and Technology to support India –
Russia collaboration joint R&D for technology development?
1) 25 Crores
2) 20 Crores
3) 15 Crores
4) 12 Crores
5) 10 Crores
Answer-3) 15 Crores
Explanation:
The Department of Science and Technology (DST) under the Ministry of Science and Technology launched a India-
Russia Joint Technology Assessment and Accelerated Commercialization Programme in partnership with Russia’s
Foundation for Assistance to Small Innovative Enterprises (FASIE). On the Indian side, the programme will be
implemented & funded by Federation of Indian Chambers of Commerce & Industry (FICCI) on behalf of the DST. The
programme will run through two annual cycles and top five selected Indian and Russian partners will be provided
partial funding support. DST will fund up to Rs 15 Crores to Indian SMEs/Start-ups and FASIE will provide similar
funding to the Russian projects.

152. The central government amended General Financial Rules (GFRs) 2017 to impose restrictions on public
procurement from countries sharing land border with India. How many countries shares land border with
India?
1) 9
2) 8
3) 7
4) 6
5) 5
Answer-3) 7
Explanation:
Government of India (GoI) has amended the Rule 144 of General Financial Rules (GFRs) 2017 entitled “Fundamental
principles of public buying” by inserting sub rule (ix) which has imposed restrictions on bidders of countries sharing
land borders with India in public procurement tenders. This decision has been taken to strengthen the defence and
national security of India. The order in this regard has been issued by the Department of Expenditure (DoE), Ministry
of Finance. It should be noted that India shares its land border with Afghanistan, China, Nepal, Bhutan, Pakistan,
Bangladesh and Myanmar (7 countries).

153. Which organization along with International Energy Agency (IEA) released ‘Cooling Emissions and
Policy Synthesis Report’?
1) World Bank (WB)
2) International Labour Organization (ILO)
3) United Nations Industrial Development Organization (UNIDO)
4) United Nations Environment Programme (UNEP)
5) United Nations Educational, Scientific and Cultural Organization (UNESCO)
Answer-4) United Nations Environment Programme (UNEP)
Explanation:
The Cooling Emissions and Policy Synthesis Report from the United Nations Environment Programme (UNEP) and
the International Energy Agency (IEA) states that, by combining energy efficiency improvements with the transition
away from super-polluting refrigerants, the world can avoid cumulative greenhouse gas emissions of between 210-
460 gigatonnes of carbon dioxide equivalent(GtCO2e) over the next four decades. This equals 4-8 years(roughly) of
total annual global greenhouse gas emissions, based on 2018 levels. An estimated 3.6 billion cooling appliances are in
use worldwide. If cooling is provided to everybody who needs it, this will require about 14 billion cooling appliances
by 2050. The report was authored by a range of experts under the guidance of a 15-member steering committee co-

Report Errors in the PDF - ebooks@affairscloud.com Copyright 2014-2020 @ AffairsCloud.Com 126


Banking & Economy PDF
chaired by Mario Molina- President, Centro Mario Molina, Mexico, and Durwood Zaelke- President, Institute for
Governance & Sustainable Development, USA.

154. The Reserve Bank of India signed USD 400 million currency swap facility for which country?
1) Bhutan
2) Bangladesh
3) Nepal
4) Maldives
5) Sri Lanka
Answer-5) Sri Lanka
Explanation:
Reserve Bank of India (RBI) has agreed to a USD 400 million currency swap facility for Sri Lanka, valid till November
2022. This will help Sri Lanka to meet its short-term international liquidity requirements and to boost its foreign
exchange reserves following the economic crisis resulting from COVID-19.

155. As per RBI financial stability report, Gross NPA of bank may rise to _____ by March 2021.
1) 6.25%
2) 14.75%
3) 8.5%
4) 12.5%
5) 5.5%
Answer-4) 12.5%
Explanation:
In accordance with the 21st issue of Financial Stability Report (FSR), July 2020 by Reserve Bank of India (RBI)
released on July 24, 2020, the gross nonperforming assets (GNPA) ratio of all scheduled commercial banks (SCBs)
may increase from 8.5% in March 2020 to 12.5% by March 2021 under the baseline scenario; the ratio may escalate
to 14.7% under a very severely stressed scenario.

156. How many MoUs were signed by Tamil Nadu government to create employment opportunities for over
6,555 people worth Rs 5,137 crores?
1) 14
2) 16
3) 15
4) 12
5) 13
Answer-2) 16
Explanation:
The Tamil Nadu (TN) government has signed 16 Memorandum of Understanding (MoUs) worth Rs 5,137 crore to
create employment opportunities for over 6,555 people. The MoUs were signed by TN Chief Minister Edappadi K.
Palaniswami. The government has signed three tranches of MoUs worth Rs 30,664 crore within 2 months, which will
provide employment to over 67,212 persons.

157. What is the currency of North Korea?


1) Dollar
2) Yen
3) Won
4) Dinar
5) None of the Above
Answer-3) Won
Explanation:
The capital and currency of North Korea are Pyongyang & North Korean won respectively.

Report Errors in the PDF - ebooks@affairscloud.com Copyright 2014-2020 @ AffairsCloud.Com 127


Banking & Economy PDF
158. What is the currency of Sri Lanka?
1) Yen
2) Won
3) Rupee
4) Dollar
5) Rufiyaa
Answer-3) Rupee
Explanation:
The capital and currency of Sri Lanka are; Capital – Colombo, Sri Jayawardenepura Kotte & Currency– Sri Lankan
Rupee.

159. The Karnataka cabinet approved the new industrial policy for 2020-2025 recently. The state will hold
____% shares in companies under Karnataka digital economy mission.
1) 51%
2) 49%
3) 25%
4) 75%
5) 10%
Answer-2) 49%
Explanation:
Karnataka Cabinet approved the new Industrial Policy for 2020-2025 that aims to attract Rs 5 lakh crore investment
and create about 20 lakh employment over the next 5 years to help mitigate the economic impact of the COVID-19
crisis. The cabinet also approved, to set up the Karnataka Digital Economy Mission under section 8 of the Companies
Act, 2013 to promote 7,000 start-ups, e-commerce and other gig economy companies. 49% of the share will be held
by the state and remaining 51% by the industries.

160. Which public sector bank partnered with Uniken to secure its Internet banking channels?
1) Punjab National Bank
2) Bank of Maharashtra
3) Union Bank of India
4) Bank of Baroda
5) State Bank of India
Answer-2) Bank of Maharashtra
Explanation:
The state-owned Bank of Maharashtra has joined hands with Uniken, a pioneer in the field of digital security in India
to secure its Internet banking channels on mobile, card, call centers and ATMs, among others, against cyber fraud. In
this regard, the bank will use Uniken’s REL-ID technology for Omni-channel security for its online banking app
MahaSecure.

161. The Croatian parliament approved the new government led by Prime Minister Andrej Plenkovic
recently. What is the capital and currency of Croatia?
1) Tripoli & Dinar
2) Buenos Aires & Peso
3) Zagreb & Kuna
4) Mogadishu & Shilling
5) None of the Above
Answer-3) Zagreb & Kuna
Explanation:
The Croatian parliament approved the new government led by Prime Minister Andrej Plenkovic whose Croatian
Democratic Union (HDZ) party won the most parliamentary seats in the general election held on July 5, 2020. The
capital and currency of Croatia are Zagreb & Croatian kuna respectively.

Report Errors in the PDF - ebooks@affairscloud.com Copyright 2014-2020 @ AffairsCloud.Com 128


Banking & Economy PDF
162. Which edition of India-EU Joint Economic and Trade Committee (JETCO) meeting 2020 held recently?
1) 14th
2) 15th
3) 13th
4) 17th
5) 20th
Answer-1) 14th
Explanation:
The 14th Joint Economic and Trade Committee (JETCO) meeting 2020 was held virtually for the first time amid
COVID-19 between India and United Kingdom (UK), which was co-chaired by Minister for Commerce and Industry
Piyush Vedprakash Goyal and UK Secretary of State for International Trade, Elizabeth Mary Truss. They were assisted
by Minister of State (MoS) for Commerce and Industry Hardeep Singh Puri and his counterpart, UK Minister of State
for International Trade, Ranil Jayawardena.

163. When was the Income Tax day or Aaykar Diwas observed every year?
1) July 31
2) July 24
3) July 22
4) July 27
5) July 20
Answer-2) July 24
Explanation:
24th July 2020 is marked as the 160th Income Tax day or Aaykar Diwas by the Income Tax Department. The Income
Tax Day has been celebrated in India on 24th July since 2010. In 2010, The Income Tax Department declared July
24th as the annual Income Tax Day to mark the 150th year of this levy, the income tax was first imposed as a duty and
came into force on 24th July 1860.

164. As per the report titled, “Implications of AI on the Indian Economy”, released by NASSCOM along with the
Indian Council for Research on International Economic Relations (ICRIER) and Google, a unit increase in
Artificial Intelligence (AI) intensity can boost India’s GDP by ______%.
1) 1.5%
2) 2.25%
3) 2.5%
4) 1.75%
5) 2.75%
Answer-3) 2.5%
Explanation:
According to The report titled, ’Implications of AI on the Indian Economy’ by National Association of Software and
Services Companies (NASSCOM) along with the Indian Council for Research on International Economic Relations
(ICRIER) and Google, a unit increase in Artificial Intelligence(AI) intensity (measured as the ratio of AI to total sales in
the firm) can result in a 2.5% increase in India’s Gross Domestic Product(GDP) in the immediate term.

165. What is the outlay of the scheme for promotion of bulk drug parks approved by the Government of India
to create 3 drug parks from FY21 to FY25?
1) 1,000 crores
2) 3,000 crores
3) 2,000 crores
4) 1,500 crores
5) 2,500 crores
Answer-2) 3,000 crores

Report Errors in the PDF - ebooks@affairscloud.com Copyright 2014-2020 @ AffairsCloud.Com 129


Banking & Economy PDF
Explanation:
Union Minister for Chemicals and Fertilizers, Devaragunda Venkappa Sadananda Gowda launched guidelines for four
schemes of Department of Pharmaceuticals (under ministry of chemical & fertilizers), two each for Bulk Drugs and
Medical Devices parks, on the lines of making India Atma Nirbhar (self-reliant) in pharma sector. The scheme for
Promotion of Bulk Drug Parks, which has been approved by the Government of India (GoI) on March, 2020 was
launched with an outlay of Rs. 3,000 crores for a period of 4 years i.e. from FY2020-2021 to FY2024-2025 to create 3
bulk drug parks in India. Also, the scheme for Promotion of Medical Devices Park, which has been approved by the
GoI on March, 20, 2020 was launched with an outlay of Rs. 400 Crore for a period of 4 years. In addition to that
Production Linked Incentive (PLI) Scheme for Promoting Domestic Manufacturing of Medical Devices with the Outlay
of Rs. 3,420 cr & PLI scheme for promotion of domestic manufacturing of KSMs, DIs, & APIs in India with the Outlay of
Rs 6,490 cr.

166. IRCTC and SBI Card launched a co-branded contactless credit card on RuPay platform recently. Where is
the HQ of SBI card located?
1) Bengaluru
2) New Delhi
3) Kolkata
4) Gurugram
5) Mumbai
Answer-4) Gurugram
Explanation:
Indian Railway Catering and Tourism Corporation Limited (IRCTC) and SBI Card launched a co-branded contactless
credit card, IRCTC -SBI Platinum Card on RuPay Platform to reward the frequent railway (train) travelers. The card is
equipped with Near Field Communication (NFC) technology by which users can facilitate their transactions at the
point of sale(POS) machines by Tap and Pay Option. This enables consumers to transact safely and securely online.
SBI Card: Headquarters– Gurugram, Haryana.

167. Which bank provided liquidity support of USD 16.20 million as a COVID-relief for the Government of
Maldives (On occasion of Maldives’ independence day on July 26, 2020)?
1) Union Bank of India
2) Bank of Baroda
3) State Bank of India
4) Bank of Maharashtra
5) Punjab National Bank
Answer-3) State Bank of India
Explanation:
On the occasion of Independence Day of the Maldives on July 26, 2020, the largest Indian public sector bank, State
Bank of India (SBI) provided liquidity support of USD 16.20 million as a COVID-relief for the Government of Maldives
to overcome liquidity shortage. The fund will support economic relief measures by utilizing it for local businesses and
defers loan repayment for over 200 retail accounts.

168. Who has been appointed as the MD&CEO of Tata AIA Life Insurance company?
1) Manoj Kumar
2) Rishi Srivastava
3) Naveen Tahilyani
4) Anil Kumar Jha
5) None of the Above
Answer-3) Naveen Tahilyani
Explanation:
Naveen Tahilyani was appointed as the new Managing Director (MD) and Chief Executive officer (CEO) of the Tata
American International Assurance (AIA) life insurance company. His appointment was under regulatory approval

Report Errors in the PDF - ebooks@affairscloud.com Copyright 2014-2020 @ AffairsCloud.Com 130


Banking & Economy PDF
from Insurance Regulatory and Development Authority of India (IRDAI). Tahilyani succeeds Rishi Srivastava who has
been moved to the Group Agency Distribution, AIA Group, Hong Kong, as CEO.

169. Government of India has handed over symbolic cheque of USD 5.6 million for High Impact Community
Development Projects (HICDP) to which country (July 2020)?
1) Sri Lanka
2) Australia
3) Nepal
4) Cyprus
5) Maldives
Answer-5) Maldives
Explanation:
Financial Assistance for High Impact Community Development Projects (HICDP) from the Government of India was
handed over to the Government of Maldives in the form of a symbolic cheque of USD 5.6 million. The cheque was
handed over by the High Commissioner of India to the Maldives Sunjay Sudhir to the Minister of Foreign Affairs
Abdulla Shahid. In this regard, Government of Maldives launched nine community-based development projects.

170. Who has been re-elected for 2nd term as the president of Asian Infrastructure Investment Bank (AIIB)
(with effect from Jan 16, 2021)?
1) Andrey Kruglov
2) Adel AlHosani
3) Muhammad Aslam
4) Dinanath Dubhashi
5) Jin Liqun
Answer-5) Jin Liqun
Explanation:
During the 5th annual meeting of Board of Governors of Asian Infrastructure Investment Bank (AIIB), Jin Liqun was
elected President of AIIB for a second five-year term commencing January 16, 2021. The election result was
announced by chair of AIIB’s Board of Governors, Liu Kun, who is a Minister of Finance, People’s Republic of China.
Under President Jin’s leadership, AIIB has grown from 57 founding members to 103 approved members from around
the world.

171. How much amount was approved by Asian Development Bank (ADB) to India to support COVID-19
response (Financed by Japan)?
1) USD 5 Million
2) USD 3 Million
3) USD 1 Million
4) USD 15 Million
5) USD 10 Million
Answer-2) USD 3 Million
Explanation:
The Asian Development Bank (ADB) approved the 3 Million USD grant (about Rs.22 Crore) from the ADB’s Asia
Pacific Disaster Response Fund (APDRF), financed by Japanese government to India to support the Government’s
emergency response to COVID-19.

172. The Power Finance Corporation signed agreement with which IIT for training, research and
Entrepreneurship Development (ED) in smart grid technology?
1) IIT Delhi
2) IIT Kanpur
3) IIT Madras

Report Errors in the PDF - ebooks@affairscloud.com Copyright 2014-2020 @ AffairsCloud.Com 131


Banking & Economy PDF
4) IIT Ahmedabad
5) IIT Calcutta
Answer-2) IIT Kanpur
Explanation:
The PFC (Power Finance Corporation) signed an agreement with the Indian Institute of Technology- Kanpur (IIT-K)
in order to provide training, research and Entrepreneurship Development (ED) in Smart grid technology. Objective:
To provide support for the IIT-Kanpur to develop the infrastructure of the research and development on smart grid
technology. Budget: The PFC will provide financial assistance of Rs. 2.38 crore to the IIT-K under its CSR initiative.

173. Which edition of annual meeting of the board of governors of Asian Infrastructure Investment Bank
(AIIB) held recently?
1) 5th
2) 2nd
3) 7th
4) 9th
5) 15th
Answer-1) 5th
Explanation:
The 5th Annual Meeting of the Board of Governors of Asian Infrastructure Investment Bank (AIIB) was held virtually
from Beijing, China, was originally planned to be held in-person there, focusing on “Connecting for Tomorrow”. The
meet saw the re-election of Jin Liqun as AIIB President, and a roundtable discussion on the theme “AIIB 2030-
Supporting Asia’s Development over the Next Decade”. Notably, India’s Union Minister of Finance & Corporate Affairs
Nirmala Sitharaman was the Lead Speaker at the roundtable discussion.

174. Which country is going to host the 6th annual meeting of Asian Infrastructure Investment Bank (AIIB) in
2021?
1) Azerbaijan
2) Australia
3) UAE
4) Cambodia
5) Cyprus
Answer-3) UAE
Explanation:
During the meet it was also decided that the sixth Annual Meeting will be held in Dubai, United Arab Emirates (UAE)
on Oct. 27-28, 2021 which will be the first annual meeting in the Middle East. It will take place in parallel with the
Dubai World Expo. UAE was also one of the founding members of AIIB.

175. How many approved members were present in Asian Infrastructure Investment Bank (AIIB) as on July
2020?
1) 112
2) 110
3) 103
4) 98
5) 89
Answer-3) 103
Explanation:
During the 5th Annual Meeting of the Board of Governors of Asian Infrastructure Investment Bank (AIIB), approved
the application of Liberia to join the Bank. With this AIIB now has 103 approved members from around the world.
Also, now AIIB has 10 members and nine prospective members from Africa.

Report Errors in the PDF - ebooks@affairscloud.com Copyright 2014-2020 @ AffairsCloud.Com 132


Banking & Economy PDF
176. The government plans to increase the fund allocation of education sector to ______% of GDP.
1) 4%
2) 5%
3) 6%
4) 2%
5) 3%
Answer-3) 6%
Explanation:
Union Cabinet chaired by the Prime Minister Narendra Modi approved the first education policy of the 21st century
i.e. National Education Policy (NEP) 2020 which has replaced 34 years old NPE, 1986. This was announced by Union
Minister of Information and Broadcasting Prakash Keshav Javadekar. With this, the Ministry of Human Resource
Development (MHRD) will officially be renamed the Ministry of Education (MoE). It should be noted that from now
onwards the education sector will get allocation of 6% of total Gross Domestic Product (GDP).

177. The central government targets to achieve ______ % Gross Enrolment Ratio (GER) in higher education by
2035.
1) 30%
2) 40%
3) 35%
4) 50%
5) 45%
Answer-4) 50%
Explanation:
This new policy aligned to the Sustainable Development Goal (SDG)-4 of 2030 Agenda is aimed at Universalization of
Education from pre-school to secondary level with 100 % Gross Enrolment Ratio (GER) in school education by 2030.
Policy also aims to achieve 100% youth and adult literacy. GER in higher education is to be raised to 50% by 2035
with the addition of 3.5 crore seats.

178. Which world organization along with Pure Earth released the report titled “The Toxic Truth: Children’s
Exposure to Lead Pollution Undermines a Generation of Future Potential”?
1) International Monetary Fund (IMF)
2) United Nations Industrial Development Organization (UNIDO)
3) United Nations Children’s Fund (UNICEF)
4) Food and Agriculture Organization (FAO)
5) United Nations Educational, Scientific and Cultural Organization (UNESCO)
Answer-3) United Nations Children’s Fund (UNICEF)
Explanation:
United Nations Children’s Fund (UNICEF) and Pure Earth, international non-profit organization authored and
published the first of kind report on the health crisis titled “The Toxic Truth: Children’s Exposure to Lead Pollution
Undermines a Generation of Future Potential” states that around lead poisoning affects large number of children all
across the globe. The report states that around 1 in every 3 that is around 800 million children across the world has
the blood lead level above or at 5 micrograms per decilitre, the amount at which action is required and Almost 50% of
these children live in South Africa.

179. Which company has collaborated with Fiserv, Inc. for digital transformation?
1) PNB Financial Solutions Limited
2) IOB Financial Solutions Limited
3) SBI Financial Solutions Limited
4) BOB Financial Solutions Limited
5) None of the Above

Report Errors in the PDF - ebooks@affairscloud.com Copyright 2014-2020 @ AffairsCloud.Com 133


Banking & Economy PDF
Answer-4) BOB Financial Solutions Limited
Explanation:
BOB Financial Solutions Limited (BFSL) has chosen Fiserv, Inc. to enable the digitization of their end-to-end card
issuance and processing cycle and support the launch of several new and high-tech products including contactless
credit cards, virtual credit cards among others. For this purpose, BFSL will utilize Fiserv’s FirstVisionTM, an end-to-
end managed services solution.

GA Questions Asked in Exams

• GA Questions Asked in IBPS Clerk Mains Exam 2020 – 90% Questions Covered by AC
• GA Questions asked in SBI PO 2019 Main Exam
• GA Questions asked in SBI Clerk Main Exam 2019
• GA Questions asked in IBPS PO Main Exam 2019
• GA Questions asked in RBI Grade B Prelims Exam 2019
• GA Questions asked in IBPS RRB Clerk Mains Exam 2019
• GA Questions asked in IBPS RRB PO Mains Exam 2019

All The Best for Your Exams!!!

Suggestions & Feedback are welcomed

Support@affairscloud.com

Report Errors in the PDF - ebooks@affairscloud.com Copyright 2014-2020 @ AffairsCloud.Com 134

You might also like